You are on page 1of 259

WWW.BANKEXAMSTODAY.

COM

REASONING
MADE EASY
BY RAMANDEEP SINGH

SECOND EDITION
Table of Contents
Seating Arrangement .................................................................................................... 1
Puzzles ........................................................................................................................ 29
Syllogism .................................................................................................................. 116
Blood Relation ........................................................................................................... 170
Coding Decoding ....................................................................................................... 190

Inequality ................................................................................................................. 209

Problem Based on Ages ............................................................................................. 237

Order and Ranking .................................................................................................... 242

www.bankexamstoday.com Page 1
Seating Arrangement

Key points to Remember:


Facing CenterLeft→ Clockwise Direction
Right→ Anti-Clockwise Direction
Facing Outside
Left→ Anti-Clockwise Direction
Right→ Clockwise Direction

Direction:

 Eight executives J, K, L, M, N, O, P and Q are sitting around a circular table for a


meeting. All are facing Center
 J is second to the right of P, who is third to the right of K
 M is second to the left of O, who sits between P and J
 L is not a neighbor of K or N

Step 1: Assign Number from 1 to 8

Step 2: J is second to the right of P, who is third to the right of K( here who represents P)

www.bankexamstoday.com Page 2
Step 3: M is second to the left of O, who sits between P and J (First assign place for O who
sits between P and J )

Step 4: L is not neighbour of K nor N implies ( L is not neighbour of K implies clearly his
position as 8. Again L is not neighbour of N implies clearly his position as 5)

www.bankexamstoday.com Page 3
Step 5: Finally one who sits in the 7th position will be Q.

Q.1: Who is to the immediate left of L?


a. Q
b. O
c. K
d. N
e. None of these

Q.2: Who is the immediate left of K?


a. N

www.bankexamstoday.com Page 4
b. J
c. Q
d. Cannot be determined
e. None of these

Q.3: Which of the following is the correct position of N?


a. Second to the right of K
b. To the immediate left of K
c. To the immediate right of M
d. To the immediate right of K
e. None of these

Q.4: Who is third to the right of P?


a. L
b. J
c. O
d. N
e. None of these

Q.5: Which of the following groups of persons have the first person sitting
between the other two?
a. PJO
b. OPJ
c. OPM
d. MPO
d. None of these

Directions

 Eight persons J, K, L, M, N, O, P and Q are sitting around a circular area at equal


distances between each other, but not necessarily in the same order.
 Some faces center while some faces outside.
 O sits second to the left of M
 M faces the center
 K sits to the immediate left of O
 Only three persons sit between K and J
 P sits third to the left of J
 L sits to the immediate Left of N
 Immediate neighbours of K face Opposite directions
 N sits third to the right of K
 Both N and L face opposite directions
 Q faces the same direction as K
Step 1: Assign Number from 1 to 8 and a tabular column for those who sits facing center
and outside

www.bankexamstoday.com Page 5
Step 2: M faces Center
Step 3: O sits second to the left of M

Step 4: Only 3 persons sit between K and J (we don’t know whether they seated facing
center or outside. So there are 2 possibilities for their positions)

Step 5: P sits third to the left of J (though we don’t know the direction J faces, there are 4
possible positions for P)

Step 6: Since 1st and 3rd positions are already allotted to M and O respectively, Exact
Position of K, J and P will be 4, 8 and 5 respectively.

www.bankexamstoday.com Page 6
Step 7: Since P is 3rd to the left of J, the above diagram clearly implies that J is facing
outside.

Step 8: K sits to the immediate left of O implies O is facing center

Step 9: Immediate neighbours of K faces opposite direction Implies, P faces outside

Step 10: N sits 3rd to the right of K (there are 2 possible positions for N )

From the above tabulation, position 1 is already allotted to M, so the exact position of N will
be 7)

www.bankexamstoday.com Page 7
Step 11: From the above diagram, N sits third to the right of K implies K faces outside and
Q faces same direction as K implies Q also faces Outside.

Step 12: L sits to the immediate Left of N implies 6th is the only position of L and also N
faces Outside

www.bankexamstoday.com Page 8
Step 13: Both N and L face opposite directions Implies L faces center, and from the above
diagram it is clear that 2nd position is for Q.

Q.6: Who sits exactly between O and P, when counted from the left of O?
a. Q
b. K
c. L
d. N
e. J

Q.7: Four of the following five are alike based on the given seating arrangement
and so form a group. Which is the one that does not belong to that group?
a. P
b. J
c. Q
d. N
e. L
(Since, P, J, Q and N Faces outside while L faces Center)

Q.8: Who sits second to the left of J?


a. M
b. K
c. L
d. P
e. Q

Q.9: What is K‟s position with respect to N?

www.bankexamstoday.com Page 9
a. Second to the left
b. Immediate left
c. Immediate right
d. Third to the left
e. Third to the right

Q.10: How many persons in the given arrangement face the center?
a. Three
b. Two
c. Four
d. Five
e. One

Directions:

 Eight Friends A, B, C, D, E, F, G and H are sitting around a circle facing the center
 A sits fourth to the left of D
 C Sits second to left of A
 E is not an immediate neighbour of D and A
 H sits second to the left of B
 B is not an immediate neighbour of A and E
 G sits second to the left of F

Q.11: In which of the following groups of people is the first person sitting exactly
in the middle of the second and the third persons?
a. GCB
b. ADF
c. BCG
d. HDE
e. None

Q.12: Which of the following pairs represents the immediate neighbours of B?


a. EH
b. AE
c. CD
d. AG
e. None

Q.13: Who sits third to the left of E?


a. B
b. G
c. A
d. D
e. None

www.bankexamstoday.com Page 10
Q.14: Starting from A, If all the friends are made to sit in the alphabetical order of
their names in the anti- clockwise direction, the positions of how many(except A)
will remain unchanged?
a. None
b. One
c. Two
d. Three
e. Four

Q.15: Four of the following five are alike in a certain way based on their positions
in the above arrangement and so form a group. Which is the one that does not
belong to that group?
a. HF
b. GB
c. ED
d. AC

Direction:

In a family there are 8 persons P, Q, R, S, T, U, V, W sitting around a circular table facing the centre. There are 2 couples
in a family. No couple is sitting together. There are equal number of male and female in the family.
S is the son of T and sitting opposite of his father. S is sitting between P and his sister. U is sitting second to the left of V.
U is the husband of T. Q, the father of P, is sitting opposite to P. V is the grandmother of W. R is sitting third to the right
of T. R is the brother of V and brother in law of Q.

Q.16. How is V related to S?


A. Mother
B. Daughter
C. Aunt
D. Grandmother
E. None of these

Q.17. What is the position of W regarding to V?


A. Fourth to the right
B. Fifth to the left
C. Third to the right
D. Second to the left
E. None of these

Q.18. How many male members in the family?


A. 2
B. 4
C. 3
D. 5

www.bankexamstoday.com Page 11
E. 6

Q.19. Who among the following is one of the couples?


A. SW
B. PQ
C. VQ
D. RV
E. None of these

Q.20. If all the persons are made to sit in alphabetical order in clockwise direction,
starting from P, the position of how many (excluding P) will remain unchanged as
compared to their original sitting position?
A. 2
B. 5
C. 3
D. 4
E. 1

Four Boys Nishanth, Satheesh, Aravind and Prakash and Four girls Anitha, Nisha,
Kamatchi and Kani are sitting around a circular table.
1. Two girls and two boys are not facing the centre.
2. No three girls are sitting together
3. Aravind is second to the left of Nisha, who is not sitting next to Satheesh.
4. Anitha is third to the left of Kani and one of them is not facing the centre.
5. Nishanth is third to the right of Satheesh, who is facing the centre.
6. Kamatchi and Prakash are facing each other and both are not sitting next to Satheesh
or Kani

Q.21. Who among the following is sitting between Nishanth and Nisha?
A. Satheesh
B. Anitha
C. Kani
D. None
E. None of these

Q.22. Who is sitting third to the left of Aravind?


A. Anitha
B. Kani
C. Nishanth
D. Kamatchi
E. None of these

Q.23. Which of the following groups of four members is not facing the centre?
A. Aravind, Anitha, Nisha and Prakash

www.bankexamstoday.com Page 12
B. Nishanth, Kani, Kamatchi and Prakash
C. Nishanth, Aravind, Anitha and Kani
D. Anitha, Nisha, Aravind and Nishanth
E. None of these

Q.24. Four of the Following Five are alike in some way, find the odd one?
A. Kani
B. Aravind
C. Kamatchi
D. Anitha
E. Nishanth

Q.25. Which pair is facing opposite to each other?


A. Kamatchi and Nisha
B. Kani and Anitha
C. Nishanth and Satheesh
D. Satheesh and Nisha
E. None of these

Q.26. If the persons sitting opposite, interchange their positions, then who will be
immediate neighbours of Kamatchi?
A. Anitha and Nishanth
B. Aravind and Nisha
C. Kani and Anitha
D. Anitha and Nisha
E. None of these

Q.27. Anitha and Aravind are immediate neighbours of which person?


A. Kamatchi
B. Prakash
C. Satheesh
D. Kani
E. None of these

Q.28. Who are sitting 5th to the left of Kani and 3rd to the right of Nisha
respectively?
A. Kamatchi and Aravind
B. Aravind and Satheesh
C. Satheesh and Aravind
D. Aravind and Kamatchi
E. None of these

Q.29. If the position of Nisha and Aravind gets interchanged, then who will be 3rd
to left of Nishanth?

www.bankexamstoday.com Page 13
A. Prakash
B. Nisha
C. Aravind
D. Satheesh
E. None of these

Q.30. How many Boys are sitting on the right of Kani and left of Kamatchi?
A. One
B. Two
C.Three
D. Four
E. None of these

Eight Persons A, B, C, D, E, F, G and H are sitting around a circular table facing the
center but not necessarily in the same order. Each one is working in different Public and
Private sector banks namely, SBI, Canara bank, PNB, IDBI, Dena, Axis, ICICI and HDFC
Bank.
1. A sits third to right of the one who works in HDFC bank2. Only two people sit between
person who work in HDFC and H
3. Those who work in ICICI bank and Axis bank are immediate neighbours of each
other.
4. Neither A nor H works in ICICI or in Axis bank.
5. One who works in ICICI bank is not an immediate neighbour of HDFC bank
employee.
6. IDBI bank employee sits second to left of E
7. E is not an immediate neighbour of H
8. The IDBI bank employee is an immediate neighbour of both SBI and PNB employee.
9. PNB employee sits third to right of B
10. B doesn‘t work in ICICI bank
11. C sits to the immediate right of the Dena bank employee
12. A doesn‘t work in Dena bank
13. F is not an immediate neighbour of A
14. G is not an immediate neighbour of IDBI employee

Q.31. Who among the following sits third to the left of E?


A. IDBI Employee
B. G
C. A
D. PNB Employee
E. B

Q.32. Four of the following five are alike in a certain ay based on the given
arrangement and thus form a group. Which is the one does not belong to that

www.bankexamstoday.com Page 14
group?
A. F- Dena Bank
B. G- Canara Bank
C. D- IDBI Bank
D. A- PNB Bank
E. B- HDFC Bank

Q.33. Who among the following works in Canara Bank?


A. A
B. C
C. H
D. G
E. D

Q.34. Which of the following is true with respect to the given seating
arrangement?
A.A sits second to the left of B
B.H and E are neighbours of each other
C. Dena Bank and SBI Employees are neighbours of each other
D. HDFC Employee sits opposite to A
E. G is an immediate neighbour of both A and C

Q.35. Which of the following bank does B works?


A. HDFC Bank
B. Canara Bank
C. SBI
D. Axis Bank
E. Dena Bank

Direction:-10 persons M, T, R, Q, L, J, O, P, N, S are sitting in a straight line. Some of them


facing north while others facing south. (Note: Facing the same direction means, if one is
facing north then the other also faces north and vice-versa. Facing the opposite direction
means, if one is facing north then the other faces south and vice-versa)
Q is the third to the right of J who sits at any of the extremes ends of the line. The
neighbours of Q faces opposite directions. S is the second to the right of Q. P is not an
immediate neighbour of Q. The face of S is an as same direction as Q. T is the immediate
left of M. Q faces north. M is the immediate neighbour of S. L is the immediate left of P. P
and Q faces opposite direction. L does not sit at any ends of the line. The person between J
and N is as same as between N and O. R is the seventh to the left of L. O is not facing
south. The immediate neighbours of O face same direction as O.

Q.36. How many persons, whose facing to the south are sitting to the left of S?
A. 4
B. 5

www.bankexamstoday.com Page 15
C. 3
D. 6
E. None of these

Q.37. Who is sitting exactly between P and S?


A. N
B. Q
C. M
D. T
E. None of these

Q.38. How many persons are facing north?


A. 4
B. 5
C. 8
D. 6
E. None of these

Q.39. Who amongst the following sit at the extreme ends of the rows?
A. J,O
B. P,M
C. J,R
D. P,Q
E. None of these

Q.40. Which of the following is true regarding T?


A. T is third to the right of N.
B. T is sitting between O and S.
C. Q is fourth to the left of T.
D. T faces South.
E. None of these.

Q.41. Who is to the immediate left of L?


a. Q
b. O
c. K
d. N
e. None of these

Q.42. Who is the immediate left of K?


a. N
b. J
c. Q
d. Cannot be determined

www.bankexamstoday.com Page 16
e. None of these

Q.43. Which of the following is the correct position of N?


a. Second to the right of K
b. To the immediate left of K
c. To the immediate right of M
d. To the immediate right of K
e. None of these

Q.44. Who is third to the right of P?


a. L
b. J
c. O
d. N
e. Cannot be determined

Q.45. Which of the following groups of persons have the first person sitting
between the other two?
a. PJO
b. OPJ
c. OPM
d. MPO
d. None of these

Direction: Eight Persons P, Q, R, S, T, U, V and W are sitting around a circular table facing
the center but not necessarily in the same order. Each one has different weight such as 50
Kg, 54 Kg, 60 Kg, 65 Kg, 68Kg, 72 Kg, 74 Kg and 78 Kg, but not necessarily in the same
order.

1. P is only greater than S


2. W, whose weight is 72Kg and One with least weight are opposite to each other
3. T is shorter than only V, who sits 3rd to the left of U
4. P and V are opposite to each other
5. Q is greater than R and Lesser than T
6. Immediate neighbour of U and Q is the one with least weight
7. R is not an immediate neighbour of V or Q
8. sum of U and T is 139

Q.46. Which of the following is true?


A. S<P<R<U<W<Q<T<V
B. V>T>W>U>Q>R>P>S
C. V<T<Q<W<U>R>P<S
D. V>T>W>Q>U>R>P>S

www.bankexamstoday.com Page 17
E. None of these

Q.47. What is the exact value of (P+Q)* (V+T)/(R-S)?


A. 1854.4
B. 1855.4
C. 1854.8
D. 1855.8
E. None of these

Q.48. If all are asked to sit in alphabetic order starting from P in Anti-clockwise
direction, then how many of them remain in same place?
A. None
B. One
C. Two
D. Three
E. None of these

Q.49. Which of the following are 3rd to the right of T and 2nd to the left of U
respectively?
A. P and Q
B. S and V
C. S and Q
D. P and R
E. None of these

Q.50. Who among the following are heavier than Q?


A. T and W
B. W and U
C. V and U
D. U and T
E. None of these

Q.51. Who is sitting between person with 60 Kg and 74 Kg?


A. Q
B. R
C. S
D. W
E. None of these

Q.52. How many persons are sitting between P and U when counted in clockwise
direction?
A. None
B. One
C. Two

www.bankexamstoday.com Page 18
D. Three
E. None of these

Q.53. Find the odd one from the following options?


A. W and P
B. R and S
C. U and Q
D. S and V
E. V and W

Answers
Ans. 1. None of these

Ans. 2. Q

Ans. 3. To the immediate right of K

Ans. 4. L

Ans. 5. OPJ

Ans. 6. K

Ans. 7. L

Ans. 8. L

Ans. 9. Third to the left

Ans. 10. Three

Ans. 11. D

Ans. 12. C

Ans. 13. B

Ans. 14. A

Ans. 15. C

Ans. 16. D

www.bankexamstoday.com Page 19
Ans.17. C

Ans.18. B

www.bankexamstoday.com Page 20
Ans. 19. C

Ans. 20. E

www.bankexamstoday.com Page 21
Ans. 21. Option C

Ans. 22. Option D

Ans. 23. Option D

Ans. 24. Option B

Ans. 25. Option E

Ans. 26. Option A

Ans. 27. Option C

Ans. 28. Option D

Ans. 29. Option A

Ans. 30. Option C

Steps to solve:

www.bankexamstoday.com Page 22
Step 1: from 5, we get

Step 2: From 6, we get

Facing Centre
Boys- Satheesh, Prakash
Girls- Kamatchi

Step 3: From 3, we get

Step 4: From 4, 6 and 2, we can come to a conclusion

www.bankexamstoday.com Page 23
Facing Centre
Boys- Satheesh, Prakash
Girls- Kamatchi, Kani

Ans. 31. D

Ans. 32. E

Ans. 33. A

Ans. 34. C

Ans. 35. D

Solution

Step 1: From 1, we get

Step 2: From 2, we get

www.bankexamstoday.com Page 24
Step 3: From 3, 4 and 5 we get,
ICICI Bank Axis Bank

2 3

6 5

Step 4: From 6, 7 and 8 we get,


E IDBI SBI/PNB

4 6 5/7

5 7 6/8

Step 5: by comparing step 3 and step 4, we get


E IDBI SBI PNB B Axis ICICI

5 7 6 8 3 3 2

Step 6: from 12, we get


 E works in Dena Bank
 A works in Canara Bank

www.bankexamstoday.com Page 25
Step 7: From 11, 13 and 14: we can come to a conclusion
C F G D

4 6 2 8

Ans. 36. C

Ans. 37. B

Ans. 38. D

www.bankexamstoday.com Page 26
Ans. 39. C

Ans. 40. C

Ans. 41. a

Ans. 42. d

Ans. 43. e

Ans. 44. e

Ans. 45. e

Ans. 46. D

Weight Person

50 Kg S

54 Kg P

60 Kg R

65 Kg U

68 Kg Q

72 Kg W

74 Kg T

78 Kg V

Ans. 47. A
(P+Q)* (V+T)/(R-S)
(54+68)*(78+74)/(60-50)
122*152/10
18544/10
1854.4 (ans)

Ans. 48. B
www.bankexamstoday.com Page 27
P alone remains in same place

Ans. 49. C

Ans. 50. A
Weight Person

50 Kg S

54 Kg P

60 Kg R

65 Kg U

68 Kg Q

72 Kg W

74 Kg T

78 Kg V

Ans. 51. D

Ans. 52. A

www.bankexamstoday.com Page 28
Ans. 53. B
All other have single person in between except Option B

Puzzles

There are 8 persons - A ,B,C,D,E,F,G and H standing on gates of 2 rectangular parks.


The rectangular parks are such that one is small and second large with small inside the
larger one. The gates are on the 4 sides of the park. The ones on the outer park are
standing facing center and ones on inner-park are standing facing outside the center. So
in this way, the persons are facing each other when they are standing on the same side
of the park. There is one person standing between B and D. C faces B. A is to the
immediate right of C. G is not either to the immediate right or immediate left of D. G
faces neither D nor F . One person is standing between H and F. E is facing the center.
(2 MARKS )

Q.1: F is facing which of the following ?


(a)C
(b)E
(c ) H
(d)A
(e)B

Q.2: Which of the following does not form part of the group ?
(a)D
(b)B
(c)H
(d)G
(e)F
First 12 even numbers are written from top to bottom .The letters of the word ―SACRED‖
are written in alphabetical order against each multiple of 4 (one letter against one

www.bankexamstoday.com Page 29
number).There are two letters between N and S . There are as many letters between E
and N as between P and D. P is not against number 14. There are 5 letters between U
and T . U is above T. I is written against number 6. (no letter is repeated against any
number)

Q.3: Which is the second letter in the word formed by letters against number 6
, 12, 14 and 20 ?
(a)T
(b)D
(c) N
(d)I
(e)R

Q.4: If there are 3 alphabets in English alphabetical series between alphabets


written against numbers 10and 22, then how many alphabets in English
alphabetical series are there between the alphabets written against 18 and 22?
(a) 3
(b)5
(c )1
(d)4
(e)cannot be determined

Directions: Read the following information carefully and answer the questions given
below:
There are six cars – P, Q, R, S, T, U – parked in a row facing north direction, but not
necessarily in the same order. The distances between two adjacent cars are successive
multiples of three (i.e., if the distance between the first and the second car is 3 m, first
and the third cars are 6 m and between first and fourth cars is 9 m and so on.)
 The distance between car T and car U is 51 m. Car T is to the left of U but
not to the immediate left of U.
 Car S is to the immediate right of Q and the distance between car Q and S
is 33 m.
 The distance between car Q and T is 81 m.
 P is to the immediate left of T and the distance between car P and T is 21
m.
 Car „S‟ starts moving towards the north and after going 20 m, it turns left,
then it moves 63 m and then it turns to the left and covers the distance of
8 m. and stops at point „A‟.
 Car „T‟ moves 27 m towards south direction, and then takes a left turn and
goes 51 m straight. Then it turns again to its left direction and moves
another 12 m and stops at point „B‟.

www.bankexamstoday.com Page 30
Q.5. Which of the following statement is true?
A. Distance between point A and car U is 10 m.
B. Distance between car R and U is 30 m.
C. There is only 1 car between car T and Q.
D. Car R is to the immediate left of car U.
E. None is true.'

Q.6. How many cars are parked between T and Q?


A. More than three
B. One
C. Two
D. Three
E. None

Q. 7. What is the distance between point A and point B?


A. 30 m
B. 18 m
C. 25 m
D. 20 m
E. 27

Q.8. If car „Y‟ is parked at 72 m to the west of point A then what is distance
between Car Y and Car P?
A. 18 m
B. 10 m
C. 12 m
D. 15 m
E. 16 m

Q.9. In which direction is car „R‟ with respect to point „B‟?


A. East - North
B. West - North
C. North
D. South - West
E. East

Direction: Study the following information carefully to answer the given


questions.
There is an apartment in Delhi. The eight members have decided to play some games
around a round table facing to the canter, so their eight members are P, Q, R, S, W, X, Y
and Z . Each person has different cars viz; Swift, i20, Honda City, Duster, Accent,

www.bankexamstoday.com Page 31
Bolero, Scorpio and Dustan Go, but not necessarily in the same order and these 8
members live on different floor from floor number 1 to 8.
Z lives on top most floor and P lives on first floor. The one who has Duster lives on 2
floor. The person who has i20, lives on fifth floor and sits third to the right of Y. R lives
on 3rd floor and sits immediate neighbour of Y. The person who has Duster sits second
to the right of R. Q sits third to the right of Z. Z‘s car is neither i20 nor Duster. Only one
person sits between R and the person who has Honda city. The one who has Swift, lives
on immediate floor below from the one has Dustan Go. P and X are immediate
neighbours of each other. Neither P‘s nor X‘s has i20. The person who has Dustan Go
sits second to the right of P. Two persons sit between S and the person who has Accent.
S has not i20. The one who has Honda city lives on 6th floor. The person who has Swift
is not an immediate neighbour of the person who has i20. The person who has Bolero
sits second to the left of P. The one who has Bolero, lives on just floor above from on
which floor R lives.

Q.13. S lives on which floors?


A. 1
B. 3
C. 4
D. 2
E. 7

Q.14. Who is the immediate left neighbor of S ?


A. The one who has Scorpio
B. The one who has Swift
C. The one who has Bolero
D. The one who has Honda City
E. The one who has Duster

Q.15. Who sits exactly between the persons who have Honda City and Accent
(clockwise from R)?
A. R and Z
B. R and Q
C. Only one person, Z
D. P and Q
E. None of these

Q.16. Who among the following has Scorpio?


A. P
B. W
C. R
D. X

www.bankexamstoday.com Page 32
E. S

Q.17. What is the position of S with respect to the person who has i20?
A. Second to the left
B. Second to the right
C. Immediate right
D. Immediate left
E. None of these

Directions:There are 8 persons – P, Q, R, S, T, U, V and W standing on 2 gates of 2


rectangular parks. The rectangular parks are such that one is small and second is large
with small inside the larger one. The gates are on the 4 side of the park. The ones on
outer park are standing facing center and ones on Inner Park are standing facing outside
the center. So in this way, the persons are facing each other when they are standing on
same side of parks.
 There is one person standing between P and T.
 S faces P.
 R is to the immediate right of S.
 Q is not either to the immediate right or immediate left of T.
 Q faces neither T nor V.
 One person is standing between W and V.
 U is facing the center.

Q.18. V is facing which of the following persons?


A. T
B. U
C. P
D. R
E. W

Q.19. Who is the immediate right of R?


A. P
B. U
C. T
D. Q
E. S

Q.20. Which of the following does not form a part of group?


A. T
B. U
C. W

www.bankexamstoday.com Page 33
D. P
E. V

(26 – 30) Study the following information carefully to answer the given
questions.
There are seven friends namely L, M, N, O, P, Q, R and S are going to movies on
different days i.e. Tuesday, Wednesday, Monday, Thursday, Friday, Sunday and
Saturday, but not necessarily in the same order. Each person have an Birthday, in seven
different months (of the same year) namely February, May, April, June, September,
March and November but not necessarily in the same order. Q has a Birthday in the
month which has more than 30 days. Both R and N have a Birthday in a month after the
one who goes market on Tuesday. The one who goes market on Monday has a Birthday
in the month which has less than 30 days. Only three people have a Birthday between
the one who goes market on Monday and the one who goes market on Saturday. R has
a Birthday immediately before N. Only two people have a Birthday between R and the
one who goes market on Thursday. O has a Birthday immediately after the one who
goes market on Thursday. Only one person has a Birthday between Q and the one who
goes market on Tuesday. Only two people have a Birthday between O and P. L has a
Birthday immediately before the one who goes market on Wednesday. N does not go on
Sunday.
Q.26. As per the given arrangement Monday is related to April and Thursday is
related to September following a certain pattern, which of the following is
Saturday related to following the same pattern?
A. February
B. June
C. March
D. November
E. May

Q.27. Which of the following represents the people who have a Birthday in April
and November respectively?
A. M, L
B. P, L
C. P, N
D. M, N
E. M, R

Q.28. How many people have a Birthday between the months in which P and L
have a Birthday?
A. None
B. One
C. Three

www.bankexamstoday.com Page 34
D. Two
E. More than three

Q.29. Which of the following represents the month in which R has a Birthday?
A. Cannot be determined
B. March
C. May
D. April
E. September

Q.30. In which of the following days does N go to the market?


A. Tuesday
B. Wednesday
C. Thursday
D. Friday
E. Saturday

Direction: Study the following information carefully to answer the given questions.
In a coaching institute Seven students i.e. P, R, S, T, U and V are doing preparation for
competitive exam, so all the students have their different exams in different days of
week from Monday to Sunday in a particular same month. And their exams are IBPS,
RRB, RBI, SSC, Railway, Delhi Metro and SBI. P is giving exam just before S‘s exam.
V is not giving Railway‘s exam. There are 4 days gap between SSC exams‘ and RRB‘s
exam. The number of students, who are giving exams before when R is giving, is same
number of students giving exams after R‘s exam. There are two days gap between when
S is giving exam and When RBI‘s exam will conduct. Immediate after SSC‘s exam, R is
giving exam. After RRB‘s exam, SBI‘s exam will conduct. On Saturday Q is giving his
RRB exam. There is 2 days gap between RRB‘s exam and Railway‘s exam. IBPS exam
will conduct on 4th day of a week. There is 2 days gap between Delhi Metro exam and
U‘s exam.

Q.31. Who among the following is his exam on Wednesday?


A. S
B. R
C. P
D. T
E. U

Q.32. How many students is giving exam between Railway‟s exam and V‟s
exam?
A. Five
B. Three
www.bankexamstoday.com Page 35
C. Four
D. One
E. None of these

Q.33. Who among the following is giving exam of Railway‟s?


A. U
B. Q
C. S
D. R
E. T

Q.34. How many students are giving exam before when U is giving?
A. Two
B. Three
C. Four
D. One
E. No one

Q.35. Which exam, S is giving on Tuesday?


A. Delhi Metro
B. RBI
C. RRB
D. IBPS
E. SBI

Direction: Study the following information carefully to answer the given


questions.
In Delhi University there are seven students i.e. A,B,C,D,E,F,G,H and I live in a hostel
which having nine storey-building. They live on different floors. Lowermost floor of
building is numbered 1, one above that is numbered 2 and so on till top most floor is
numbered 9. They all have different branded Mobile phone which name is Nokia,
Samsung, Gionee, Oppo, Vivo, Sony, Lenevo, Asus and Apple.
There are five floors between G and the one who has Asus. D does not lives on top floor.
The one which has Nokia lives immediate below the one which has Lenevo. The one
which has Lenevo lives on even numbed floor. The one who has Gionee lives on Odd
numbered floor but below from 6th floor. B lives on 6th floor and has Samsung. The one
who has Sony lives in top floor. The one who has Vivo lives below from the one who has
Gionee. The one who has Apple lives on 2nd floor. I lives above from H. A lives
immediate below the one who has Sony. F lives immediate above floor from the one who
has Asus. The one who lives on odd numbered floor, have Vivo. There are 2 floors
between D and B. The number of persons who live above E‘s floor, is same number of
person who lives below from him/her (E). C lives on even numbered floor and has Oppo.

www.bankexamstoday.com Page 36
Q.36. Who among following has Vivo?
A. I
B. D
C. H
D. B
E. E

Q.37. How many members live between C and The one who has Samsung?
A. Five
B. Three
C. One
D. Four
E. No one

Q.38. Who lives on 2nd Bottom most floors?


A. F
B. C
C. D
D. E
E. H

Q.39. Who among has Apple Mobile?


A. E
B. I
C. H
D. F
E. None of these

Q.40. The one who has Lenevo, lives on which floor?


A. 5th
B. 7th
C. 1st
D. 8th
E. 4th

Direction: Study the following information carefully to answer the given


questions.
After 8 years of college, eight friends i.e. P, Q, R, S, T, U, V, and W has decided to meet
at dinner in a Restaurant at Mall road in Manali. On 7th June, they sit around a circular
table facing outside. Each of them is working in different city- Delhi, Mumbai, Jaipur,
Bangalore, Chennai, Shimla, Chandigarh and Gurugram but not necessarily in the same

www.bankexamstoday.com Page 37
order. All friends has worn different colored shirt i.e. Yellow, Blue, Green, Black, White,
Red, Violet and Orange. The one who is working in Chandigarh, has worn Orange colored
shirt. R has worn Blue shirt and Q has worn Green shirt. The one who has worn Violet
colored shirt and The one, who has worn White colored shirt, sit opposite to each other.
T and W sit immediate diagonally opposite to each other but W is immediate neighbor of
Q. The one who is working in Bangalore and the one who is working at Gurugram sit
diagonally opposite to each other. S sits 2nd to the right of Q. The immediate neighbor
of U is P and R. The one who has worn White colored shirt, sits 2nd right of U. The one
who is working in Delhi sits diagonally opposite to the one who is working in Jaipur. Q is
not from Bangalore. V is working in Jaipur and sits immediate right of Q and has worn
Red colored shirt. The one, who sits 3rd to left of U, is working in Chandigarh. The one
who is working in Shimla sits immediate right of the who is working in Chennai. R sits
2nd right of P. The one who has worn Yellow colored shirt, sits opposite to V. The one
who is working in Mumbai sits immediate left of P.
Q.41. Who sits immediate right of the one who has worn Black shirt?
A. Q
B. R
C. S
D. V
E. U

Q.42. Who is immediate neighbor of the one who is working in Delhi?


A. The one who is Mumbai and S
B. Q, T
C. P, R
D. V, Q
E. W, R

Q.43. How many person sit between the one who has worn Green shirt and Q,
when counted anticlockwise from Q?
A. Two
B. No One
C. Three
D. Four
E. Five

Q.44. Who sits 3rd right of P?


A. W
B. P
C. R
D. V
E. None of these

www.bankexamstoday.com Page 38
Q.45. Who is working in Gurugram?
A. P
B. R
C. Q
D. V
E. T

Directions: Asha, Hema, Beena, Geeta, Meenu, Cheena, Deepa, Sheela and Veena are
nine people live on nine different floors of a building but not necessarily in the same
order. The lower most floor of the building is numbered 1, the one above that is
numbered 2 and so on till the topmost floor is numbered 9. Each of them belongs to
various states - Punjab, Madhya Pradesh, Haryana, Uttar Pradesh, Jharkhand, Gujarat,
Maharashtra, Rajasthan and Uttrakhand but not necessary in the same order. Each of
them is going to different cities like Kashmir, Ujjain, Haridwar, Nainital, Lucknow,
Shimla, Pune, Mumbai and Jaipur but not necessary in the same order.
 Two Persons live between Deepa and the one who belongs to Gujarat.
 Deepa lives an even numbered floor but not floor number 2.
 The person who is going to Pune lives immediately above the one who belongs to
Gujarat.
 Only three persons live between Meenu and the one who is going to Pune.
 Two persons live between the one who belong to Jharkhand and Meenu.
 The one who is going to Jaipur live immediately above the one who belongs to
Jharkhand.
 The one who is going to Jaipur live immediately below Beena.
 Five persons live between the one who belongs to Uttrakhand and the one who
belongs to Haryana.
 The one who belongs to Haryana live on an even numbered floor but not floor
number 2.
 Asha belongs to Uttar Pradesh and live immediately above Cheena.
 Only one person lives between the one who belongs to Uttar Pradesh and the one
who is going to Kashmir.
 One person lives between Hema and Geeta. Hema does not belong to Gujarat.
Beena is going to Lucknow.
 The one who belongs to Rajasthan is going to Nainital. Only one person lives
between the one who is going to Haridwar and the one who is going to Ujjain.
 Hema is not going to Ujjain. The one who belongs to Madhya Pradesh live
immediately above the one who belongs to Maharashtra.
 Only one person lives between Sheela and the one who is going to Shimla.

Q.46 Which of the following does belong to Maharashtra?


A. Cheena

www.bankexamstoday.com Page 39
B. Deepa
C. Hema
D. Sheela
E. Meena

Q.47 How many persons live between Veena and the one who is going to
Mumbai?
A. 4
B. 6
C. 5
D. 3
E. None

Q.48 Which of the following is correct pair?


A. Rajasthan – Ujjain - Asha
B. Punjab – Kashmir - Hema
C. Jharkhand – Pune - Hema
D. Madhya Pradesh – Lucknow - Beena
E. Maharashtra – Nainita - Deepa

Q.49. Where is Veena going?


A. Maharashtra
B. Haridwar
C. Shimla
D. Uttrakhand
E. Pune

Q.50. Which of the following statement is true?


A. Two floors between Hema and Sheela
B. Geeta is going to Gujarat.
C. Sheela lives in floor number 8.
D. Meenu belongs to Rajasthan.
E. None of these.

Direction: A, B, C, D, E, F, G, H and I are nine people living on nine different floors of a


building but not necessarily in the same order. The lowermost floor of the building is
numbered 1, the one above that is numbered 2 and so on till the topmost floor is
numbered 9. Each of them plays a different game like- Chess, Football, Kho-Kho,
Cricket, Hockey, Carrom, Table-tennis, Badminton, Tennis.

 There are 3 floors between I and the one who plays cricket.
 I lives on an even number floor.

www.bankexamstoday.com Page 40
 The one who plays Kho-Kho lives immediately below the one who plays cricket.
 2 person live between I and G. G does not like Kho-Kho. H lives immediately
above C.
 One person lives between H and A.
 The one who plays table tennis lives immediately below G. C likes Kho-Kho.
 Two persons live between the one who likes hockey and the one who likes
badminton.
 B plays hockey. B does not live immediately above H.
 The one who plays chess lives immediately above E.
 G does not play chess.
 D plays football. E plays tennis.

Q.51 Which of the following does play Carrom?


A. B
B. G
C. I
D. A
E. None of these

Q.52 Who among the following lives immediately above B?


A. The one who plays tennis
B. A
C. The one who plays table tennis
D. E
E. D

Q.53 Four of the following five are alike in a certain way and hence form a
group. Which of them does not belong to the group?
A. Carrom-B
B. Chess-A
C. Badminton-D
D. Tennis-D
E. Hockey-E

Q.54 Which of the following is not true?


A. F-Table tennis
B. H-Cricket
C. I-Chess
D. C-Hockey
E. D-Football

Q.55 C plays which of the following games?


www.bankexamstoday.com Page 41
A. Tennis
B. Badminton
C. Kho Kho
D. Hockey
E. Cricket

Direction: 10 persons M, T, R, Q, L, J, O, P, N, S are sitting in a straight line. Some of


them facing north while others facing south. (Note : Facing the same direction means, if
one is facing north then the other also faces north and vice-verse. Facing the opposite
direction means, if one is facing north then the other faces south and vice-versa)

 Q is the third to the right of J who sits at any of the extremes ends of the line.
 The neighbours of Q faces the same direction.
 S is the second to the right of Q.
 P is not the immediate neighbour of Q.
 The face of S is in the same direction as Q.
 T is the immediate left of M.
 Q faces north.
 M is the immediate neighbour of S.
 L is the immediate left of P.
 P and Q face opposite directions.
 L does not sit at any ends of the line.
 The person between J and N is as same as between N and O.
 R is the seventh to the left of L.
 O is not facing south.
 The immediate neighbours of O face the same direction as O.

Q.56. How many persons, whose facing to the south are sitting to the left of S?
A. 4
B. 5
C. 3
D. 6
E. None of these

Q.57. Who is sitting exactly between P and S?


A. N
B. Q
C. M
D. T
E. None of these

www.bankexamstoday.com Page 42
Q.58. How many persons are facing north?
A. 4
B. 5
C. 8
D. 6
E. None of these

Q.59. Who amongst the following sit at the extreme ends of the rows?
A. J, O
B. P, M
C. J, R
D. P, Q
E. None of these

Q.60. Which of the following is true regarding T?


A. T is third to the right of N.
B. T is sitting between O and S.
C. Q is fourth to the left of T.
D. T faces South.
E. None of these.

Directions: Study the following information carefully and answer the questions
given below:
A, B, C, D, E, F and H are seven persons working in a seven different cities viz. Patna,
Mumbai, New Delhi, Kolkata, Lucknow, Chandigarh and Chennai but not necessarily in
the same order. They use different mode to reach their places
, viz; Bus, Train, Aeroplane, Cycle, Bike, Car and Ship, but not necessarily in the same
way. D is not working either in Kolkata or in Patna. The person who uses Train to travel,
works in Chandigarh. A works in Lucknow. Neither B nor F works in Mumbai. The person
who works in Kolkata uses Bike to travel. B uses Bus to travel. One who uses Aeroplane
works in Mumbai. The one who works in Patna doesn't use Ship as the mode of
travelling. D uses cycle to travel. One who works in Lucknow neither uses Ship not
Train. H uses Ship to travel. C doesn't use either Train or Bike. E doesn't work in
Chandigarh. The one who lives in New Delhi doesn't use Ship.

Q.61. A uses which of the following modes of travel?


1. Train
2. Ship
3. Car
4. Bus
5. Cycle

www.bankexamstoday.com Page 43
Q.62. C lives in which of the following place?
1. Patna
2. Lucknow
3. New Delhi
4. Mumbai
5. Kolkata

Q.63. Who among the following uses Bike to travel?


1. E
2. D
3. B
4. F
5. A

Q.64. D works in which of the following cities?


1. Chennai
2. Kolkata
3. Mumbai
4. Patna
5. New Delhi

Q.65. Which of the following is matched correctly?


1. A- Lucknow - Cycle
2. B- Patna - Car
3. H - Mumbai - Bus
4. D - New Delhi - Ship
5. F- Chandigarh - Train

Q.66. H uses which of the following modes of travel?


1. Train
2. Ship
3. Cycle
4. Bus
5. Car

Directions: Study the following information carefully and answer the questions
given below:
N, P, R, S, T, V, W and Z are eight people who live on eight different floors of a building
but not necessarily in the same order. The lowermost floor is numbered one, the one
above it is numbered two and so on till the topmost floor is numbered eight. They work
in eight different banks viz, UCO Bank, SBI, ICICI Bank, PNB, Union Bank, Indian Bank,

www.bankexamstoday.com Page 44
Dena Bank and Andhra Bank but not necessarily in the same order.
They work at three different posts such that at least two persons work on the same post
but not more than three persons. These posts are PO, Clerk and Manager but not
necessarily in the same order.
 The one who works at PNB lives on an even- numbered floor.
 Only three people live between the one who works at PNB and R. Those who are
clerks live at odd - numbered floor - but not at the ground floor.
 Only one person lives between the persons who work at Dena Bank and at Andhra
Bank. There is a Clerk, between Dena Bank's employees and Andhra Bank's
employee Andhra Bank's employees lives at second floor.
 Only two people live between R and S. S does not live on the lowersmost floor. One
manager lives on the topmost floor. There is an equal gap of floors between two
pairs of floors at which the managers live. The gap is of two floors.
 Only three people live between S and the one who works at Union Bank and only
two people live between N and the one who works ad Dena Bank.
 T lives immediately above N. T lives on an even- numbered floor. N does not work
at Union Bank. N does not live either at floor number 3 or 5.
 P lives immediately above W. Only one person lives between W and one who works
at ICICI Bank. The one who works at Indian Bank lives immediately above the one
who works at UCO Bank.
 V does not work at PNB and N does not work at SBI. V does live on groundfloor and
the one who work at SBI live immediately below Dena Bank.

Q.67. Which of the following pairs represent those who live immediately above
and immediately below P?
1. W,S
2. Z,W
3. S,W
4. R,Z
5. W,Z

Q.68. In which of the following banks does N work?


1. ICICI Bank
2. PNB
3. Indian Bank
4. UCO Bank
5. SBI

Q.69. Who among the following works at Union Bank?


1. V
2. P
3. Z

www.bankexamstoday.com Page 45
4. N
5. T

Q.70. Which among the following is group of POs?


1. Z, P, R
2. N, W, V
3. P, Z, T
4. T, V, Z
5. P, V, Z

Q.71. Which of the following is matched correctly?


1. W- 3rd floor - Union Bank - PO
2. R- 2nd floor- Andhra Bank - Manager
3. S- 5th floor - UCO Bank - Clerk
4. V- 7th floor - Union Bank - Manager
5. All are true

Q.72. P lives on which of the following floors?


1. 1st floor
2. 3rd floor
3. 2nd floor
4. 4th floor
5. Other than the given options

Q.73. How many persons are there between the floors on which Z and R lives?
1. Three
2. Four
3. Two
4. One
5. None

Directions: Study the following information carefully and answer the questions
given below:
There are six National parks viz. Corbett, Gir forest, Keoladeo, Kanha, Kaziranga and
Manas have six different animals viz.
Elephant, Tiger, Rhinoceros, Deer, Wild Ass and Lion but not necessarily in the same
order. These parks are in six different states namely Assam, Manipur, Kerala, Gujarat,
MP and UP but not necessarily in the same order. Tiger is not in Gir. Lion is in the park
which is in MP but neither in Kanha nor in Keoladeo. Kaziranga is in UP but it does not
have Tiger or Wild Ass. Deer is in the park which is in Assam. Neither Corbett nor manas
are in Manipur. Kerala's National park have no Tiger. Gir forest is neither in Kerala's
National park nor has Tiger. Gir forest is neither in Kerala nor in M.P. Corbett park is

www.bankexamstoday.com Page 46
neither in Assam nor in Kerala. The elephant is in the park which is in Manipur. Deer is
neither in Corbett nor in Gir forest. Manas park has neither in Corbett nor in Gir forest.
Manas Park has neither Wild Ass nor Deer. Keoladeo is not in Assam. Corbett park has
no lion.

Q.74. Which of the following animals is in Manas Park?


1. Tiger
2. Deer
3. Lion
4. Wild Ass
5. Cannot be determined

Q.75. 'Rhinoceros' is in which park?


1. Gir forest
2. Corbett
3. Kanha
4. Keoladeo
5. Kaziranga

Q.76. 'Wild Ass' is in which of the following states?


1. U.P
2. Kerala
3. Assam
4. Manipur
5. Gujarat

Q.77. Gujarat's national park is-


1. Kaziranga
2. Manas
3. Gir forest
4. Corbett
5. Other than the given options

Q.78. 'Keoladeo' park is in which of the following states?


1. Kerala
2. U.P
3. M.P
4. Assam
5. Gujarat

Q.79. Which of the following is matched correctly?


1. Gir forest - Wild Ass - Gujarat

www.bankexamstoday.com Page 47
2. Corbett - Tiger - M. P
3. Keoladeo - Deer - Assam
4. Manas - Lion - M.P
5. Kaziranga - Elephant- U.P

Directions: Study the following information carefully and answer the questions
that follow.
There are eight persons named E, L, O, P, X, M, F and Q on different posts viz. Engineer,
Principal, Teacher, Bank Manager, Guard, Counsellor, Marketing Head and Doctor. Each
one of them has different hobbies like watching T.V., Cooking, Playing cricket, Dancing,
Singing and Reading books.

 Two persons have Reading books as their hobbies and two have hobbies playing
cricket. The rest have different hobbies.
 Engineer likes dancing
 Q loves playing cricket while X loves Singing.
 F is a Counsellor and P likes cooking.
 E is a Doctor while the one who likes cooking is a Principal.
 X is a Teacher.
 O is a Guard and likes playing cricket.
 E and M like reading books.
 The person who likes cricket is Marketing Head.
Q.80. Who is a Bank Manager?
1. L
2. M
3. P
4. E
5. Other than the given options

Q.81. What is the hobby of F?


1. Dancing
2. Watching TV
3. Reading books
4. Singing
5. Other than the given options

Q.82. What is the profession of Q?


1. Bank Manager
2. Engineer
3. Guard
4. Marketing Head
5. Other than the given options

www.bankexamstoday.com Page 48
Q.83. Dancing is the hobby of who among the following?
1. X
2. L
3. M
4. P
5. Other than the given options

Q.84. What is the profession of P?


1. Doctor
2. Teacher
3. Engineer
4. Principal
5. Other than the given options

Directions: Study the following information carefully and answer the questions
given below:
Eight persons M, N, O, P, Q, R, S and T live on a separate floor each of an 8-floor
building but not necessarily in the same order.
The ground floor is numbered 1, the first floor is numbered 2 and so on until the
topmost floor is numbered 8.
Each person likes different fruits viz, Banana, Papaya, Orange, Apple, Pineapple,
Coconut, Cucumber, Radish but not necessary in the same order.
M doesn't live on an odd numbered floor and likes neither Coconut nor Radish.
 The one who likes Coconut lives above the 8th floor.
 Q likes papaya lives on the 7th floor.
 T does not like either Orange or Coconut.
 O does not live on the lowest and topmost floor and likes Pineapple.
 The one who likes Apple lives on odd numbered floor
 S likes cucumber and lives on an odd numbered floor which is above the 4th floor.
 The one who likes Radish lives on the 6th floor.
 N lives on the 3rd floor. Neither P nor T lives on the ground floor.
 Neither O nor P likes Orange. There are only two floors above T's floor.
 The one who likes Orange lives on the first floor. O doesn't live on the fourth floor.
M does not like Pineapple and lives on immediate above N's floor.
Q.85. Who likes 'Banana'?
1. P
2. Q
3. M
4. T
5. Other than the given options

www.bankexamstoday.com Page 49
Q.86. Which combination is correct?
1. Floor no. 6-S-Cucumber
2. Floor no. 3-O- Apple
3. Floor no. 2-T- Radish
4. Floor no. 8-P-Coconut
5. Other than the given options

Q.87. At which floor, does T live?


1. First
2. Fourth
3. Third
4. Sixth
5. Cannot be determined

Q.88. How many floors are there above 'M'?


1. Four
2. Five
3. Three
4. None
5. Two

Q.89. Which fruits like by 'T'?


1. Apple
2. Orange
3. Papaya
4. Banana
5. Other than the given options

Q.90. Who likes 'Pineapple'?


1. R
2. M
3. T
4. Q
5. Other than the given options

Directions: Study the following information carefully and answer the questions
given below:

Seven person P, Q, R, S ,T , U and V like different dance forms, viz- Mask dance, Bihu,
Nati, Mandi, Bharatnatyam, Nautanki and Lavani, but not necessarily in the same
order. Each of them has a

www.bankexamstoday.com Page 50
different profession, viz- Banking, Acting, Business, Dancing, Anchoring, teaching and
Writing, but not necessarily in the same order. P is an Anchor. S likes Lavani. The
person who likes Bharatnatyam is doing a job of Acting. The person who does teaching
is fond of Nati dance. U does not like Teaching, Acting, Banking or Dancing. U does not
like Nautanki, Bihu or Mandi. Q doesn't have a profession of Business, Acting, Banking or
Dancing. Q does not like Mandi, Mask dance, Bihu or Nautanki. R does not like
Bharatnatyam and his profession is neither Banking nor Dancing. Neither S nor V does a
job a Banking. P does not like Bihu dance and mask dance. The person who does
Business likes Nautanki.

Q.91. Who among the following does a job of teaching?


1. Q
2. U
3. S
4. T
5. Can't say

Q.92. Which of the following combinations is true according to the given


information?
1. P- Mask dance- Anchoring
2. R- Nati- Teaching
3. T- Bihu- Writing
4. S- Lavani- Dancing
5. All the above

Q.93. Which of the following is true about V?


1. Bharatnatyam - Acting
2. Lavani- Writing
3. Nati - Teaching
4. Nautanki - Business
5. Mask dance - writing

Q.94. The person who has the profession of Banking, likes?


1. Mask dance
2. Bharatnatyam
3. Mandi
4. Lavani
5. Bihu

Q.95. Who among the following likes a Nautanki?


1. P
2. R

www.bankexamstoday.com Page 51
3. T
4. V
5. Other than the given options

Q.96. Which of the following Folk dance is liked by U options


1. Nati
2. Bihu
3. Mask dance
4. Nautanki
5. Other than the given options

Direction: Study the following information carefully to answer the given


question:
There are seven teams, owners L, M, N, O, P Q and R. All of them have different teams,
viz, Mumbai Indians, Delhi Daredevils, Kolkata Knight Riders, Kings XI Punjab, Chennai
Super kings, Pune Worries and Rajasthan Royals
, but not necessarily in the same order. Each of the teams has one caption, viz-
Sangkara, Flintoff, Akram, Stain, Watson, Fleming and V. Richards but not necessarily in
the same order. Captains belong to three different countries, viz, Sri Lanka, Africa and
Australia. Atleast two captains are from one country.

O has team named Delhi Daredevils, the captain of this team is Akram. N and Q selected
the captain from Sri Lanka. N has Pune worries but doesn't have captain either Flintoff
or Stain. The one who has team Rajasthan Royals has captain from Australia only with
O. P has Mumbai Indians and Captain of Mumbai Indian is Stain, who is not from
Srilanka. Watson is captain of that team which is owned by Q. Fleming is the caption of
Rajasthan Royal but neither his owner is M nor he belongs to Africa. R owns Kings XI
Punjab. Fleming is not from Africa or Srilanka. The one who owns Chennai Super King
has no captain from Africa, from where there are three captains. V. Richards is not the
captain of Kolkata Knight Riders and Belongs to Africa.

Q.97. Who among the following is from Africa?


1. Sangkara
2. Akram
3. Stain
4. Flintoff
5. Both 3) and 4)

Q.98. Which of the following captains are from Srilanka?


1. V. Richards and Stain
2. Akram, Fleeming and Virat
3. Sangkara and Watson

www.bankexamstoday.com Page 52
4. Flintoff and Stain
5. None of these

Q.99. Who among the following has captain from Africa?


1. Q and R
2. P,R and M
3. R and L
4. O and N
5. None of these

Q.100. How many captains are from Sri Lanka?


1. Two
2. Three
3. One
4. More than two
5. Can't say

Q.101. Which of the following combination is true?


1. L- Rajasthan Royals- Fleming- Sri Lanka
2. O-Pune warriors- Akram- Africa
3. P-Africa-Stain-Mumbai Indians
4. R-Kings XI Punjab- Watson- Africa
5. M- Flintoff - Srilanka - Kolkata Knight Riders

Q.102. Which of the following statements is not true?


1. Stain is the captain of Mumbai Indians which is owned by Q.
2. N is from Sri Lanka and has team Pune Warriors.
3. Rajasthan Royals has captain Fleming who is from Africa and owned by L.
4. Both 1) and 3)
5. None of these

Directions: Study the following information carefully and answer the questions
given below it.
Eight Person A, B, C, D, E, F, G and H are sitting around a circular table facing the
center but not necessarily in the same order.
Each one of them are Indian Army Officers in different rank namely- Field Marshal,
General, Lieutenant General, Major General, Brigadier, Colonel, Major and Captain, but
not necessarily in the same order.
G is second to the right of the Field Marshal. The Major General and the General are
immediate neighbours of G. C is second to the right of E who is the Brigadier. E is an
immediate neighbour of the Major General. Only one person sit between H and F. D is
third to the left of E. A sits exactly between F and D. The Major is second to the left of A.

www.bankexamstoday.com Page 53
The Lieutenant General and the Captain are immediate neighbours of the Major. H is not
the Captain.

Q.103. How many persons are sitting between A and G, when counted from the
right of G?
a. One
b. Two
c. Three
d. Four
e. Five

Q.104. Four of the following five are alike in a certain way and so from a group.
which one does not belongs to that group?
a. E – Brigade
b. H – Lieutenant General
c. F – Captain
d. D – Major
e. G - Colonel

Q.105. Who sits second to the left of H?


a. B
b. G
c. F
d. A
e. D

Q.106. Who is third to the right of F?


a. Brigadier
b. B
c. G
d. General
e. H

Q.107. What is position of C with respect to G?


a. Third to the left
b. Fourth to the left
c. Third to the right
d. Immediate left
e. Second to the left

Q.108. Who amongst of the following is the Colonel?


a. B

www.bankexamstoday.com Page 54
b. F
c. C
d. G
e. H

Directions: Study the following information carefully and answer the questions
given below it.
Seven person A , B , C , D , E , F and G are sitting in a straight line , but not necessarily
in the same order. Some of them are facing north while some of them facing south.
Only two people are sitting to the left of G. Only two people are sitting between G and B.
A sits second to the left of B . The immediate neighbour of A face opposite directions (
i.e. If one neighbour faces south then the other faces north and vice versa).
Only one person sit between A and C. F is third to the left of C. D is not immediate
neighbour of B. Both the immediate neighbour of C faces the same direction ( i.e. if one
neighbour faces south then the other neighbour faces south and vice versa). A faces
same direction as C. E faces north. B sits on the immediate left of E.

Q.109. What is position of E with respect to F?


a. Second to the right
b. Immediate left
c. Immediate Right
d. Second to the left
e. Third to the left

Q.110. Who amongst the following sit exactly between F and C?


a. G
b. A
c. G and A
d. D
e. E

Q.111. Who amongst the following sitting at the extreme end of the line?
a. E , D
b. E , C
c. E , G
d. D , F
e. D , B

Q.112. How many persons are facing south?


a. One
b. Two
c. Three

www.bankexamstoday.com Page 55
d. Four
e. Five

Q.113. Who sits third to the left of G?


a. D
b. E
c. F
d. B
e. Cannot be determined

Q.114. Four of the following five are alike in a certain way based on the given
arrangement and so form a group. Which is the one that does not belong to
that group?
a. E
b. B
c. D
d. A
e. G

Direction: Study the following information carefully and answer the questions
given below:

Each of the seven exhibition viz. A,B,C,D,E,F and G are scheduled to be exhibited on a
different days of a week starting from Monday and ending on a sunday of the same
week. Exhibition F is
scheduled on Thursday. Four states are participating in the exhibition, not more than
two exhibitions are exhibited by any state. Four states are Goa, Tamil Nadu, Assam and
Maharashtra. Assam participates on the weekend. Two exhibitions are scheduled to be
held between exhibition F and the exhibition in which Tamil Nadu participates. Only one
exhibition is scheduled between exhibition E and exhibition D. Maharashtra participates
in only one exhibition that is scheduled to be held on Saturday. Exhibition E is not
scheduled on the day immediately before or immediately after the day when exhibition F
is scheduled. Assam participates exactly in the middle of the week. Exhibition C is
scheduled on the day immediately before the day when exhibition G is scheduled. D
participates between the day when Assam and Maharashtra participate. Tamil Nadu does
not participate on even number of days but not on the 5th day of the week. Exhibition A
is not scheduled after exhibition G.

Q.115. How many exhibitions are scheduled to be exhibited between exhibition


C and exhibition D?
1. Three
2. Four

www.bankexamstoday.com Page 56
3. Two
4. None
5. Other than the given options

Q.116. On which of the following days is exhibition G scheduled?


1. Wednesday
2. Tuesday
3. Monday
4. Saturday
5. Cannot be determined

Q.117. Which of the following exhibition is scheduled on Saturday?


1. E
2. B
3. A
4. C
5. Other than the given options

Q.118. In which of the following exhibitions does Goa participate?


1. C,B
2. A,C
3. C,A and G
4. C,D
5. Other than the given options

Q.119. Which of the following exhibitions is scheduled on Friday?


1. A
2. C
3. E
4. G
5. D

Q.120. Which of the following do/does not match correctly?


1. A-Monday-Goa
2. D-Saturday-Assam
3. F-Thursday-Assam
4. Both 1) and 2)
5. Other than the given options

Direction: Study the following information carefully and answer the question
given below:

www.bankexamstoday.com Page 57
There are five Players viz. P, R, S, T and V are from five different countries viz. Russia,
China, India, Britain and USA, but not necessarily in the same order. They are the
players of five different games viz. Football, Hockey,
Cricket, Tennis and Badminton and use three cars viz. Mercedes, Maruti and Honda to
travel but not necessarily in the same order. Maximum two players can share one car.
 The player who goes along with the player who is from USA, plays Tennis. R plays
football.
 V does not go with Russian footballer.
 S is from Britain but does not travel in Mercedes.
 Hockey players belong to Russia.
 One who is a player of football travels in that car which is shared by two persons.
 The one who plays Cricket likes to travel with whom who plays Badminton.
 Mercedes car is used by only one player. V is the player of Cricket.
 Chinese does not play Hockey or Tennis and does not like Honda. The one who play
Football like Honda.
 P, who is from India, cannot share his ride either with the player of 'China' or
'Britain'.
 The player who is from Britain does not play Tennis.
Q.121. Who among the following uses Mercedes car?
1. P
2. T
3. R
4. S
5. V

Q.122. Who among the following is from USA?


1. R
2. P
3. S
4. V
5. T

Q.123. 'V' belongs to which of the following countries?


1. Russia
2. USA
3. China
4. India
5. Britain

Q.124. Which of the following players use (s) 'Honda' car?


1. R
2. S and T

www.bankexamstoday.com Page 58
3. V and P
4. P and R
5. R and S

Q.125. 'S' plays which of the following games?


1. Tennis
2. Badminton
3. Hockey
4. Football
5. Cricket

Q.126. 'P' travels with which country's player?


1. India
2. Britain
3. USA
4. Russia
5. China

Q.127. Which of the following is matched correctly?


1. S-badminton- Maruti-USA
2. R-Badminton- Honda-USA
3. V-Cricket- Maruti-China
4. P-Cricket-Maruti-USA
5. All are wrong

Directions: Read the information given below and answer the question (1-3).
O is the father of two children, P and R who are different genders. Q is P's spouse. S is
the same gender as R. P and Q have two children. T is the same gender as P and U is
the same gender as Q. S‘s
mother V is married to Z and V is the is the sister of R‘s mother M. S and S‘s spouse I
have two children, X and Y who are the same gender as W.
No person has married more than once and no child has been born out of wedlock. Only
restrictions on marriage are that marriage to a sibling, to a direct descendant, or to
more than one person at the same time is forbidden.

Q.128. R can marry


(a) T only
(b) U only
(c) X only
(d) X and Y only
(e) Either T or X or Y

www.bankexamstoday.com Page 59
Q.129. T is
(a) U‘s brother
(b) U‘s sister
(c) P‘s daughter
(d) R‘s niece or nephew
(e) The same gender as V

Q.130. If the generation of T and Y‘s parents and their siblings contains more females
than males, then which of the following must be true?
(a) There are more females than males in T and Y‘s generation
(b) X is male
(c) O is the same gender as R
(d) Y and U are the same gender.
(e) R is V‘s nephew.

Ten film stars from Tollywood are sitting in two parallel rows containing five persons
each in dining hall. In row 1, Venkatesh, Nagarjuna, Bala Krishna, Jagapathi Babu and
Chiranjivi are sitting and all of them are facing south.
In row 2, N.T.R, Pavan Kalyan, Mahesh Babu, Allu Arjun and Ram Charan are sitting and
all of them are facing north. In the given seating arrangement, each member seated in a
row faces another member of the other row. Moreover, each of them going to different
countries for shooting — Switzerland, USA, Netherlands, Canada, Germany, AUSTRALIA,
Dubai, Egypt, Italy and Turkey but not necessarily in the same order.

There are only two persons sitting between the person going to Switzerland, who sits at
an extreme end, and Chiranjivi. N.T.R, who sits in the middle of the row, is not an
immediate neighbour of Pavan Kalyan, who is not going to Turkey.Ram Charan is sitting
at an extreme end. Chiranjivi, is going to German, sits on the immediate right of the
person going to USA and faces the immediate neighhour of Pavan Kalyan. Nagarjuna is
not sitting at the extreme left end. N.T.R is not going to Italy.

There is only one person between Bala Krishna and Jagapathi Babu, who is going to
Switzerland. Allu Arjun, who is going to AUSTRALIA, is an immediate neighbour of the
person going to Turkey and does not face the person going to Netherlands.Mahesh
Babu, who is going to Canada, is an immediate neighbour of the person going to Italy,
who in turn faces the immediate neighbour of the person going to Germany. There are
two persons between the person going to Dubai and Netherlands. Nagarjuna is not going
to Dubai. Ram Charan is not going to Egypt.

Q.131. Venkatesh going to which of the following countries?


1) Egypt
2) Dubai

www.bankexamstoday.com Page 60
3) Netherlands
4) None of these
5) can't be determined

Q.132. Who is going to Egypt?


1) Mahesh Babu
2) Nagarjuna
3) Ram Charan
4) N.T.R
5) can't be determined

Q.133. „Switzerland‟ is related to 'Dubai' in a certain way, based on their


seating positions. Then Turkey is related to whom, following the same seating
positions?
1) Egypt
2) Italy
3) USA
4) AUSTRALIA
5) Canada

Q.134. Four of the following five are alike in a certain way based on their
seating positions and so form a group. Which of the following is different from
the group?
1) Germany
2) Dubai
3) Turkey
4) Canada
5) AUSTRALIA

Q.135. Which of the following statements is/are definitely false?


1) Nagarjuna is going to Netherlands.
2) There are two persons sitting between the person going to AUSTRALIA and the
person going to Italy.
3) The person going to USA faces the person going to Egypt.
4) The person who is going to Italy sits opposite the person going to Switzerland.
5) All are true

Read the followings and answers the questions given below:


Students - A, B, C, D, E, F, G and H are sitting in three lines L1, L2 and L3. And they like
three different Subjects, either Math or Chemistry or Physics. In each line at least two
and at most three students are sitting there. And at least two students are in each
category but not more than three.

www.bankexamstoday.com Page 61
B and C are in different lines but not in line L2, and both like same subject. H and G are
in same line and like same subject, but not all Physics. D and E are in different lines but
not in line L1 and like different subjects but not all Physics. A is in Line L2 and D is not in
same line as in A. Both students of line L2 like Chemistry. F likes Physics and not in line
L1. Those who like Physics, two of them are line in L3. F and C are in same line and
there are three students who like math.

Q.141. Who are like Math among these eight students?


1) B, D, F
2) G, D, B
3) B, G, H
4) H, G, C
5) D, H, G

Q.142. In which line G and D are sitting?


1) L2 and L3
2) L1 and L2
3)L1 and L3
4) both in L3
5) Can't be determined

Q.143. Which two are in same subject and in same line?


1) F and B
2) H and D
3) G and B
4) C and F
5) E and D

Q.144. In which subject and line E is there?


1) Chemistry—L2
2) Chemistry—L3
3) Maths—L2
4) Maths—L3
5) Cant' be determined

Q.145. In which subject 3 students are there?


1) Chemistry and Physics
2) Maths and Physics
3) Chemistry and Maths
4) Can't be determined
5) None of these

www.bankexamstoday.com Page 62
Ten persons are sitting in two parallel rows containing five persons each. In row 1, A, B ,
C, D and E are sitting and all of them are facing south. In row 2, P, Q, R, S and T are
sitting and all of them are facing north. In the given seating arrangement, each member
seated in a row faces another member of the other row. Moreover, each of them belongs
to different cities — Chennai, Gurgaon, Hyderabad, New Delhi, Noida, Cuttack,
Bengalore, Cochin, Allahabad and Mumbai but not necessarily in the same order.

There are only two persons sitting between the person from Chennai, who sits at an
extreme end, and E. P, who sits in the middle of the row, is not an immediate neighbour
of Q, who is not from Mumbai. T is sitting at an extreme end. E, from Noida , sits on the
immediate right of the person from Gurgaon and faces the immediate neighhour of Q. B
is not sitting at the extreme left end. P is not from Allahabad. There is only one person
between C and D, who is from Chennai. S, from Cuttack, is an immediate neighbour of
the person from Mumbai and does not face the person from Hyderabad. R, who is from
New Delhi, is an immediate neighbour of the person from Allahabad, who in turn faces
the immediate neighbour of the person from Noida. There are two persons between the
person from Bengalore and the person from Hyderabad. B is not from Bengalore. T is
not from Cochin.

Q.146. A belongs to which of the following cities?


1) Cochin
2) Bengalore
3) Hyderabad
4) None of these
5) Can't be determined

Q.147. Who is from Cochin?


1) R
2) B
3) T
4) P
5) Can't be determined

Q.148. „Chennai‟ is related to 'Bengalore' in a certain way, based on their


seating positions. Then Mumbai is related to whom, following the same seating
positions?
1) Cochin
2) Allahabad
3) Gurgaon
4) Cuttack
5) New Delhi

www.bankexamstoday.com Page 63
Q.149. Four of the following five are alike in a certain way based on their
seating positions and so form a group. Which of the following is different from
the group?
1) Noida
2) Bengalore
3) Mumbai
4) New Delhi
5) Cuttack

Q.150. Which of the following statements is/are definitely false?


1) B is from Hyderabad.
2) There are two persons sitting between the person from cuttack and the person from
Allahabad.
3) The person from Gurgaon faces the person from Cochin
4) The person who is from Allahabad sits opposite the person from Chennai.
5) All are true

Directions: A, B, C, D, E, F and G are employees in different companies. Each of them


works on different floors number from I to VII, but not necessarily in the same order.
Each of them works in different company, viz Google, Facebook, IBM, HP, EY, Analog
Device and HCL but not necessarily in the same order.
B works on floor IV but he does not work in either EY or HP. C works in Google Company
but he does not work on floor II or VI. E works on floor V and he works in Analog Device
Company. The one who works in the Facebook company works in floor VII. D works on
floor I. G works HCL in a company. A does not work on VII. The one who works in HP
company works on floor II.

Q.151. G works on which of the following Floors?


1) II
2) III
3) VI
4) VII
5) None of these

Q.152. A works in which of the following company?


1) HP
2) Google
3) EY
4) IBM
5) None of these

www.bankexamstoday.com Page 64
Q.153. Which of the following combinations is/are true?
1) F - IBM – VII
2) D – EY - I
3) A – Facebook – I
4) Both 1) and 3)
5) None of these

Q.154. Who among the following works in the Facebook Company?


1) A
2) F
3) D
4) Can‘t be determined
5) None of these

Q.155. Who among the following works on floor II?


1) F
2) C
3) A
4) G
5) None of these

Directions: In a conference 8 people A, B, C, D, E, F,G and H from different cities


Chennai, Hyderabad, Bangalore, New Delhi, Mumbai, Pune , Vizag and Kolkata not
necessarily in same order sitting around a rectangular table.

3 persons are sitting on each longer side and 1 on each smaller sides. D is sitting second
to the right of the person who is from Kolkata. E is sitting third to the left of the person
who is from Pune. F and G are sitting opposite each other. C is sitting diagonally
opposite the person from Vizag. B is sitting opposite the person who is from Chennai.
The person from New Delhi is sitting second to the right of the person from Mumbai and
second to the left of A, who is not sitting near the person who is from Vizag. G is sitting
on the smaller side and to the right of the person who is from Vizag. Persons from
Mumbai and New Delhi are not on the same side of the table. The person from New
Delhi is sitting third to the right of the person from Kolkata, who is not sitting diagonally
opposite the person from Hyderabad. The person from New Delhi is third to the left of C.
F sits second to the left of H .The person from Hyderabad sits opposite to the person
from Mumbai.

Q.156. Who is sitting third to the right of the person from Pune?
1) E
www.bankexamstoday.com Page 65
2) Person from Mumbai
3) A
4) Person from Vizag
5) None of these

Q.157. Who is definitely sitting diagonally opposite to D?


1) A
2) C
3) Person from Hyderabad
4) Person from Bangalore
2) Can't be determined

Q.158. According to the sitting arrangement what will come in place of


question mark?
D : New Delhi :: C : ?
1) Pune
2) Mumbai
3) Bangalore
4) Kolkata
5) None of these

Q.159. From which city does A belong?


1) Hyderabad
2) Chennai
3) Mumbai
4) Kolkata
5) Can't be determined

Q.160. Which of the following combinations is definitely correct?


1) H –Kolkata
2) D – Mumbai
3) A– Bangalore
4) G – New Delhi
5) None of these

Directions: P, Q, R, S, T, U, V and W are sitting in a straight line equidistant from each


other (but not necessarily in the same order). Some of them are facing south while
some are facing north. P faces north. Only two people sit to the right of P. Q sits third to
the left of P.
Only one person sits between Q and U. U sits to the immediate right of T. Only one
person sits between T and W. Both the immediate neighbors of Q face the same
direction. R sits third to the left of U. Q faces the opposite direction as P. V does not sit

www.bankexamstoday.com Page 66
at any of the extremes ends of the line. S faces the same direction as T. Both V and R
face the opposite direction of W.

Q.161. How many persons in the given arrangement are facing North?
(a) More than four
(b) Four
(c) One
(d) Three
(e) Two

Q.162. Four of the following five are alike in a certain way, and so form a
group. Which of the following does not belong to the group?
(a) T, U
(b) W, V
(c) Q, P
(d) Q, V
(e) S, R

Q.163. What is the position of U with respect to W?


(a) Second to the left
(b) Third to the right
(c) Third to the left
(d) Fifth to the right
(e) Second to the right

Q.164. Who among the following sits exactly between W and T?


(a) Q
(b) V
(c) U
(d) T
(e) R

Q.165. Who is sitting SECOND to the right of Q?


(a) W
(b) S
(c) U
(d) T
(e) None of these

Directions: A, B, C, D, E, F, G and H eight family members are sitting around a circular


table, not necessarily in the same order. Four of them are facing inside and others are
facing outside. They like different Colors viz – Red, Blue, Green, Yellow, Violet, White,

www.bankexamstoday.com Page 67
Brown and Gray, but not necessarily in the same order. F faces the Center and sits third
to the right of C. D likes Green color and faces the person who likes White color. G sits
third to the right of B who likes Red color.
The persons who like Yellow and Violet are facing to the same direction (inside or
outside). E is sitting between the person who like Green and the one who likes Gray
respectively. A likes Violet and C likes Blue. The person who likes Brown is facing
outward and is an immediate neighbor of the person who likes Gray. H is
immediate neighbor of the persons who likes Violet and Brown. E is immediate left of D.

Q.166. Who likes the color White?


(a)A
(b) D
(c) H
(d) G
(e) E

Q.167. Who among the following are the immediate neighbours of F?


(a) B and C
(b) E and D
(c) H and B
(d) G and E
(e) D and F

Q.168. Which of the following combination is true?


(a) A – Red
(b) H – White
(c) D – Yellow
(d) F – Yellow
(e) G – Blue

Q.169. What is the Position of B with respect to E?


(a) Third to the right
(b) Fourth to the left
(c) Fifth to the right
(d) Second to the right
(e) Third to the left

Q.170. If D and H interchange their positions and similarly F and C interchange


their positions then what is the position of A with respect to C ?
(a) Third to the left
(b) Third to the right
(c) Second to the left

www.bankexamstoday.com Page 68
(d) Immediate left
(e) none of these

Directions: Seven people, A, B, C, D, E, F and G have an exam, but not necessarily in


the same order, in seven different months (of the same year), namely January,
February, March, June, August, October and December. Each of them also likes a
different color, namely Red, Yellow, Blue, Orange, Green, Brown and White, but not
necessarily in the same order.

C has an exam in a month which has less than 31 days. Only two people have an exam
between C and D. The one who likes Red has an exam immediately before D. Only one
person has an exam before the one who likes Blue. B has an exam immediately after the
one who likes Blue. Only three people have an exam between B and the one who likes
Green. E likes neither Green nor Blue. A has an exam immediately before E. G likes
White. The one who likes Yellow has an exam in the month which has less than 31 days.
The one who has an exam in March does not like Orange.

Q.171. In which of the following months does D have a exam?


1) January
2) Cannot be determined
3) October
4) December
5) June

Q.172. Who among the following have exams in January and June respectively?
1) G, D
2) F, D
3) B, E
4) G, C
5) F, C

Q.173. How many people has/have a exam between the month in which G and
C have exams?
1) None
2) Two
3) Three
4) One
5) More than three

Q.174. As per the given arrangement, C is related to Red and B is related to


Orange following a certain pattern. Which of the following is G related to
following the same pattern?

www.bankexamstoday.com Page 69
1) Green
2) Brown
3) White
4) Blue
5) Yellow

Q.175. Which of the following colors does F like?


1) Orange
2) Blue
3) Green
4) Red
5) Yellow

Directions: A group of eight friends – A, B, C, D, E, F, G and H – are sitting in a straight


line facing north. Each of them has different cities – Kolkata , Mumbai , New Delhi ,
Hyderabad , Chennai, Bangalore, Bhopal and Kochin . Each of them likes different colors
– Pink, Yellow, Red, Black, Blue, Orange, White and Green, but not necessarily in the
same order.
A, who is from Chennai , sits third to the left of F. Neither A nor F sits at the extreme
ends of the line. E, who likes Pink colors, is from New Delhi . E is not an immediate
neighbor of either A or F. H is two places away from C and likes Orange colors. C, who is
from Bhopal , likes Red colors. G, who is from Kolkata, sits at an extreme end of the line
and likes White colors. B, who is from Hyderabad , likes Green colors and sits on the
immediate left of A. A does not like either Blue or Yellow colors. One who is from Kochin
sits on the immediate left of the Kolkata person . F, who is from Mumbai, does not like
Yellow colors.

Q.176. D is from which of the following city ?


1) Chennai
2) Bhopal
3) Bangalore
4) Kochin
5) None of these

Q.177. Who sits second to the left of C?


1) B
2) A
3) F
4) H
5) None of these

Q.178. If „B‟ is related to „Bangalore, „A‟ is related to „Bhopal, „C‟ is related to

www.bankexamstoday.com Page 70
which of the following?
1) Kolkata
2) Chennai
3) Kochin
4) Hyderabad
5) None of these

Q.179. Who sits at the extreme ends of the line?


1) BG
2) AD
3) EH
4) DF
5) None of these

Q.180. Four of the following five are alike in a certain way based on their
position. Which one does not belong to the group?
1) EB
2) DC
3) GH
4) AD
5) CF

Directions: There are nine persons Anil , Balaji, Chaitu, Dinesh, Eswar, Feriq, Ganesh,
Hari and Ishant stay on a nine floors building, but not necessarily in the same order.
Only one person stays on each floor. All of them have either Bike or Car of different
colors i.e. yellow, golden, red, orange, purple, black, white, silver and grey, but not
necessarily in the same order. Only five of them have Car . The ground floor is
numbered 1 and the topmost floor is numbered 9.

There are two floors between the floors on which the persons having a black Bike and
silver Car stay. Chaitu owns a red Bike. Feriq does not own a grey Bike. Anil stays on
even numbered floor below the even numbered floor on which Hari stays. The one who
own a purple Car stays on the fourth floor. Dinesh stays on the second floor and owns
the orange Car. The one who owns the golden Car stays on the topmost floor. Anil does
not own a white Car. Feriq owns a black Bike and stays on an odd-numbered floor.
There is only one floor between the floors on which Ferriq and Ganesh stay. Eswar stays
immediate above the floor on which Ishant stay. Ganesh does not stay on the ground
floor. Balaji stays immediate above the floor on which Hari stay. There are three floors
between the floors on which Chaitu and Feriq stay. The one who owns the silver car
stays immediate above Ganesh .Eswar stays on the fourth floor. The one who owns the
grey Bike stays on the third floor.

www.bankexamstoday.com Page 71
Q.181. What is the color of the Car of the person who stays on the 4th floor?
(a) Red
(b) White
(c) Orange
(d) Purple
(e) Yellow

Q.182. How many persons are between the one who has golden car and the one
who stays on 4th floor?
(a) Five
(b) Three
(c) Seven
(d) Four
(e) None of these

Q.183. Who lives on the 7th floor?


(a) The one who has white car
(b) Balaji
(c) Ganesh
(d) Both (a) and (c)
(e)The one who has silver car

Q.184. Four of the following five are alike in a certain way and hence they form
a group. Which one of the following does not belong to that group?
(a) Red
(b) White
(c) Golden
(d) Silver
(e) Grey

Q.185. Which of the following colored car does Feriq has?


(a) Yellow
(b) Red
(c) Purple
(d) Grey
(e) Black

Directions: Study the following information carefully and answer the questions
given below:
Seven members A, B, C, D, E, F and G represent different countries in Rio Olympics, viz,
USA, China, Brazil, india, Russia, Jamaica and Japan; each One competes for a different
sport and gets

www.bankexamstoday.com Page 72
gold medals in the event, viz. swimming, Archery, Rifle Shooting, Tennis, Wrestling,
Athletics and gymnastics‘. The order of persons, countries and games is not necessarily
the same. C represents China for Archery. D represents USA but not for swimming or
Rifle Shooting. The one who represents Japan competes for Wrestling. E competes for
swimming but not for Brazil .A represents Jamaica for Athletics. The one who represents
Russia competes for Tennis. F does not represent Brazil or Japan.G competes for Rifle
Shooting.

Q.186. Which of the following combinations is correct?


a) F - Tennis -India
b) G-Tennis - Russia
c) G - Tennis - India
d) F - Tennis - Russia
e) None of these

Q.187. Who represents Japan?


a) E
b) G
c) F
d) B
e) None of these

Q.188. E represents which country?


a) India
b) Russia
c) Japan
d) Brazil
e) None of these

Q.189. The one who competes for Rifle Shooting, represents which country?
a) India
b) Brazil
c) Japan
d) USA
e) None of these

Q.190. For which game does D compete?


a) Wrestling
b) Gymnastic
c) Tennis
d) Cannot be determined
e) None of these

www.bankexamstoday.com Page 73
Directions: Seven people, namely M, N, O, P, Q, R and S will be attending seven
different meetings in seven different cities namely, Mumbai, Delhi, Pune, Surat,
Chennai, Baroda and Kolkata, not necessarily in the same order, from Monday to Sunday
(of the same week).And also likes different colors red,
white, pink, yellow, green, black, and blue color and not necessarily in the same order.

Q will attend a meeting on Thursday and like yellow color. Only two people will attend a
meeting between Q and the one who will attend a meeting in Chennai. P will attend a
meeting immediately before N. P will attend a meeting on one of the days before Q. P
will neither attend a meeting in Chennai or black color. S like green color .Only two
people will attend a meeting between P and the one who will attend a meeting in Pune.
Only one person will attend a meeting between Q and the one who will attend the
meeting in Kolkata. Neither P nor N will attend a meeting in Kolkata or pink. M will
attend the meeting immediately before the one who will attend the meeting in Baroda.
One who attend the meeting Baroda not like red color. P does not like red color. The
who attend the meeting Kolkata likes pink color. Only three people will attend a meeting
between S and O. S will attend a meeting before O and O like‘s blue color. P will neither
attend a meeting in Mumbai nor in Surat. Q will not attend a meeting in Mumbai.

Q.191. In which of the following Cities will N attend a meeting?


a. Baroda
b. Mumbai
c. Chennai
d. Delhi
e. None

Q.192. On which of the following days will S attend a meeting?


a. Wednesday
b. Friday
c. Tuesday
d. Monday
e. Saturday

Q.193. Which of the following pairs represent those who will participate
immediately before and immediately after Q?
a. S, R
b. None
b. S, M
d. N, O
e. N, R

www.bankexamstoday.com Page 74
Q.194. As per the given arrangement P is related to Friday and Q is related to
Sunday in a certain way. To which of the following is S related to in the same
Way?
a. Tuesday
b. Wednesday
c. Saturday
d. Monday
e. None

Q.195. In which of the following colors will R like?


a. Black
b. Pink
c. Yellow
d. Green
e. None

Q.196. Four of the following five are alike in a certain way as per the given
arrangement and hence form a group. Which of the following does not belong
to that group?
a. O-Kolkata
b. Q-Chennai
c. P-Mumbai
d. N-Surat
e. S-Delhi

Q.197. Which of the following combination is true?


a. N-red-Mumbai
b. R-black-Baroda
c. Q-surat-yellow
d. P-Delhi-white
e. All true

Directions: Eight people Manish, Naresh, Ravi, Pawan, kalyan, Raghu, Satish and Tarun
live on separate floors of an 8floor Building, not necessarily in the same order Ground
floor is numbered 1, first floor is numbered 2 and so on until the topmost floor is
numbered 8.
Tarun lives on floor numbered three floor. Only one person lives between Tarun and
Manish. Raghu lives immediately above Ravi. Raghu lives on an even numbered floor.
More than one person lives between Satish and Naresh. Satish lives immediately below
Ravi.
Pawan lives on an even numbered floor but not the top most floors. Manish does not live

www.bankexamstoday.com Page 75
immediately above or below Naresh. Only one person lives between Satish and Raghu.
Raghu lives one of the above floor on Satish lives.

Q.198. How many people live between the floors on which Raghu and kalyan
live?
a. Two
b. More than three
c. One
d. No one
e. Three

Q.199. Who among the following lives on floor no. 4?


a. Ravi
b. Manish
c. Raghu
d. Satish
e. Naresh

Q.200. Which of the following is true with respect to the given information?
a. Naresh lives immediately below Tarun
b. Raghu lives on floor numbered two
c. Kalyan lives on an odd numbered floor
d. Only one person lives between Pawan and Raghu
e. Only one person lives between Ravi and Tarun

Q.201. Who lives on the floor immediately below Satish?


a. Manish
b. Other than those given as options
c. Kalyan
d. Tarun
e. Pawan

Q.202. Who amongst the following lives between Naresh and Pawan?
a. Raghu
b. Ravi
c. All of these
d. Satish
e. Tarun

Directions: Ten people are sitting in two parallel rows, containing five people each in
such a way that there is an equal distance between adjacent persons. In row 1 J, K, L, M
and N are seated and all of them are

www.bankexamstoday.com Page 76
facing north and in row 2 A, B, C, D and E are seated and all facing south (but not
necessarily in the same order). Each person also travels to different places, namely
Berlin, Brisbane, Dubai, Frankfurt, Kenya, Quebec, Ottawa, Perth, Toronto and Zurich
(but not necessarily in the same order).

N sits at one of the extreme ends of the line and faces one of the immediate neighbors
of E. N travels neither to Zurich nor to Dubai. Only one person sits between E and the
one travelling to Toronto. Only one person sits between N and J. J faces one of the
immediate neighbors of the one travelling to Quebec. M neither sits at the extreme end
of the line nor faces the one travelling to Toronto. M faces one of the immediate
neighbors of C. The one travelling to Berlin sits second to the right of C. The one
travelling to Zurich faces C. Neither K nor B sits at any of the extreme ends of the line. A
faces the one travelling to Dubai. The one travelling to Frankfurt and the one travelling
to Brisbane face each other. The one travelling to Perth sits third to the left of the one
travelling to Brisbane. E does not face the one travelling to Kenya.

Q.203. Who among the following travels to Perth?


a. D
b. K
c. M
d. C
e. E

Q.204. To which of the following places does D travel?


a. Brisbane
b. Dubai
c. Kenya
d. Perth
e. Frankfurt

Q.205. Which of the following pairs represent the people sitting exactly in the
middle of both the lines?
a. A, L
b. B, K
c. J, C
d. B, M
e. E, M

Q.206. Which of the following is definitely true as per the given arrangement?
a. The one who travels to Ottawa faces E
b. C is an immediate neighbor of D
c. M is an immediate neighbor of the one travelling to Perth

www.bankexamstoday.com Page 77
d. E travels to Frankfurt
e. None of the given statements is true

Q.207. Who amongst the following is an immediate neighbor of the one who
travels to Brisbane?
a. D
b. The one who travels to Ottawa
c. L
d. The one who travels to Zurich
e. C

Directions: Seven people, namely S, T, U, V, W, X and Y will attend a function but not
necessarily in the same order, in seven different months (of the same year) namely
January, March, April, July, October, November and December. Each of them also likes a
different fabric namely Cotton, Silk, Jute, Khaddar, Linen, Muslin and Velvet but not
necessarily in the same order.

The one who likes cotton will attend a function in a month which has only 30 days. Only
one person will attend a function between the one who likes cotton and Y. The one who
likes silk will attend a function in one of the months after Y. Only two people will attend
a function between Y and the one who likes silk. Only three people will attend a function
between the one who likes cotton and the one who likes khaddar. V will attend a
function immediately before the one who likes khaddar. Only two people will attend a
function between V and U. S will attend a function immediately before W. Only two
people will attend a function between W and the one who likes jute. Y likes neither linen
nor muslin. The one who likes linen will attend a function immediately before X

Q.208. How many people will attend a function between the months in which T
and W will attend a function?
1. More than Three
2. Two
3. None
4. One
5. Three
Q.209. Which of the following fabrics does X like?
1. Khaddar
2. Cotton
3. Jute
4. Muslin
5. Velvet

www.bankexamstoday.com Page 78
Q.210. Which of the following represents the month in which T will attend a
function?
1. April
2. March
3. Cannot be determined
4. October
5. July

Q.211. As per the given arrangement Velvet is related to July and Silk is related
to December following a certain pattern, which of the following is Cotton
related to following the same pattern?
1. November
2. October
3. March
4. January
5. April

Q.212. Which of the following represents the people who will attend a function
in January and December respectively?
1. Y, W
2. Y, V
3. T, V
4. Y, X
5. T, X

Directions: Eight friends Amar, Kumar, Shekhar, Kishore, Kamal, Vimal, Jamal and
Vishal are sitting in a straight line equidistant from each other (but not necessarily in the
same order). Some of them are facing south while some are facing north.
(Note : Facing the same direction means, if one is facing north then the other
also faces north and vice-versa. Facing the opposite directions means, if one is
facing north then the other faces south and vice-versa)
Amar faces north. Only two persons sit to the right of Amar. Kumar sits third to the left
of Amar. Only one person sits between Kumar and Vimal . Vimal sits to the immediate
right of Kamal. Only one person sits between Kamal and Vishal. Both the immediate
neighbors of Kumar face the same direction as Shekar sits third to the left of Vimal.
Kumar faces the opposite direction as Amar. Jamal does not sit at any of the extremes
ends of the line. Kishore faces the same direction as Kamal. Both Jamal and Shekhar
face the opposite direction of Vishal.

Q.213. How many persons in the given arrangement are facing South?
(a) More than four
(b) Four

www.bankexamstoday.com Page 79
(c) One
(d) Three
(e) Two

Q.215. What is the position of vimal with respect to kishore?


(a) Second to the left
(b) Third to the right
(c) fourth to the left
(d) fourth to the right
(e) Second to the right

Q.216. Who amongst the following sits exactly between jamal and kamal?
(a) vimal and amar
(b) vimal and shekhar
(c)vishal and kishore
(d) shekhar and kumar
(e) none

Q.217. Who is sitting 2nd to the right of vishal?


(a) shekar
(b) kamal
(c) kumar
(d) kishore
(e) None of these.

Directions: A, B, C, D, E, F, G and H are seated in a straight line but not necessarily in


the same order Some of them are facing South while some are facing north .H sits
fourth to left of A .A faces South. E sits second to right of A. D sits second to right of G.
G is neither an immediate neighbours of A nor H. Both the immediate neighbours of F
face North. F is not an immediate neighbour of H .Immediate neighbours of E face
opposite directions (i.e. if one neighbour faces north then the other faces south and vice
versa). Both the immediate neighbours of B face the direction opposite to that of D (i.e.
if D faces north then both the immediate neighbours of B face south and vice versa) E
and C face opposite directions(i.e. if E faces north then C faces south and vice versa).

Q.218. Which of the following pairs represents immediate neighbor‟s of the


persons seated at the two extreme ends of the line ?
1).B, A
2).D, H
3).E, B
4).E, F
5).None

www.bankexamstoday.com Page 80
Q.219. If each of the persons is made to sit in alphabetical order from right to
left the positions of how many will remain unchanged as compared to the
original seating arrangement?
1).Three
2).Two
3).Four
4).One
5).None

Q.220. What is the position of G with respect to A?


1).Second to the left
2).Third to the right
3).Third to the left
4).Second to the right
5).Fifth to the right

Q.221. How many persons are seated between A and B?


1).Two
2).Four
3).Three
4).One
5).More than four

Q.222 How many persons in the given arrangement are facing North?
1).More than Four
2).One
3).Three
4).Four
5).Two

Direction :-
The six faces of a cube are colored, each with a different colors.
I. The white face is between yellow and green
II. The red face is adjacent to brown.
III. The green face is opposite the yellow side.
IV. The blue face is adjacent to red.
V. The yellow face is the top face of the cube.

www.bankexamstoday.com Page 81
Q.223. The faces adjacent to white bear the colors
a. Yellow, green, brown and red
b. Yellow, brown, blue, and green
c. Yellow, green, blue and red
d. None of the above

Q.224. The face opposite the red face is


a. Green
b. White
c. Blue
d. Brown

Q.225. The colors of the bottom face of the cube is


a. Red
b. Brown
c. Green
d. Blue

Directions:
(i) Khanna transport company operates 7 buses M, N, O, P, Q, R and s once for daily 4
hours sight- seeing tours.
(ii) From Monday to Friday first bus leaves at 8 a.m., after that one bus leaves
alternatively after a gap of 45 minutes, followed 30 minutes, again 45 minutes and 35
minutes and so on.
(iii) On Saturday and Sunday, first bus leaves at 7:30 a.m. and others follows regularly
after a gap of I Hour.
(iv) Bus 'O' leaves immediately after 'M' and is immediately followed by 'S'.
(v) Bus 'O' is not followed by any other bus.
(vi) Bus 'R' leaves immediately before 'M' but not immediately after 'P'.

Q.226. At what time, 'bus 'M' leaves on Saturday?


(a) 10 a.m.

www.bankexamstoday.com Page 82
(b) 9:45 a.m.
(c) 10:30 a.m.
(d) Data inadequate
(e) None of these

Q.227. On Sunday , when bus 'p' completes its tour which of the following
buses begins its tour?
(a) Q
(b) S
(c) O
(d)Data inadequate
(e) None of these

Q.228.If the time gap after Bus 'M' leaves on Saturday -Sunday is increased by
30 minutes for the subsequent trips, at what time ,tour of bus 'O' will be
completed ?
(a) 3 p.m.
(b) 2 p.m.
(c) 6 p.m.
(d) 7 p.m.
(e) None of these

Q.229. If the time gap between two buses is uniformly kept as 45 minutes from
Monday to Friday, then the beginning of tour of bus 'O' will mark completion of
tour of which of the following buses?
(a) N
(b) R
(c) M
(d) Data inadequate
(e) None of these

Directions: Arvind, Nayan, Mayank, Rohit, Jai, Varun, Sumit and David are eight friends
sitting around a circular table. Two of them are not facing the center. All of them like
different types of Automobile brands viz.
Mahindra, Porsche, Hyundai, TVS, Fiat, Rolls Royce, Maruti and Opel, but not necessarily
in the same order. Rohit and Mayank are sitting third and second to the left of Arvind
respectively. Rohit and Sumit are neighbours of Nayan,who likes Fiat and facing
outside.
Sumit is sitting third to the right of Arvind who is facing outside and likes Mahindra.
David is third to the right of Jai and likes Maruti. Arvind and Davind are not the
neighbour of that person who likes Rolls Royce. Varun does not like Hyundai and TVS.
The person who likes Opel is sitting opposite of Jai, who is not an immediate neighbour

www.bankexamstoday.com Page 83
of TVS. An immediate neighbour of Summit likes Rolls Royce.

Q.230. The person who is sitting second to the right of Arvind likes which of the
following automobile brand?
1) Mahindara
2) Rolls Royce
3) Porsche
4) Opel
5) Other than given options

Q.231. Who among the following likes TVS?


1) Nayan
2) Mayank
3) Rohit
4) David
5) Other than given options

Q.232. What is the position of Varun with respect to Arvind?


1) Immediate left
2) Second to the right
3) Immediate right
4) Cannot be determined
5) Other than given options

Q.233. If Sumit is immediate right of Nayan then in the same way who among
the following sits third to the right of Nayan?
1) Mayank
2) Jai
3) David
4) Varun
5) Other than given options

Q.234. How many person sit between the person who likes Mahindra and the
one who likes Hyundai (count in clockwise direction starting from Hyundai)?
1) Two
2) One
3) None
4) Three
5) Four

Q.235. Which of the following statements is true?


1) Mayank is the neighbour of Rohit and Nayan

www.bankexamstoday.com Page 84
2) Arvind likes Hyundai and faces outside the center
3) Sumit likes Mahindra and does not face inside the center.
4) All are true
5) None is true

Directions: A newspaper sought the help of a psychologist to solve psychological


problems of the readers. Among the readers. Jai,Gaurav, Jumman, Nawal, Bibhuti are
the five persons who are victims of different types of problems and residing in different
towns. Each one is not afflicted with the problem for the same number of years.

One of them is afflicted with the problem for three years and another for seven years.
Jai resides in Faizabad. He has been afflicted with a psychological problem (which is
older than the problem of the person who is afflicted with agoraphobia) for the last five
year.

Nawal has been afflicted with aerophobia for several years but not less than three years.
But he does not reside in Mumbai. The person who resides in Raipur is afflicted with
acrophobia for ten years. But he is not Jumman. Gaurav has been facing the problem for
several years but not more years than the person who resides in Danapur. Jumman has
been afflicted with monophobia for the last eight years. Gaurav resides in Maisur. Jai is
afflicted with nyctophobia.

Q.236. Who resides in Raipur?


1) Bibhuti
2) Jumman
3) Gaurav
4) Cannot be determined
5) Other than given options

Q.237. One who is afflicted with acrophobia has been suffering from for how
many years?
1) Eight
2) Seven
3) Six
4) Ten
5) Other than given options

Q.238. One who is afflicted with nyctophobia resides in which town?


1) Faizabad
2) Raipur

www.bankexamstoday.com Page 85
3) Mumbai
4) Cannot be determined
5) Other than given options

Q.239. Which of the following statement(s) is/are true?


I. Gaurav is the person residing in Mumbai
II. Jumman has been suffering from monophobia for the last eight years.
1) Only I
2) Both I and II
3) Only II
4) Neither I nor II
5) Either I or II

Q.240. Who resides in Mumbai?


1) Jai
2) Nawal
3) Gaurav
4) Bibhuti
5) Jumman

Directions : A family consists of seven members P, Q, R, S, T, U and V. While one of


them a student, the other Six follow different profession of Architect, Lawyer, professor,
Manager , Doctor and Engineer.
There are two married couples in the family. P and U Architect and Doctor respectively.
S, the father of Q and the grandfather of V, is an Engineer.No lady in the family is an
Engineer or a Lawyer .V, who is student , is the grandson of T.

Q.241. Which of the following is one of the married couples?


(a) Q R
(b) Q U
(c) P R
(d) Data inadequate

Q.242. How is R related to V?


(a) Mother
(b) Aunt
(c) Sister
(d) Data inadequate

Q.243.what is T's Profession?


(a) Professor
(b) Manager

www.bankexamstoday.com Page 86
(c) Professor or Manager
(d) Doctor

Directions: Seven persons J, K, L, M, N, O and P participate in a series of swimming


races in which the following are always true about the results:

 K finishes ahead of L.
 N finishes directly behind M.
 Either J finishes first and O last, or O following first and J last.
 There are no ties in any race everyone finishes each race.

Q.244. If exactly two swimming finish between J and L, then which of the
following must be true?
(a) J finishes first
(b) o finishes first
(c) K finishes second
(d) M finishes fifth
(e) None of the above

Q.245. Which of the following cannot be true?


(a) K Finishes third
(b) K finishes sixth
(c) M finishes second
(d) N finishes fourth
(e) None of the above

Q.246. If N finishes directly ahead of K, then which of the following is a


complete and accurate listing of the positions in which M could have finished?
(a) Second
(b) Third
(c) Second, third
(d) Second, third , fourth
(e) None of the above

Q.247. If O and K finish so that one is directly behind the other, Then which of
the following must be true?
(a) K finishes sixth
(b) O finishes seventh
(c) J finishes first
(d) L finishes third, J finishes seventh
(e) None of the above

www.bankexamstoday.com Page 87
Q.248. If J finishes first, and if L finishes ahead of N, then in how many
different orders is it possible for the other swimmers to finish?
(a) 2
(b) 3
(c) 4
(d) 5
(e) none of the above

Answers

Directions:
 B/D - D/B (one person between B and D , here we do not know exact position of B
and D )
 C faces B
 A is to the right of C
 G is not to the immediate right or immediate left of D , also given G neither faces D
nor F
 H/F - F/H ( one person between them )
 E faces centre 0
Here you get two using B and C, they can stand on the outer or inner rectangle
Case (i) When B and D on outer rectangle , In any order D on left and B on right (or ) B
on left and D on right , E will always come in the inner rectangle i.e. facing outside ,this
does not satisify the given condition of E to face centre. So B and D cannot stand on
outer rectangle gates.

Case (ii) When B and D on inner rectangle , in any order will satisfy all the conditions.

www.bankexamstoday.com Page 88
Ans.1. ( d) F is facing A

Ans.2.(d) G doesn‘t form the part of the group as it lies in the outer rectangle other all
are in inner rectangle.

Ans.3. D
2 U

4 A

6 I

8 C

10 P

12 D

14 T

16 E

18 N

20 R

22

24 S

6- I
12 – D
14- T
20 – R
Rearranging this DIRT is formed ….So (d) I is the second word.

www.bankexamstoday.com Page 89
Ans.4.
2 U

4 A

6 I

8 C

10 P

12 D

14 T

16 E

18 N

20 R

22

24 S

Alphabet against 18 – P and against letter numbered 22 have 3 alphabets in the


English alphabetical order… it can be either backward or forward direction.
In forward direction : P - - - T ( T already exists )
In backward direction : L - - - P ( L is placed against 22)
Now letter numbered 18 i.e. N and letter numbered 22 i.e. L has only O in between .
So answer is one(c ) number .

Ans.5. D

Ans.6. C

Ans.7. E

Ans.8. C

Ans.9. B

Solution:

www.bankexamstoday.com Page 90
Ans.13. C

Ans.14. E

Ans.15. C

Ans.16. A

Ans.17. D

Solution

Floor Members

8 Z

7 Y

6 X

5 W

4 S

3 R

2 Q

1 P

www.bankexamstoday.com Page 91
Ans.18. D
Ans.19. B
Ans.20. B
Solution:

Ans.26. D. November

Ans.27. D. U, V

Ans.28. C. Three

Ans.29. B. October

Ans.30. D. Friday

Month Person Week days


February P Monday
May Q Sunday

www.bankexamstoday.com Page 92
April M Thursday
June O Tuesday
September L Saturday
March R Wednesday
November N Friday

Ans.31.D

Ans.32.B

Ans.33.E

Ans.34.C

Ans.35.A

Solutions:

Week Days Students Exam

Monday P SSC

Tuesday S Delhi Metro

Wednesday T Railways

Thursday R IBPS

Friday U RBI

Saturday Q RRB

Sunday V SBI

Ans.36.B

Ans.37.C

Ans.38.A

Ans.39.D

Ans.40.D

www.bankexamstoday.com Page 93
Solution: (36-40)
BankexamToday.com

Floors Floors Floors

9 I Sony

8 A Lenovo

7 G Nokia

6 B Sumsung

5 E Gionee

4 C Oppo

3 D Vivo

2 F Apple

1 H Asus
Solution: (41-45)

Ans.41.E

Ans.42.C

Ans.43.B

Ans.44.A

Ans.45.C

NOTE: T(YELLOW) WILL BE U(YELLOW)

Ans.46. B
www.bankexamstoday.com Page 94
Ans.47. C

Ans.48. D

Ans.49. E

Ans.50. D

9. Asha Shimla Uttar Pradesh

8. Cheena Mumbai Haryana

7. Sheela Kashmir Punjab

6. Meenu Nainital Rajasthan

5. Beena Lucknow Madhya Pradesh

4. Deepa Jaipur Maharashtra

3. Hema Haridwar Jharkhand

2. Veena Pune Uttrakhand

1. Geeta Ujjain Gujarat

Ans.51.B

Ans.52.C

Ans.53.C

Ans.54.D

Ans.55.C

Solution

www.bankexamstoday.com Page 95
Ans.61. 3

Ans.62. 4

Ans.63. 1

Ans.64. 5

Ans.65. 5

Ans.66. 2

City Mode of travelling

Pr Pt Mu Nd Ko Lu Cg Ch Bus Tr Ap Cy Bi Car Sh

A X X X X ✓ X X X X X X X ✓ X

B ✓ X X X X X X ✓ X X X X X X

C X ✓ X X X X X X X ✓ X X X X

D X X ✓ X X X X X X X ✓ X X X

E X X X ✓ X X X X X X X ✓ X X

F X X X X X ✓ X X ✓ X X X X X

www.bankexamstoday.com Page 96
H X X X X X X ✓ X X X X X X ✓

Note :

Pr -Person
Mu - Mumbai
Ko - Kolkata
Cg - Chandigarh
Tr- Train
Cy - Cycle
Sh - Ship
Pt - Patna
Nd - New Delhi
Lu - Ludhiana
Ch - Chennai
Ap - Airoplane
Bi - Bike

Person Place Transport Medium

A Lucknow Car

B Patna Bus

C Mumbai Aeroplane

D New Delhi Cycle

E Kolkata Bike

F Chandigarh Train

H Chennai Ship

Ans.67. 3

Ans.68. 4

Ans.69. 1

Ans.70. 5

Ans.71. 2

www.bankexamstoday.com Page 97
Ans.72. 4

Ans.73. 1

Descriptions:

People Floors Banks Post

T 8 Indian Bank Manager

N 7 UCO Bank Clerk

Z 6 PNB P.O

S 5 ICICI Bank Manager

P 4 Dena Bank P.O

W 3 SBI Clerk

R 2 Andhra Bank Manager

V 1 Union Bank P.O

Ans.74. 3 Lion

Ans.75. 5 Kaziranga

Ans.76. 2 Kerala

Ans.77. 4 corbett

Ans.78. 1Kerala

Ans.79. 4 Manas-Lion-M.P

State Animal

NP As Ma Ke Gu MP UP EI Ti Rh De Wi Li

Cor X X X ✓ X X X ✓ X X X X

Gir X ✓ X X X X ✓ X X X X X

www.bankexamstoday.com Page 98
Keo X X ✓ X X X X X X X ✓ X

Kan ✓ X X X X X X X X ✓ X X

Kaz X X X X X ✓ X X ✓ X X X

Man X x X X ✓ X X X X X X ✓

Note:
NP - National Park, Cor - Corbett,,
Gir - Gir forest, Keo - Keoladeo,
Kan - Kanha, Kaz - Kaziranga,
Man - Manas, Kaz - Kaziranga,
State:
As - Assam, Ma - Manipur,
Ke - Kerala, Gu - Gujarat
Animals :
EI - Elephant, Ti - Tiger,
Rh - Rhinoceros, De - Deer,
Wi - Wild ass, Li - Lion
National Park State Animal

Corbett Gujarat Tiger

Gir forest Manipur Elephant

Keoladeo Kerala Wild ass

Kanha Assam Deer

Kaziranga UP Rhinoceros

Manas MP Lion

Ans.80. 2 M

Ans.81. 2 Watching Tv

Ans.82. 4 Marketing Head

Ans.83. 2 L

Ans.84. 4 Principal

www.bankexamstoday.com Page 99
Solutions :
Post Hobby

Person En Pr Te Ba Gu Co Mh Do TV C Cr Da Si Rb

E X X X X x X X ✓ X X X X X ✓

L ✓ X X X X X X X X X X ✓ X X

O X X X X ✓ X X X X X ✓ X X X

P X ✓ X X X X X X X ✓ X X X X

X X X ✓ X X X X X X X X X ✓ X

M X X X ✓ X X X X X X X X x ✓

F X X X X X ✓ X X ✓ X X X X X

Q X X X X X X ✓ X X X ✓ X X X

Note : - En- Engineer, Pr - Principal, Te- teacher, Ba - Bank manager, Gu - Guard, Co -


Counsellor, Mh - marketing head, Do - Doctor, C - Cooking, Cr - Cricket, Da - Dancing,
Si - Singing, Rb - reading book.

Person Post Hobby

E Doctor Reading book

L Engineer Dancing

O Guard Playing Cricket

P Principal Cooking

X Teacher Singing

M Bank Manager Reading book

F Counsellor Watching TV

Q Marketing Head Playing Cricket

Ans.85. 3 M
www.bankexamstoday.com Page 100
Ans.86. 4 Floor no. 8-P-Coconut

Ans.87. 4 Sixth

Ans.88. 1 Four

Ans.89. 5 Other than the given options

Ans.90. 5 Other than the given options

Description:

Floor Person Fruit

8 P Coconut

7 Q Papaya

6 T Radish

5 S Cucumber

4 M Banana

3 N Apple

2 O Pineapple

1 R Orange

Ans.91. 1 Q

Ans.92. 4 S-Lavani Dancing

Ans.93. 1 V Bharatnatyam Acting

Ans.94. 5 Bihu

Ans.95. 2 R

Ans.96. 3 Mask Dance

Description:

www.bankexamstoday.com Page 101


Person Dance form Profession

P Mandi Anchoring

Q Nati Teaching

R Nautanki Business

S Lavani Dancing

T Bihu Banking

U Mask dance Writing

V Bharatnatyam Acting

Ans.97. 5 Both 3) and 4)

Ans.98. 3 Sangkara and Watson

Ans.99. 2 P, R and M

Ans.100. 1 Two

Ans.101. 3 P-Africa-Stain-Mumbai Indians

Ans.102. 4 Both 1) and 3)

Description:
Owner Country Captain Team

L Australia Fleming Rajasthan Royals

M Africa Flinto off Kolkata Knight Riders

N Sri Lanka Sangkara Pune Warriors

O Australia Akram Delhi Daredevils

P Africa Stain Mumbai Indians

Q Sri Lanka Watson Chennai Super Kings

R Africa V.Richards Kings XI Punjab

Ans.103. e

Ans.104. d

www.bankexamstoday.com Page 102


Ans.105. a

Ans.106. c

Ans.107. b

Ans.108. d

Ans.109. d

Ans.110. c

Ans.111. a

Ans.112. c

Ans.113. e

Ans.114. d

Ans.115. 3 Two

Ans.116. 1 Wednesday

Ans.117. 2 B

www.bankexamstoday.com Page 103


Ans.118. 4 C, D

Ans.119. 5 D

Ans.120. 4 Both 1) and 2)

Description:

Days Exhibition State


Monday A Tamil Nadu
Tuesday C Goa
Wednesday G Tamil Nadu
Thursday F Assam
Friday D Goa
Saturday B Maharashtra
Sunday E Assam

Note: - Mon-1, Tue-2, Wed-3, Thr-4, Fr-5, Sat- 6, Sun-7

Ans.121. 2 T

Ans.122. 1 R

Ans.123. 3 China

Ans.124. 4 P, R

Ans.125. 2 Badminton

Ans.126. 3 USA

Ans.127. 3 V-Cricket-Maruti-China

Solution:
Players Games Countries Cars

P Tennis India Honda

R Football USA Honda

S Badminton Britain Maruti

T Hockey Russia Mercedes

www.bankexamstoday.com Page 104


V Cricket China Maruti

Ans.128.1.Either T or X or Y. R is an a, and hence can marry any unmarried

Ans.129.2. R‘s niece or nephew

Ans.130.3. X is male. This generation (middle) contains three a‘s and two b‘s. if the
more numerous a's are female, b must be male.

Ans.131. (2)

Ans.132. (4)

Ans.133. (4)

Ans.134. (3)

Ans.135. (4)

Ans.141. 5

Ans.142. 3

Ans.143. 4

Ans.144. 1

Ans.145. 2
Descriptions:
L1= H(Math), G(Math) and B(Physics); L2= A(Chemistry), E(Chemistry); L3= D(Math),
F(Physics), C(Physics)

www.bankexamstoday.com Page 105


Ans.146. (2)

Ans.147. (4)

Ans.148. (4)

Ans.149. (3)

Ans.150. (4)

Ans.151. (3)

Ans.152. (1)

Ans.153. (2)

Ans.154. (2)

Ans.155. (3)

Person Company Floor

A HP II

B IBM IV

C Google III

D EY I

E Analog Device V

F Facebook VII

www.bankexamstoday.com Page 106


G HCL VI

Ans.156. (4)

Ans.157. (2)

Ans.158. (1)

Ans.159. (1)

Ans.160. (4)

Ans.161. (b)

Ans.162. (d)

Ans.163. (b)

Ans.164. (a)

Ans.165. (b)

Ans.166. C

Ans.167. D

www.bankexamstoday.com Page 107


Ans.168. B

Ans.169. E

Ans.170. A

Ans.171. 4

Ans.172. 4

Ans.173. 2

Ans.174. 2

Ans.175. 2

Solution :

Person Month Color

A August Red

B March Brown

C June Yellow

D December Green

E October Orange

F February Blue

G January White

www.bankexamstoday.com Page 108


Ans.176. 3

Ans.177. 2

Ans.178. 3

Ans.179. 5

Ans.180. 3

Solution:

Ans.181. Purple

Ans.182. Four

Ans.183. Ganesh

Ans.184. Silver

Ans.185. Black

Solution:
Floor Person Color Vehicle

9 Balaji Golden Car

8 Hari Silver Car

7 Ganesh White Car

6 Anil Yellow Bike

5 Feriq Black Bike

4 Eswar Purple Car

3 Ishant Gray Bike

www.bankexamstoday.com Page 109


2 Dinesh Orange Car

1 Chaitu Red Bike

Ans.186. d

Ans.187. d

Ans.188. a

Ans.189. b

Ans.190. e

Persons country game

A Jamaica Athletics

B Japan wrestling

C china archery

D USA gymnastics

E India swimming

F Russia tennis

G Brazil Rifles hooting

Ans.191. b

Ans.192. d

Ans.193. d

Ans.194. e

Ans.195. a

Ans.196. b

Ans.197. e

Persons attend meeting different days and like colors

www.bankexamstoday.com Page 110


Days Persons Different places Colors

Monday S Chennai Green

Tuesday P Delhi White

Wednesday N Mumbai Red

Thursday Q Surat Yellow

Friday O Pune Blue

Saturday M Kolkata Pink

Sunday R Baroda Black

Ans.198. c

Ans.199. d

Ans.200. e

Ans.201. d

Ans.202. c

Floor arrangement:

floor persons

8 kalyan

7 naresh

6 raghu

5 ravi

4 satish

3 tarun

2 pawan

1 manish

Ans.203. b

www.bankexamstoday.com Page 111


Ans.204. e

Ans.205. c

Ans.206. a

Ans.207. b

Berlin Toronto Kenya Quebec Frankfurt


R ------- ------- ------- ------- ------- L
A B C E D

L K J M N
L ------- ------- -------- -------- -------- R
Dubai Perth Zurich Ottawa Brisbane

Ans.208. 2

Ans.209. 1

Ans.210. 2

Ans.211. 1

Ans.212. 4

Arrangement:

Different months Name of the person Different fabric

January Y velvet

March T jute

April U cotton

July S silk

October W muslin

November V linen

www.bankexamstoday.com Page 112


December X khaddar

Ans.213. b

Ans.215. d

Ans.216. a

Ans.217. c

Line arrangement: Kishore, Vishal, Kumar, Kamal, Vimal, Amar, Jamal, Shekar

Ans.218. 3

Ans.219. 2

Ans.220. 2

Ans.221. 1

Ans.222. 1

Directions:

Ans.223. B

Ans.224. B

www.bankexamstoday.com Page 113


Ans.225. C

Order in which the Leaving Time from Monday Leaving Time on


buses leave to Friday Saturday and Sunday

P 8: a.m. 7:30 a.m.

N 8:45 a.m. 8:30 a.m.

R 9:15 a.m. 9:30 a.m.

M 10 a.m. 10:30 a.m.

Q 10:35 a.m. 11:30 a.m.

S 11:20 a.m. 12:30 a.m.

O 12 noon 1:30 a.m.


Ans.226. C
Ans.227. A Bus P leaves at 7:30 a.m. And completes its four hour sight-seeing tour at
11:30 a.m. Bus Q begins its tour at 11:30 a.m.

Ans.228. D Bus M leaves at 10:30 am. If time gap of 1 hr is increased by 30 minutes,


bus Q will leave at 1 noon; bus S at 1:30 p.m. and bus ) at 3 p.m. bus O will complete
its four hour tour at 7 p.m.

Ans.229. Bus N will leave at 8:45 a.m., R at 9:30 a.m., M at 10:15 a.m., Q at 11 a.m.,
S at 11:45 a.m. and O at 12:30 p.m. 4 hours before 12:30 p.m. is 8:30 a.m. No bus
starts at 8:30 a.m.

Ans.230. 2

Ans.231. 3

Ans.232. 3

Ans.233. 4

Ans.234. 5

Ans.235. 5

www.bankexamstoday.com Page 114


Ans.236. 1

Ans.236. 4.

Ans.238. 1

Ans.239. 3

Ans.240. 5

Direction: We have the following information:

P Architect
Q Lawyer or Professor
R Lawyer or Professor
S Engineer (Male)

www.bankexamstoday.com Page 115


T Professor or Manager (female)
U Doctor
V Student
ST is a married couple
S is the father of Q
S is grandfather of V
V is grandson of T

Ans.241. (d) Data inadequate


Since, for each of the pair, in (a), (b) and (c) we cannot have any confirmed idea about
the gender of each individual.

Ans.242. (d) Data inadequate


There is not enough information to say about the relationship between R and V.

Ans.243. (c) professor and Manager


From the above information that we obtained, the profession of T is either Professor or
Manager.

Ans.244. a J finishes first


J>P>k>L

Ans.245. b K finishes sixth


K > L and either J or O has to be the last.

Ans.246. c Second, third


J >P > M > N > K > L > O or
J >M > N >K >L >P> O

Ans.247. d L finishes third, J Finishes seventh


O > K > L > M > N > P > J or
O>K>L>P>M>N>J

Ans.248. c 4
(1) J > P > K > L > M >N > O
(2) J > K > P > L > M > N > O
(3) J > K > L > P > M > N > O
(4) J > K >L > M > N > P > O

Syllogism
www.bankexamstoday.com Page 116
Direction: In the following questions, two Conclusions and five statements are given. You
have to take the statements to be true even if they seem to be at variance with commonly
known facts and then decide which of the given statements logically follow.

Q. 1.
Conclusion:
No book is pen,
Some pens are bag.
Statements:
I. Some pens are book; No book is bag; some bags are erasers.
II. Some book are not pens; all pens are bag; No book is an eraser.
III. No pen is a bag; No bag is a book; all book are erasers.
IV. No book is an eraser; all pens are erasers; some bag are pen.
V. All pens are bag; all bag are book; all book are erasers.

Options
A. Only statement V.
B. Only statement III.
C. Only statement I.
D. Only statements II
E. Only statement IV.

Q. 2.
Conclusion:
Birds are definitely not swans,
All swans are duck.
Statements:
I. Some swans are ducks; all ducks are Birds; No peacock is a swan.
II. All ducks are peacock; all peacocks are swans; No peacock is a Bird.
III. All swans are peacock; all peacocks are ducks; No peacock is a Birds.
IV. Some swans are Birds; all Birds are ducks; some ducks are definitely not peacock.
V. Some swans are definitely not Birds; all swans are peacock; some Birds are ducks.

Options
A. Only statement V.
B. Only statement III.
C. Only statement I.
D. Only statements II
E. Only statement IV.

Q. 3.
Conclusion:
All papers are needles,

www.bankexamstoday.com Page 117


No needle is a cloth.
Statements:
I. All papers are clothes; No cloth is a needle; all needles are files.
II. No file is a cloth; all needles are files; all papers are clothes.
III. All papers are files; all files are needles; No cloth is a needle.
IV. No needle is a file; all papers are files; some clothes are needles.
V All clothes are files; all files are papers; No paper is a needle.

Options
A. Only statement V.
B. Only statement III.
C. Only statement I.
D. Only statements II
E. Only statement IV.

Q. 4.
Conclusion:
Some machines are definitely not racks;
Some machines are wheels.
Statements:
I. No rack is pin; some pins are machines; all wheels are machines.
II. All racks are machines; No machine is a wheel; all wheels are pins.
III. All wheels are racks; No rack is a machine; all machines are pins.
IV. All machines are racks; all racks are wheels; all wheels are pins.
V. No rack is pin; all machines are racks; all wheels are pins.

Options
A. Only statement V.
B. Only statement III.
C. Only statement I.
D. Only statements II
E. Only statement IV.

Q. 5.
Conclusion:
Some boxes are definitely not writers,
No writer is a desk.
Statements:
I. All desks are boards; all boards are writers; all writers are boxes.
II. No writer is a board; some boards are boxes; all desks are boards.
III. All boxes are writers; some boxes are desks; No desk is a board.
IV. No writer is a box; all boards are desks; some writers are boards.
V. No writer is a board; some writers are desks; all desks are boxes.

www.bankexamstoday.com Page 118


Options
A. Only statement V.
B. Only statement III.
C. Only statement I.
D. Only statement II
E. Only statement IV.

Directions: In each of the questions below, Some statements are given followed by some
conclusions. You have to consider the statements to be true even if they seem to be at
variance with commonly known facts. You have to decide which of the following conclusions
logically follows from the given statements. Give answer.

Q. 6.
Statements:
Some apple is orange.
Some orange is grapes.
No grapes are banana
Conclusions:
I. All orange being banana is a possibility.
II. All banana being orange is a possibility.

A. I and II follow.
B. Only II follow.
C. Only I follow.
D. Either I or II follows
E. Neither I nor II follows.

Q. 7.
Statement:
Some A are B.
Some B is C.
No C is E.
Some D is E.
No A is D.
Conclusions:
I. Some B is not D.
II. Some E is B.
III. Some B is not E.

A. Either II or III and I follows.


B. I and III follow.
C. Only I follow.
D. All follows
E. None of these.

www.bankexamstoday.com Page 119


Q. 8.
Statement:
All mouse is a keyboard.
Some keyboard is CPU.
No CPU is monitor.
Conclusion:
I. Some mouse is a keyboard.
II. Some keyboard are not monitor.

A. I and II follow.
B. Only I follows.
C. Only II follows.
D. None follow.
E. None of these

Q. 9.
Statement:
All day is a month.
All month are a date.
Some date are year.
Conclusion:
I. All day is a date.
II. Some year are a month.

A. Only II follows.
B. I and II follow.
C. Neither I nor II follows.
D. Only I follows.
E. None of these.

Q. 10.
Statement:
All pink are yellow.
Some yellow is black.
No white is pink.
Conclusion:
I. All white being yellow is a possibility.
II. Some pink is black.

A. Only I follows.
B. Only II follows.
C. I and II follow.
D. Either I or II follows.

www.bankexamstoday.com Page 120


E. None of these.

Directions: In each group of questions below are two conclusions followed by five
statements. You have to choose the correct set of statements that logically satisfies
given conclusions.

Q. 11.
Conclusions:
I. Some D are B.
II. Some A are not E.

Statements:
I. Some A are B. Some B are D. No D is E.
II. Some A are B. All B are D. No D is E.
III. All A are B. All B are D. Some D are E.
IV. All A are B. Some B are D. No D is E.
V. Some A are B. Some B are D. No B is E.

A. Both statements I and III.


B. Only statement III.
C. Only statement I.
D. Both statements II and V.
E. Only statement V.

Q. 12.
Conclusions:
I. All note being metal is a possibility.
II. No note is plastic.
Statements:
I. Some note are coin. No coin is plastic. All plastic are metal.
II. Some note are coin. No coin is plastic. Some plastic are metal.
III. All note are coin. No coin is plastic. All plastic are metal
IV. No note is plastic. Some plastic are coin. All metal are plastic.
V. All note are coin. No coin is plastic. All metal are plastic.

A. Both statements III and IV.


B. Only statement II.
C. Both statements I and V.
D. Only statement IV.
E. Only statement III.

Q. 13.

www.bankexamstoday.com Page 121


Conclusions: I. Some speaker are not mobile.
II. Some laptop are speaker.
Statements:
I. Some mobile are laptop. No laptop is speaker. Some speaker are charger.
II. No mobile is laptop. All laptop are charger. Some charger are speaker.
III. All mobile are laptop. No laptop is speaker. Some speaker is charger.
IV. No mobile is laptop. Some laptop are charger. All charger are speaker.
V. Some laptop are mobile. No laptop is charger. Some charger are speaker.

A. Only statement III.


B. Only statement I.
C. Both statements II and V.
D. Only statement IV.
E. Both statements I and IV.

Q. 14.
Conclusions: I. Some cow are dog.
II. All ox being cat is a possibility.
Statements:
I. Some dog are cat. Some cat are cow. Some cow are ox.
II. Some dog are cat. Some cat are cow. All cow are ox.
III. All dog are cat. All cat are cow. Some cow are ox.
IV. All dog are cat. Some cat are cow. Some cow are ox.
V. Some dog are cat. All cat are cow. Some cow are ox.

A. Only statement I.
B. Only statement III.
C. Both statements III and V.
D. Only statement IV.
E. Both statements II and IV.

Q. 15.
Conclusions: I. No pen is eraser.
II. All eraser being paper is a possibility.
Statements:
I. No paper is pen. Some pen are pencil. No pencil is eraser.
II. All paper are pen. Some pen are pencil. No pencil is eraser.
III. No paper is pen. All pen are pencil. No pencil is eraser.
IV. Some paper are pen. No pen is pencil. Some pencil are eraser.
V. All paper are pen. No pen is pencil. Some pencil are eraser.

A. Both statements III and IV.


B. Only statement III.
C. Only statement V.

www.bankexamstoday.com Page 122


D. Both statements I and V.
E. Only statement II.

Direction: Read the following questions carefully and choose the statement which
follows both the given conclusions.

Q. 16.
Conclusions:
i. All Orange are Apple
ii. No Orange is Carrot
Statements:
A. Some rose are Orange. All rose are Apple. No Orange is flower. Some flower is Carrot.
B. All Carrot is Apple. Some Apple are flower. No Orange is Apple. All rose are Orange.
C. All Orange are Apple. All Apple are Carrot. Some Apple are flower. No Orange is rose.
D. All rose are Apple. Some rose are Orange. No Apple is Carrot. All flower are Carrot.
E. None of these

Q. 17.
Conclusion:
i. All tab are mobile is a possibility
ii. Some Laptop is not Radio
Statements:
A. All mobile are pen drive. All pen drive are Radio. No Radio is tab. Some mobile are
Laptop.
B. No pen drive is Radio. All mobile are pen drive. All tab are pen drive. Some pen drive is
Laptop.
C. All tab are Radio. No Radio is mobile. Some mobile are Laptop. All Radio is pen drive.
D. Some pen drive are tab. Some Radio are Laptop. All mobile are Radio. No tab is mobile.
E. None of these

Q. 18.
Conclusions
i. All plant are Fruit
ii. Some Seed are Branch
Statements
A. All Leaf are Fruit. Some Leaf is Plant. No Leaf is Seed. Some Seed are Root. All Root are
Branch.
B. All Fruit are Root. All Root are Branch. Some Root are Seed. Few Branch are Plant. Few
Plant are Leaf.
C. All Plant are Leaf. Some Leaf is Seed. No Leaf is Fruit. Some Fruit are Branch. No Seed is
Root.
D. No Branch is Fruit. Some Fruit are Leaf. Some Leaf are Plant. All Plant are Seed.
E. None of these

www.bankexamstoday.com Page 123


Directions: In each of the questions below, Some statements are given followed by
some conclusions. You have to consider the statements to be true even if they seem to
be at variance with commonly known facts. You have to decide which of the following
conclusions logically follows from the given statements. Give answer.

Q. 19.
Statements:
Some water are cold drink.
No cold drink is Pepsi.
All Pepsi are juice.
All water are Shake.
Conclusions:
I. Some shake are cold drink.
II. Some shake are not pepsi.
III. All cold drink being juice is a possibility.

A. I and II follows.
B. II and III follows.
C. Only I follow.
D. All follow.
E. None of these.

Q. 20.
Statements:
All buses are car.
All cars are train.
Some train are aeroplane.
No car is cycle.
Conclusions:
I. Some trains are buses.
II. Some cars are aeroplane.
III. No bus is cycle.

A. I and III follows.


B. Either III or IV follows.
C. A and B.
D. All follows
E. None of these.

Q. 21.
Statements:
No shirt is skirt.
Some skirts are tops.
All tops are socks.
www.bankexamstoday.com Page 124
No socks is jeans.
Conclusions:
I. Some socks are skirt.
II. All skirts being jeans is a possibility.
III. Some shirts are tops.

A. I, II, III follows.


B. Only I follow.
C. I and II follows.
D. None follow.
E. None of these

Q. 22.
Statements:
Some water are cold drink.
No cold drink is pepsi.
All pepsi are juice.
All water are shake.
Conclusions:
I. All water being pepsi is a possibility.
II. Some juice are not cold drink.
III. Some water are juice.

A. II and III follows.


B. I and III follows.
C. I and II follows.
D. Only II follows.
E. None of these.

Q. 23.
Statements:
No shirt is skirt.
Some skirts are tops.
All tops are socks.
No socks is jeans.
Conclusions:
I. All top being shirt is a possibility.
II. Some skirt are jeans.
III. All socks are tops.

A. II and III follows.


B. Only I follows.
C. I, II and III follows.

www.bankexamstoday.com Page 125


D. None follow.
E. None of these.

Directions: In each of the questions below, Some statements are given followed by
some conclusions. You have to consider the statements to be true even if they seem to
be at variance with commonly known facts. You have to decide which of the following
conclusions logically follows from the given statements. Give answer.

Q. 24.
Statements:
Some pen are paper.
Some paper are book.
No eraser is color.
All book are eraser.
All copy are color.
Conclusions:
I. All paper being color is a possibility.
II. Some eraser are book.
III. Some pen are color.
IV. Some paper being color is a possibility.

A. II and III follows.


B. II and IV follows.
C. Only I follow.
D. All follow.
E. None of these.

Q. 25.
Statement:
No boys is girl.
Some girl are educated.
All educated are married.
All boys are unmarried.
Conclusions:
I. Some girls are married.
II. All boys being married is a possibility.
III. Some boys are married.
IV. Some boys are not married.

A. I and II follows.
B. Either III or IV follows.
C. A and B.
D. All follows
E. None of these.

www.bankexamstoday.com Page 126


Q. 26.
Statement:
Some pearl are stone.
Some stone are diamond.
No diamond is gold.
Some gold are silver.
Conclusion:
I. Some pearl are diamond.
II. Some pearl can be diamond.
III. All stone being silver is a possibility.

A. I, II, III follows.


B. II and III follows.
C. I and II follows.
D. None follow.
E. None of these

Q. 27.
Statement:
All good are bad.
Some bad are intelligent.
All intelligent are girl.
No girl is black.
Conclusion:
I. Some black being intelligent is a possibility.
II. Some intelligent is good.
III. Some girl are bad.
IV. All girl are good.

A. II and IV follows.
B. I and III follows.
C. Only IV follows.
D. Only III follows.
E. None of these.

Q. 28.
Statement:
All red are black.
All black are white.
Some red is yellow.
Some purple is yellow.
Conclusion:

www.bankexamstoday.com Page 127


I. Some white are yellow.
II. Some black are red.
III. Some purple are black.
IV. All red are white.

A. II and III follows.


B. Only I follows.
C. I, II and IV follows.
D. All follows.
E. None of these.

Q. 29.
Conclusion: Some pens are sharpener; No eraser is a pen.
Statement 1: Some pens are erasers. All erasers are sharpeners.
Statement 2: No pen is an eraser, some erasers are sharpeners.
Statement 3: Some sharpeners are erasers. All pens are erasers.
Statement 4: All sharpeners are pen. No pen is an eraser.
Statement 5: Some sharpeners are definitely erasers. No pen is a sharpener.
Mark your answer as
A. Only Statement 1 follows
B. Only Statement 2 follows
C. Only Statement 3 follows
D. Only Statement 4 follows
E. Only Statement 5 follows

Q. 30.
Conclusion: Some flowers are leaves; some roots are leaves.
Statement 1: Some roots are leaves. Some roots are flowers.
Statement 2: No flower is a root. All roots are leaves.
Statement 3: All flowers are roots. All leaves are roots.
Statement 4: No root is a flower. Some flowers are leaves.
Statement 5: All leaves are roots. All roots are flowers.

Mark your answer as


A. Only Statement 1 follows
B. Only Statement 2 follows
C. Only Statement 3 follows
D. Only Statement 4 follows
E. Only Statement 5 follows

Q. 31.
Conclusion: Some fans are definitely switches; all switches are tubes.
Statement 1: No switch is a tube. All switches are fans.

www.bankexamstoday.com Page 128


Statement 2: Some switches are tubes. Some tubes are fans.
Statement 3: All switches are fans. All fans are tubes.
Statement 4: All tubes are switches. No tube is a fan.
Statement 5: All tubes are switches. Some fans are definitely tubes.

Mark your answer as


A. Only Statement 1 follows
B. Only Statement 2 follows
C. Only Statement 3 follows
D. Only Statement 4 follows
E. Only Statement 5 follows

Q. 32.
Statements:-
1) All A are B
2) Some B are C
3) No B is D
Conclusion:-
1) No C is D
2) No A is D
3) Some C are not D

A. II and III follows.


B. Only II follows.
C. I, II and IV follows.
D. All follows.
E. None of these.

Q. 33.
Statements:-
1) All girls are good
2) Some good are women
Conclusions:-
(I). some women are girls
(II). No woman is a girl.

A. II and III follows.


B. Only II follows.
C. Either I or II follows.
D. All follows.
E. None of these.

Q. 34.

www.bankexamstoday.com Page 129


Statements:-
1) All pens are pencils
2) Some pens are erasers
3) All papers are books
4) All erasers are books
Conclusions:-
(I) some pencils are books.
(II) some books are pens.
(III) No paper is pen.
(IV) some erasers are papers.

A. II and III follows.


B. Only II follows.
C. I and II follows.
D. All follows.
E. None of these.

Directions: In each of the following questions below are given some statements followed by
some conclusions. You have to take the given statements to be true if they seem to be at
variance with commonly known facts. Read all the conclusions and then decide which one of
the given conclusions logically definitely does not follow:

Q. 35.
All soils are stones.
Some stones are hills.
All mountains are not clouds.
Conclusions:
l) Some soils being clouds is a possibility.
2) Some soils are hills is a possibility.
3) No stone is cloud.
4) All mountains being stones is a possibility.
5) No stone is a soil.

A. II and III follows.


B. Only II follows.
C. I and II follows.
D. I,II and IV follows
E. None of these.

Q. 36.
Some waters are streams.
All streams are canals.
All canals are rivers,

www.bankexamstoday.com Page 130


Some streams are ponds.
Conclusions:
l) All ponds are canals.
2) All canals are pond is a possibility
3) All streams are rivers.
4) All canals are waters is a possibility.
5) All streams are not canals.

A. II and III follows.


B. Only II follows.
C. I and II follows.
D. I,II and IV follows
E. None of these.

Q. 37.
Some trees are forests.
Some forests are animals.
All herbivores are pets.
All animals are not herbivores.
Conclusions:
1) Some trees are animals.
2) No pet is a herbivore.
3) Some trees are animals is a possibility.
4) All herbivores being trees is a possibility.
5) All forest being animals is a possibility.

A. II and III follows.


B. Only II follows.
C. III and IV follows.
D. I,II and IV follows
E. None of these.

Q. 38.
Some buses are trains.
Some trains are metros.
Some metros are smarts.
All smarts are electronic vans.
Conclusions:
1) No electronic van is a smart.
2) Some metros are electronic vans.
3) Some trains being electronic vans is a possibility.
4) Some electronic vans are buses is a possibility
5) Some trains are buses.

www.bankexamstoday.com Page 131


A. II, III, IV and V follows.
B. Only II follows.
C. I and II follows.
D. I,II and IV follows
E. None of these.

Q. 39.
All apples are oranges
All apples are not papayas.
Some papayas are guavas.
Some oranges are sweets.
Conclusions:
l) All apples being sweets is a possibility.
2) All guavas are sweets
3) All sweets being guavas is a possibility.
4) Some papayas are not apples.
5) No orange is sweet.

A. II and III follows.


B. Only II follows.
C. I and III follows.
D. I,II and IV follows
E. None of these.

Q. 40.
No bottle is a jar.
All cans are jars.
All cans are tumblers.
Conclusions (I) : At least some cans are bottles.
Conclusions (II) : No tumbler is a bottle

a. neither conclusion I nor II is true.


b. both conclusions I and II are true.
c. only conclusion II is true.
d. either conclusion I or II is true.
e. only conclusion I is true.

Q. 41.
Some prints are designs.
All designs are copies.
All copies are motifs.
Conclusions (I) : At least some copies are prints.
Conclusions (II) : All motifs being prints is a possibility

www.bankexamstoday.com Page 132


a. either conclusion I or II is true.
b. only conclusion II is true.
c. only conclusion I is true.
d. neither conclusion I nor II is true.
e. both conclusions I and II are true.

Q. 42.
No bottle is a jar.
All cans are jars.
All cans are tumblers.
Conclusions (l) : All tumblers are jars
Conclusions (II) : All bottles being tumblers is a possibility.

a. neither conclusion I nor II is true.


b. both conclusions I and II are true.
c. only conclusion II is true.
d. either conclusion I or II is true.
e. only conclusion I is true.

Q. 43.
Some prints are designs.
All designs are copies.
All copies are motifs.
Conclusions (I) : At least some prints are motifs
Conclusions (II) : All designs are motifs

a. only conclusion I is true.


b. both conclusions I and II are true.
c. either conclusion I or II is true.
d. neither conclusion I nor II is true.
e. only conclusion II is true.

Q. 44.
All clouds are vapors.
No vapors is a gas.
All gases are rains.
Conclusions (I) : All vapors being rains is a possibility.
Conclusions (II) : All vapors are clouds

a. either conclusion I or II is true.


b. both conclusions I and II are true.

www.bankexamstoday.com Page 133


c. only conclusion I is true.
d. only conclusion II is true.
e. neither conclusion I nor II is true.

Directions :In each of the questions below are given three statements followed by three
conclusions numbered I, II and III. You have to take the three given statements to be
true even if they seem
to be at variance from commonly known facts, and then decide which of the given
conclusions logically follows from the three given statements disregarding commonly
known facts. Then decide which of answer (1), (2), (3), (4) and (5) is the correct answer

Q. 45.
Some tables are jugs.
Some jugs are pots.
All pots are plates.
Conclusions
I. Some plates are jugs.
II. Some pots are tables.
III. Some plates are tables.

1. None follows
2. Only I follows
3. Only II follows
4. Only II follows
5. only II and III follows

Q. 46.
All chairs are rings.
Some rings are sticks.
All sticks are branches.
Conclusions
I. Some branches are chairs
II. Some branches are rings
III. Some sticks are chairs

1. None follows
2. Only I follows
3. Only II follows
4. Only III follows
5. Only I and II follow

Q. 47.
All bulbs are chairs.

www.bankexamstoday.com Page 134


All chairs are tables.
All tables are mirrors.
Conclusions
I. Some mirrors are bulbs.
II. Some tables are bulbs.
III. All chairs are mirrors.

1. Only I and II follow


2. Only I and III follow
3. Only II and III follow
4. All follow
5. None of the above

Q. 48.
All knives are hammers.
No hammer is sword
Some swords are nails
Conclusions
I. Some nails are hammers.
II. Some swords are knives.
III. No nail is hammer.

1. None follows
2. Only either I or III follows
3. Only II follows
4. Only III follows
5. Only I follows

Q. 49.
Some fruits are trees.
All trees are jungles
All jungles are roads
Conclusions
I. All fruits are jungles
II. Some roads are fruits
III. Some jungles are fruits

1. Only I and II follow


2. Only I and III follow
3. Only II and III follow
4. All follow
5. None of the above

Q. 50.

www.bankexamstoday.com Page 135


Some books are pens.
Some pens are desks.
Some desks are racks
Conclusions
I. Some racks are pens
II. Some desks are books
III. Some racks are books

1. Only I follows
2. Only II follows
3. Only III follows
4. None follows
5. All follow

Q. 51.
No room is house.
No house is building
Some buildings are huts.
Conclusions
I. Some huts are rooms
II. Some huts are houses
III. Some huts are buildings

1. Only I follows
2. Only II follows
3. Only III follows
4. None follows
5. All follow

Directions : In these questions, two/three statements followed by two conclusions


numbered I and II have been given. You have to take the given statements to be true even
if they seem to be at variance from the commonly known facts and then decided which of
the given conclusions logically follows
from the given statements disregarding commonly known facts.5

Q. 52.
Some shows are plays. Some plays are movies. No movie is a theater.
Conclusion I: All movies are plays.
Conclusion II: Some shows are definitely not theaters.
(a) Only conclusion I is true
(b) Only conclusion II is true
(c) Both conclusion I and II are true
(d) Either conclusion I or II is true
(e) Neither conclusion I nor II is true

www.bankexamstoday.com Page 136


Q. 53.
Some urns are jugs. All jugs are vessels. No vessel is a plate.
Conclusion I: Some plates being jugs is a possibility.
Conclusion II: All urns can never be plates.
(a) Only conclusion I is true
(b) Only conclusion II is true
(c) Both conclusion I and II are true
(d) Either conclusion I or II is true
(e) Neither conclusion I nor II is true

Q. 54.
All streets are paths. Some streets are roads. All roads are trails.
Conclusion I: All trails being paths is a possibility.
Conclusion II: Atleast some paths are roads.
(a) Only conclusion I is true
(b) Only conclusion II is true
(c) Both conclusion I and II are true
(d) Either conclusion I or II is true
(e) Neither conclusion I nor II is true

Directions:
Decide which of the given five conclusions logically follows the set of statements.
Q. 55.
Some logos are patents. All patents are copyrights. Some copyrights are trademarks. No
copyright is goodwill.
Conclusions:
(A) At least some trademarks are patents.
(B) Some patents are goodwill.
(C) All trademarks being logos is a possibility.
(D) No goodwill is a logo.
(E) All logos are copyrights.

Q. 56.
No text is a font. Some fonts are scripts. Some scripts are essays. All essays are chapters.
Conclusions:
(A) Some fonts are definitely not chapters.
(B) No text is an essay.
(C) All fonts are essays.
(D) All texts being scripts is a possibility.
(E) At least some chapters are texts.

Directions: In each questions below, there are three statements followed by four
conclusions numbered I, II, III and IV. You have to take the given statements to be true

www.bankexamstoday.com Page 137


even if they seem to be at variable with commonly known facts and then decide which of the
given conclusions logically follow(s) from the given statements.

Q. 57.
Some keys are lock.
All keys are doors.
No windows are door.
Conclusions:
I. All doors are key.
II. No windows are keys.
III. Some locks are not windows.
IV. Some locks are windows.

a) I and either III or IV follow


b) Only I and II follow
c) Only II and III follow
d) II and either III or IV follow
e) Either III or IV follows

Q. 58.
Some venues are matches.
No match is a nest.
All nests are regions.
Conclusions:
I. Some region are nests.
II. Some region are not matches.
III. Some venues are not nests.
IV. Some nest are not matches.

a) Only I, III and IV follow


b) Only I, II and IV follow
c) Only I, II and III follow
d) Only II, III and IV follow
e) All follow.

Q. 59.
No press is a photo.
All blocks are press.
Some photos are scholars.
Conclusions:
I. No blocks are photos.
II. Some scholars are photos.
III. Some blocks are photos.
IV. Some scholars are not blocks.

www.bankexamstoday.com Page 138


a) Only I, II and IV follow
b) Only I, III and IV follow
c) Only I, II and III follow
d) Only II and IV follow
e) Only II, IV and either I or III follow

Q. 60.
All inlets are channels.
No channels are bays.
Some gulfs are bays.
Conclusions:
I. Some gulfs are not inlets.
II. Some gulfs are not channels.
III. Some channels are not gulfs.
IV. No inlets is a bay.

a) Only I, III and IV follow


b) Only II, III and IV follow
c) Only II and IV follow
d) Only III and II follow
e) Only I, II and IV follow

Q. 61.
Some spaces are spheres.
No area is an extent.
All extents are spheres.
Conclusions:
I. Some areas are spheres.
II. No area is an extent.
III. No areas are spheres.
IV. Some spheres are not areas.

A) Either I or III and IV follow


B) Either I or III, II and IV follow
C) Either I or IV follows
D) Only II and IV follow
E) Only II follows.

Direction: In each question below are given two or three statements followed by two or
three conclusions numbered I, II and III. You have to take the given statements to be true
even if they seem to be at variance with commonly known facts and then decide which of
the given conclusions logically follows from the given statements, disregarding commonly
known facts. Give Answer.

www.bankexamstoday.com Page 139


Q. 62.
Some Boys are Girls.
All Boys are Madam.
Conclusions:
I. Those Boys which are not Girls are also Madam.
II. Those Boys which are not Girls are not necessarily Madam.

a) Only Conclusion I follows.


b) Only Conclusion II follows.
c) Both I & II follow.
d) Either I or II follow.
e) None follows.

Q. 63.
All Cows are Milk.
No Milk is White.
Conclusions:
I. Some Cows are White.
II. No Cow is White.

a) Either I or II follow.
b) Only Conclusion I follows.
c) Both I & II follow.
d) Only Conclusion II follows.
e) None follows.

Q. 64.
Some Pens are Pen-drive.
All Printers are Pen-drive.
Conclusions:
I. All Pens being Printers is a possibility.
II. No Printer is a Pen.

a) Only Conclusion II follows.


b) Both I & II follow.
c) Only Conclusion I follows.
d) Either I or II follow.
e) None follows.

Q. 65.
No Stone is a Metal.
Some Metals are Paper.
All Papers are Glass.

www.bankexamstoday.com Page 140


Conclusions:
I. All Stones being Glass being is a possibility.
II. No Stone is a Paper.

a) Only Conclusion I follows.


b) Only Conclusion II follows.
c) Both I & II follow.
d) Either I or II follow.
e) None follows.

Q. 66.
Some Apples are Cake.
Some Cakes are Candle.
Conclusions:
I. All Cakes are Apple.
II. Some Apples are Candle.
III. No Apple is a Candle.

a) Either I or II follow.
b) Only Conclusion I follows.
c) Both I & II follow.
d) Only Conclusion II follows.
e) None follows.

Give answer (1) if either conclusion I or conclusion II follows.


Give answer (2) if neither conclusion I nor conclusion II follows.
Give answer (3) if only conclusion I follows
Give answer (4) if both the conclusion I and conclusion II follows.
Give answer (5)if only conclusion II follows.

Q. 67.
Statements
All amounts are principals
Some principal are interest
Conclusion:
I All interest being amount is a possibility
II Some interest are definitely not principal.

Q. 68.
Statements
Some huts are mansions
No mansion is a palace.
Conclusions
I All palaces being hut is possibility

www.bankexamstoday.com Page 141


II All huts are palaces

Q. 69.
Statements
All bicycles are cars .
Some cars are trucks .
No truck is a crane .
Conclusions:
I. All bicycles being truck is a possibility.
II. Some cars are not cranes.

Q. 70.
Statements
All leaves are flowers
No flower is a branch
Some branches are trees.
Conclusion
I. Some trees are leaves
II. No tree is a flower

Q. 71.
Statements
Some flats are apartments.
No apartment is a hall
Some halls are rooms
Conclusions :
I. At least some rooms are flats .
II. No apartment is a room.

Q. 72.
Statements
Some codes are secrets
All secrets are puzzles
Conclusions
I. All secrets being code is a possibility.
II. At least some puzzles are codes.

Q. 73.
Statements
Some wins are trophies
Some trophies are cups
No cup is a prize.
Conclusion
I. At least some cups are wins .

www.bankexamstoday.com Page 142


II. All prizes being trophies is a possibility .

Q. 74.
Statements
Some stars are moons .
All moons are planets .
No planet is universe.
Conclusion :
I . At least some planets are stars.
II No moon is universe.

Answers

Ans. 1.E
Solution:

Ans. 2.B
Solution:

Ans. 3.B
Solution:

www.bankexamstoday.com Page 143


Ans. 4.C
Solution:

Ans. 5. D
Solution:

Ans. 6.B.
Solution:

www.bankexamstoday.com Page 144


Ans. 7.B.
Solution:

Ans. 8.A.
Solution:

Ans. 9.D
Solution:

Ans. 10.A.

www.bankexamstoday.com Page 145


Solution:

Ans. 11.D
Solution:

Ans. 12.E
Solution:

Ans. 13.D
Solution:

Ans. 14.C
Solution:

www.bankexamstoday.com Page 146


Ans. 15.B
Solution:

Ans. 16.D
Solution:
From Statement A
Both Conclusions do not follow

From Statement B
Conclusion (i) Don‘t Follow
Conclusion (ii) Follow

www.bankexamstoday.com Page 147


From Statement C
Conclusion (i) Follows
Conclusion (ii) do not follow

From Statement D
Here the shaded region clearly says, Both the conclusion Follows

Ans. 17. B
Solution:

www.bankexamstoday.com Page 148


From Statement A, Both the conclusions don‘t follow

From Statement B
From the shaded region, it is clear that Both the conclusions follow

From Statement C
Conclusion (i) don‘t follow
Conclusion (ii) follow

From Statement D
Both the conclusion don‘t follow

Ans. 18. A
Solution:

www.bankexamstoday.com Page 149


From Statement A
Shaded region clearly says both the conclusions follow

From Statement B,
Conclusion (i) don‘t follow
Conclusion (ii) follow

From Statement C
Both the conclusion don‘t follow

From Statement D
Both the conclusions don‘t follow

Ans. 19.D

www.bankexamstoday.com Page 150


Solution:

Ans. 20.A
Solution:

Ans. 21.B
Solution:

Ans. 22.D
Solution:

Ans. 23.D

www.bankexamstoday.com Page 151


Solution:

Ans. 24.B.
Solution:

Ans. 25. A
Solution:

Ans. 26.B
Solution:

Ans. 27.D
Solution:

www.bankexamstoday.com Page 152


Ans. 28.C
Solution:

Ans. 29.D
Solution:

You should have complete knowledge of the basics of syllogisms and Venn diagrams in order
to understand the new pattern. In these questions, the statement should satisfy both the
conditions given in the conclusion.
Conclusion:
Some pens are sharpeners. (condition 1)
No eraser is a pen. (Condition 2)

Statement 1:
Some pens are erasers. All erasers are sharpeners.

Not satisfying condition 2 (No eraser is a pen).
There is no need to make Venn diagram as we can directly check that condition 2 is not
satisfied.

Statement 2:
No pen is an eraser, some erasers are sharpeners.

No relation is given between pen and sharpener. So, we cannot conclude some pens are
sharpener (condition 1).

Statement 3:
Some sharpeners are erasers. All pens are erasers.

www.bankexamstoday.com Page 153


Not satisfying condition 2(No eraser is a pen).
Statement 4:
All sharpeners are pen. No pen is an eraser.

We can conclude that some pens are sharpener (condition 1) and no eraser is a pen
(Condition 2). Thus, statement 4 is the correct option.

Statement 5:
Some sharpeners are definitely erasers. No pen is a sharpener.

Not satisfying condition 1(Some pens are sharpeners)
Thus, the answer is D (Only Statement 4 follows)

Ans. 30.E
Solution:
In these questions, the statement should satisfy both the conditions given in the conclusion.
Statement 1:
Some roots are leaves. Some roots are flowers.

No relation is given in flowers and leaves. So some flowers are leaves (Condition 1) cannot
be concluded

www.bankexamstoday.com Page 154


Statement 2:
No flower is a root. All roots are leaves.

Again, no relation is given in flowers and leaves. So, some flowers are leaves (Condition 1)
cannot be concluded.

Statement 3:
All flowers are roots. All leaves are roots.

No relation is given in flowers and leaves. So some flowers are leaves (Condition 1) cannot
be concluded.

Statement 4:
No root is a flower. Some flowers are leaves.

No relation between roots and leaves is given. So, we cannot conclude some roots are
leaves (Condition 2).
www.bankexamstoday.com Page 155
Statement 5:
All leaves are roots. All roots are flowers.
We can conclude from the diagram that some flowers are leaves (Condition 1) and some
roots are leaves (Condition 2).
Therefore, statement 5 is the correct option.

Ans. 31.C
Solution:
Conclusion:
Some fans are definitely switches. (Condition 1)
All switches are tubes. (Condition 2)

Statement 1:
No switch is a tube. All switches are fans.

Not satisfying all switches are tubes (Condition 2)

Statement 2:
Some switches are tubes. Some tubes are fans

No relation is given in switches and fans. So, we cannot conclude that some fans are
definitely switches (Condition 1).

Statement 3:
All switches are fans. All fans are tubes.

www.bankexamstoday.com Page 156


Thus answer is C (Only Statement 3 follows)

Ans. 32. A
Solution:

Above diagram is the Block Diagram (BD) from the given statements.
Conclusion (I) is negative & is true.
Conclusion (II) is negative & is true.
Conclusion (III) is negative & is true.
To prove conclusion (I), ―No C is D‖, to be false, we have to prove that its complementary
conclusion, ―Some C are D‖ is true. Hence, the AD for this will be as shown below.

The above diagram does not defy any of the given statements. Conclusion (I), ―No C is D‖,
is false for this diagram, hence is not valid.
To negate conclusion (II), ―No A is D‖, & (III), ―Some C are not D‖, we have to prove that
their respective complementary conclusions ―some A are D‖ & ―All C are D‖, are true. This
possible, only if ‗D‘ encroaches into ‗B‘. But this will violate statement (3). Hence, no
diagram can be drawn to negate these two conclusions.
Hence, only conclusion (II) & (III) follows.
Thus, we are able to answer the question with only two diagrams.

www.bankexamstoday.com Page 157


 From BD, we can verify the truthfulness of each statement & accordingly decide
whether a conclusion follows the given statements or not.
 In the case of a negative conclusion becoming true, we go for Alternate Diagram to
prove the negative conclusion false. If such AD is possible, the negative statement does
not follow, otherwise, it follows the given statement.

Ans. 33. C
Solution:
Clearly, the two conclusions form a complementary pair.
Basic Diagram:-

Conclusion (I) is affirmative & is false.


Conclusion (II) is negative & is true.
Now, we should draw an AD, which would make the conclusion (II) false i.e., which proves
―Some women are girls‖.
Alternate Diagram:-

From the above diagram, conclusion (II) is false, but conclusion (I) is true at the same time.
But they both cannot be true or false at the same time. Hence, either (I) or (II) follows.

Ans. 34.C
Solution:
The given statements can be represented in the following basic diagram.
Basic Diagram:-

www.bankexamstoday.com Page 158


From the above diagram,
Conclusion (I), affirmative, follows.
Conclusion (II), affirmative, follows.
Conclusion (III), negative follows.
Conclusion (IV), affirmative, does not follow.
As the affirmative conclusions (I & II) are true in the BD, they will always be true. The
affirmative conclusion (IV) is false in the basic diagram. Even if it is true in other diagrams,
it cannot be said to be true as there is a situation, where it is false.
The negative conclusion (III), which is true in the basic diagram, has to be checked whether
it can be false in any alternate diagram. The following is such diagram.
Alternate Diagram:-

There is a situation, where conclusion (III) is false. Hence, only (I) & (II) are true & follows.

Ans. 35.D
Solution:
There is no definite relation between soil and cloud. But the possibility in 1) exists. Hence
conclusion l) follow.
Again, All soil are stones (A) + some stones are hills (I) = A + I = No conclusion. But the
possibility in 2) exists and the possibility between mountains and stone also exists. Hence
conclusion 4) follows Now, All mountains are not clouds (O) some mountains are not clouds
(O). So, we can't say no stone is cloud. Hence 3) does not follow. Again, All
soils are stone (A) — conversion some stones are soils (I). Hence 5) does not follow.

Ans. 36. A
Solution:
Some streams are ponds (I) conversion Some pounds are streams (I) + All streams are
canals (A) = I+A Some ponds are canals. Thus conclusion l) may follows and possibility in 2)
exist. Hence the conclusion 2) follows.
www.bankexamstoday.com Page 159
Again, All streams are canals (A) + All canals are rivers (A) = A + A = A = All stream are
rivers (A). Hence 3) follows. Now, Some waters are streams (I) + All streams are canals (A)
= I + A = I = Some waters are canals (I). Thus, possibility in 4) exists. Hence 4) follows.
From statements second conclusion 5) does not follow.

Ans. 37. C
Solution:
From statements first and second.
Some trees are forest (I)+ Some forests are animals (I)=I+I = No conclusion. But 1) may
follows and possibility in 3) exists. Hence conclusion 3) follows. Again, all herbivores are
pets (A) – conversion – Some pets are herbivores (I). Hence conclusion 2) does not follow.
From statements second possibility in 5) exist. Hence 5) follows. And possibility in 4) also
follows.

Ans. 38. A
Solution:
Some metros are Smart (I)+ All smart are electronic vans (A)= I+A=I= Some metros are
electronic vans (I). Hence conclusion 2) follows. But from statement four conclusion 2)
follows. But from statement four conclusion 1) does not follow. There is no negative
statements. So possibility in 3) and 4) exist. Hence conclusion 3) and 4) follow.
Again, conclusion 5) follows from statement first.

Ans. 39. C
Solution:
All apples are oranges (A)-conversion – some oranges are apples (I)+ All apples are not
papayas (O)-conversion – some apples are not papayas (O)=I+O= No conclusion.
So, the possibilities in 1) and 3)exist. Again, conclusion 5) does not follow from statement
four.

Ans. 40.A
Solution:

Ans. 41.E

Solution:

www.bankexamstoday.com Page 160


p-prints
d-designs
c-copies
m-motifs

Ans. 42.C
Solution:

Ans. 43.B
Solution:

p-prints
d-designs
c-copies
m-motifs

www.bankexamstoday.com Page 161


Ans. 44.C
Solution:

Ans. 45.2

Ans. 46.3

Ans. 47.4

Ans. 48.2

Ans. 49.3

Ans. 50.4

Ans. 51.3

Ans. 52.(e)
Solution:

www.bankexamstoday.com Page 162


I. Can't say II. Can't say
Ans. 53.(b)
Solution:

Ans. 54. C
Solution:

Ans. 55. C
Solution:

Ans. 56.D
Solution:

www.bankexamstoday.com Page 163


Ans. 57.C
Ans. 58.E
Ans. 59.A
Ans. 60.E
Ans. 61.B
Ans. 62.a
Solution:

Ans. 63.d
Solution:

www.bankexamstoday.com Page 164


Ans. 64.c
Solution:

Ans. 65.a
Solution:

Ans. 66.e
Solution:

Ans. 67.(3)
Solution:

www.bankexamstoday.com Page 165


Fig 1.1
Conclusion I
From this fig 1.2 conclusion I is true as there is possibility of some principal
being interest is a possibility

Fig1.2
This is also possible so conclusion I is true
Conclusion II
From Fig 1.1 and1.2 we cant conclude that definitely that some interest are
principal or not so conclusion II is false.

Ans. 68. (3)


Solution:

Conclusion I is true as there is possibility of all palaces being hut .

www.bankexamstoday.com Page 166


Conclusion II
Conclusion II does not follow as all huts can be places bcz some huts are
mansion and no mansion is place.

Ans. 69. (4)


Solution:

fig3.1
Conclusion I
Conclusion I will follow as all bicycles being is a possibility .

fig3.2
Conclusion II
From fig3.1 cars which are trucks can‘t be crane so conclusion II is correct.

Ans. 70. (2)


Solution:

www.bankexamstoday.com Page 167


Conclusion I will not follow as from given fig there is not definite that there can
be some tress are leaves.
Conclusion II will not follow .

Ans. 71. (2)


Solution:

Conclusion I does not follow as there is no definite that at least some room are
flats or not.
Conclusion II does not follow as we are not definite that no apartment is room
or not .

Ans. 72. (4)


Solution:

Conclusion I will follow as there can be possibility.

www.bankexamstoday.com Page 168


Conclusion II will also follow .

Ans. 73. (5)


Solution:

Conclusion I will not follow as there is not definite that some cups are wins or
not
Conclusion II will follow as there can be possibility of all prizes being trophies.

Ans. 74. (4)


Solution:

Conclusion I will follow


Conclusion II will follow

www.bankexamstoday.com Page 169


Blood Relation

Direction(Q.1-3):
A$B means A is sister of B
A@B means A is father of B
A>B means A is wife of B
A<B means A is son of B
A=B means A is mother of B

1. Which of the following expressions means „P‟ is daughter of „S‟?


a) P<Q>S@R
b) S=Q>P<R
c) S>R@P$Q
d) P@S>R<Q
e) R@Q$S=P

2. How is „Q‟ related to „R‟?


P@Q=S>R<T
a) Mother
b) Son
c) Father
d) Father-in-law
e) Mother-in-law

3. How is „T‟ related to „V‟?


X$Y@V<U$T
a) Uncle
b) Aunt
c) Cousin sister
d) son
e) C.N.D.

4. M is the son of Q. N, M‟s sister has a daughter R and a son E. Y is


maternal aunt of R.
How is Y related to N? If N has only one brother.
a) Brother-in-law
b) Sister
c) Mother
d) Sister-in-law
e) Aunt

www.bankexamstoday.com Page 170


5. P is the son of Q. Q is the mother of V. V is the sister of U. U is the
father of X. Y is the daughter of P. V is the wife of N.
How is „Y‟ related to „x‟?
a) Sister
b) Brother
c) Cousin sister
d) Aunt
e) Mother

Direction(Q.6-8): Q is the son of A. P is the husband of A.Q has two


children S and T. S is the daughter of R. M is the sister of Q. T is the brother
of S.

Q.6 How is M related to R?


A. Sister
B. Mother
C. Sister-in-law
D. Aunt
E. None of these

Q.7 How many female members in the family?


A. 4
B. 5
C. 3
D. 2
E. None of these

Q.8 How many male member in the family?


A. 4
B. 3
C. 2
D. 5
E. None of these

Direction(Q.9-10): In a family there are two couples, two son and one
daughter.
C is the grandfather of S.
S is the son of M.
K is the grandmother of P and is the wife of C.
P is the daughter of Q.

Q.9 How is P related to K?

www.bankexamstoday.com Page 171


A. Grandson
B. Son
C. Granddaughter
D. Daughter
E. None of these

Q.10 Which of the following is a pair of couple?


A. MQ
B. CP
C. SP
D. KQ
E. None of these

Directions(Q.11-15):
I. M, P, S, T, R, Q, O, K, U and N are the 10 members of a family.
II. P is the daughter of M and the sister of R.
III. Q is the son in law of O.
IV. M is the grandfather of T and U.
V. P is the sister in law of S.
VI. R is the maternal uncle of N and K.
VII. N is the only son of Q.
VIII. T and U are the sisters.

Q.11 How is P related to T?


A. Daughter
B. Sister
C. Aunt
D. Mother
E. None of these.

Q.12 Which of the following is true?


A. R is the sister of P.
B. K is the son of S.
C. O is the husband of M.
D. S is the daughter in law of M.
E. None of these.

Q.13 How many couples are there in the family?


A. 3
B. 4
C. 5
D. 1
E. 2

www.bankexamstoday.com Page 172


Q.14 What is R's relation with Q?
A. Son in law
B. Husband
C. Wife
D. Sister in law
E. Brother in law

Q.15 How many male members in the family?


A. 5
B. 6
C. 3
D. 2
E. 4

Q.16.P's father is Q's son. M is the paternal uncle of P and N is the brother
of Q .How is N related to M?
(a) Brother
(b) Nephew
(c) Cousin
(d) Data inadequate
(e) None of the above

Q.17.B is the husband of P. Q is the only grandson of E, Who is the wife of D


and mother-in-law of P. How is B related to D?
(a) Cousin
(b) Son -in-law
(c) Son
(d) Nephew
(e) None of these

Q.18.A retiring President was asked who would succeed him as head
of the firm , and he replied ,"The father of my successor is my
father's son, but I have no brothers or sons.” Who will succeed him ?
(a) Himself
(b) His nephew
(c) his Daughter
(d) His sister

Q.19.Looking at a portrait, a man said ,"That man's father is my


father's son. Brothers and Sisters I have none." At whose portrait
was the man looking ?
(a) His own
(b) His son's

www.bankexamstoday.com Page 173


(c) His father's
(d) His uncle's

Q.20.Pointing to the lady in the photograph , Seema said, "Her son's


father is the son-in-law of my mother." How is Seema related to the
lady?
(a) Sister
(b) Mother
(c) Cousin
(d) Aunt

Q.21.Kailash pointing towards an old man said," His son is my son's


Uncle." How is kailash related to the old man?
(a) Brother
(b) Uncle
(c) Father
(d) Grandfather

Q.22.Pointing to a man, Rohit said,"His son is my son's uncle." How


is the man related to Rohit?
(a) Father
(b) Grandfather
(c) Uncle
(d) Brother

Q.23.Rashmi is Ram's mother's daughter's daughter. What is her


relationship to Ram?
(a) Aunt
(b) Daughter-in-law
(c) Friend
(d) Niece

Q.24. A is D's brother and D is B's father. B and C are sisters. How is
C related to A?
a. Cousin
b. Niece
c. Aunt
d. Nephew

Q.25. If X is the brother of the son of Y's son., then how is X related
to Y?
a. Son
b. Brother
c. Nephew
d. Grandson

www.bankexamstoday.com Page 174


Q.26. Pointing towards a girl in the picture, Sunita said, "She is the
mother of Renu whose father is my son." How Sunita is related to
that girl in the picture?
a. Mother
b. Aunt
c. Cousin
d. Data inadequate
e. None of the above

Q.27. A man said to a woman, "Your only brother's son is my wife's


brother." How is the woman related to the man's wife?
a. Aunt
b. Sister
c. Mother
d. None of these

Q.28. A and B form a married couple. X and Y are the brothers. X is


the brother of A. How is Y related to B?
a. Brother
b. Cousin
c. Son-in-law
d. Brother-in-law

Q.29. A and B both are children of C. If C is the mother of A, A is the


son of C but B is not the daughter of C, then how are A and B
mutually related?
a. A is the brother of B
b. A is the nephew of B
c. A is the sister of B
d. A is the cousin of B

Q.30. A, B and C are sisters. D is the brother of E and E is the


daughter of B. How is A related to D?
a. Sister
b. Aunt
c. Niece
d. Cousin

Q.31. A husband and wife had five married sons and each of these
had four children. How many members are there in the family?
a. 50
b. 40
c. 32

www.bankexamstoday.com Page 175


d. 36

Q.32. Satish is the son of my father's sister's brother's wife's


daughter's grandchild. What is the closest relationship to me that
Satish could have?
a. Son
b. Cousin
c. Nephew
d. Father

Q.33.
(A) K is the brother of J.
(B) m is the sister of K.
(C) P is the brother of N.
(D) N is the daughter of J.
(E) S is the father of M.
Who is the uncle of P?
a. M
b. K
c. N
d. S

Q.34. Deepika tells Shraddha “Your mother‟s father‟s own son is the
husband of my sister.” How is Deepika related to Shraddha?
a)Sister-in-law
b) Cousin
c) Aunt
d) Data inadequate
e) None of these

Q.35.D is A‟s son. C is the mother of P and wife of D. How is A


related to C?
a) Father
b) Uncle
c) Father-in-law
d) Data inadequate
e) None of these

Q.36. Pointing to the lady in the photograph, Mrinalini said, “Her


son‟s father is the only son-in-law of my mother”. How is Mrinalini
related to the lady.
a) Sister
b) Mother

www.bankexamstoday.com Page 176


c) Cousin
d) Aunt
e) None of these

Q.37. A is brother of R. C is mother of B. M is sister of C. How is M


related to B?
a) Nephew
b) Niece
c) Aunt
d) Cannot be determined

Q.38. R is the daughter of Q. M is the sister of B who is the son of Q.


How M is related to R?
a) Cousin
b) Niece
c) Sister
d) Aunt
e) None of these

Q.39. Pointing to a photograph of Hari, Vijay said, “The father of his


sister is the husband of my wife‟s mother‟. How is Vijay related to
Hari?
a) Brother
b) Brother-in-law
c) Uncle
d) Data inadequate
e) None of these

Q.40. M is N‟s brother. S is D‟s mother and M‟s aunt. How is D related
to M?
a) Sister
b) Cousin
c) Aunt
d) Cannot be determined
e) None of these

Q.41. Pointing to a paragraph, Sachin said “She is the grandmother


of my father‟s sister‟s son”. How is the woman in the photograph
related to Sachin?
a) Mother
b) Aunt
c) Cousin
d) Cannot be determined
e) None of these

www.bankexamstoday.com Page 177


Q.42. P is the father of N. S is the mother of N who is the brother of
J. B is the son of S. C is the sister of B. How J is related to C?
a) Data inadequate
b) Cousin
c) Brother
d) Sister
e) None of these

Q.43.D is the brother of B. M is the brother of B. K is the father of M.


T is the wife of K. How is B related to T?
a) Son
b) Daughter
c) Son or Daughter
d) Data inadequate
e) None of these

Q.44.Pointing to a woman, Nirmal said, “she is the daughter of my


wife‟s grandfather‟s only child”. How is the woman related to
Nirmal ?
a) Wife
b) Sister-in-law
c) Sister
d) Data inadequate
e) None of these

Q.45.X told Y, “Though I am the son of your father, you are not my
brother”. How is X related to Y ?
a) Sister
b) Son
c) Daughter
d) Father
e) None of these

Q.46. Introducing Rajesh, Neha said, his brother‟s father is the only
son of my grandfather. How is Neha related to Rajesh ?
a) Daughter
b) Sister
c) Mother
d) Niece
e) None of these

Q.47. Pointing to a paragraph, Arun said, „she is the mother of my


brother‟s son wife‟s daughter. „How is Arun related to the lady ?

www.bankexamstoday.com Page 178


a) Uncle
b) Daughter-in-law
c) Cousin
d) Brother
e) None of these

Q.48. A boy goes to see a film and finds a man who is his relative.
The man is the husband of the sister of his mother. How Is the
man related to the boy ?
a) Brother
b) Nephew
c) Uncle
d) Father
e) None of these

Q.49. Lakshmi and Meena were Rohan‟s wives. Shalini is Meena‟s


step-daughter. How was Lakshmi related to Shalini ?
a) Sister
b) Mother-in-law
c) Mother
d) Step-daughter
e) None of these

Q.50. Daya has a brother, Anil. Daya is the son of Chandra. Bimal is
Chandra‟s father. In terms of relationship, what is Anil of Bimal?
a) Son
b) Grandson
c) Brother
d) Grandfather
e) None of these

Q.51. Anil, introducing a girl in a party, said, she is the wife of the
grandson of my mother. How is Anil related to the girl?
a) Father
b) Grandfather
c) Husband
d) Father-in-law
e) None of these

Q.52. A man said to a woman, “Your mother‟s husband‟s sister is my


aunt.” How is the woman related to the man ?
a) Granddaughter
b) Daughter
c) Sister

www.bankexamstoday.com Page 179


d) Aunt
e) None of these

Q.53.Introducing Ramesh, Neelam said, “His brother‟s father is the


only son of my grandfather”. How Neelam is related to Ramesh?
a. Sister
b. Daughter
c. Mother
d. Niece
e. None of these

Q.54. „X + Y‟ means „X is the father of Y‟. „X – Y‟ means „X is the wife


of Y‟. „X x Y‟ means „X is the brother of Y‟, „X † Y‟ means „X is the
mother of Y‟ and „X = Y‟ means „X is the sister of Y‟ On the basis of
this information, what does „X + Y – Z‟ mean?

Q.55. Pointing to a lady on the stage, Ashita said, “She is the sister
of the son of the wife of my husband.” How is the lady related to
Ashita?

Q.56. Miley says, “Dolly‟s father Mick is the only son of my father-in-
law Jagger.” Then how is Pamela, who is the sister of Dolly, related
to Jagger?

Q.57. While discussing Tan‟s ancestry, Lan quizzes Tan while


pointing towards a man, “His mother is the only child of your father.‟
How is Tan related to that person?

Q.58. Vicky who is Dinesh‟s daughter, says to Irma, “Your mother


Reeta is the younger sister of my father, who is the third child of
Chand.” How is Chand related to Irma?

Q.59. Anand is the brother of of Bakul and Chavi. Dina is Chavi‟s


mother. Ernie is Anand‟s father. Is Bakul Ernie‟s son?

Q.60. A woman presents a man as the son of the brother of her


mother. How is the man related to the woman?

Q.61. Raina, Murli, Haiden, Patel, Gony, Balaji and Morkel are seven
members in a family, out of which there are three females and four
males. There are two managers, two lawyers, one teacher, one
engineer and one doctor. No lady is either a teacher or an engineer.
Haiden is a lawyer and is married to Raina, who is a teacher. Balaji,

www.bankexamstoday.com Page 180


the engineer is married to Patel, who is neither a lawyer, nor a
doctor. No two ladies have the same profession. Murli is the sister of
Morkel, who is a manager.

(i) What is Gony‘s profession?


(ii) Who are the four males?
(iii) Who is the teacher?
(iv) Who is married to Balaji and what is the person‘s profession?

Answers

1. Ans. C.
Solution:

2. Ans. E.
Solution:

www.bankexamstoday.com Page 181


3. Ans. E.
Solution:

4. Ans. D.
Solution:

www.bankexamstoday.com Page 182


5. Ans. C.
Solution:

6. Ans. C.
Solution:

7. Ans. A.
Solution:

www.bankexamstoday.com Page 183


8. Ans. B.
Solution:

9.Ans. C.
Solution:

www.bankexamstoday.com Page 184


10. Ans. A.
Solution:

11. Ans. C. Aunt

12. Ans. D. S is the daughter in law of M.

13. Ans. A 3

14. Ans. E Brother-in-law

15. Ans. E 4

Solution:

16. Ans.E None of the above

www.bankexamstoday.com Page 185


17. Ans.C Son

18. Ans.C His Daughter

19. Ans.B His son's

20. Ans.A Sister

21. Ans.C Father

22. Ans.A Father

23. Ans.D Niece

24.Ans. B

25.Ans. D

26.Ans. E

27.Ans. A

28.Ans. D

www.bankexamstoday.com Page 186


29.Ans. A

30.Ans. B

31.Ans. C

32.Ans. D

33.Ans. B

34. Ans.C

35. Ans.D

36. Ans.E

37. Ans.C

38. Ans.C

39. Ans.B

40. Ans.B

41. Ans.E

42. Ans.A

43. Ans.C

44. Ans.D

45. Ans.E

46. Ans.B

47. Ans.A

48. Ans.C

49. Ans.C

50. Ans.B

51. Ans.D

www.bankexamstoday.com Page 187


52. Ans.C

53. Ans.A

54. Ans. We first off need to clear the clutter with the + and – and x‘s!
Simply jot down on your rough sheet like this :-

+ father of
– wife of
X brother of
÷ mother of
= sister of

Now we solve,

X + Y – Z therefore is,

X is the father of Y, and Y is the wife of Z!

That means – X is the father in law of Z.


55. Ans. Find who you can easily relate to and be that person – then go
about creating one relation after another!

In this question – be Ashita – then start from the end of the sentence – that
way it will be easier.

‗My husband‘ = Ashita‘s husband.


‗wife of my husband‘ = is Me = Ashita!
‗son of the wife of my husband‘ = My son.
‗Sister of the son of the wife of my husband‘ = My son‘s sister = My
daughter!
‗She is the sister of the son of the wife of my husband‘ = the lady on the
stage = the lady being pointed out = my daughter!

Ashita + Husband
_________|__________
| |
Son Daughter

www.bankexamstoday.com Page 188


Now you solve! Time yourself and then look at the answers below!

56. Ans. Pamela is Jagger‘s Grand-daughter.

Jagger(m, father in law of Miley)


|
Miley(f, wife)+Mick/Husband/only son of FIL
|
Dolly – Pamela (f, sister)

57. Ans. Tan is that person‘s Mother.

Father ( first off – the father)


|
Tan (then - ‗mother is the only child‘)
|
Son (‗His‘)

58. Ans. Grandparent.


We can say Chand is Irma‘s Grandparent and not Grandfather or
Grandmother, because don‘t know!
Simply because Chand‘s gender cannot be identified from the question – we
NEVER identify gender from the names!

Chand
__________________ | _________
| |
Reeta(f,mother/sister of) ---------- Dinesh (m, father of)
| |
Irma Vicky(f)

59. Ans. NO!

www.bankexamstoday.com Page 189


Because we cannot determine Bakul‘s gender!

Ernie(m, father) + Dina(f, mother)


|
Anand(m, brother of) – Bakul – Chavi

60. Ans. The man is the woman‘s cousin brother – ‗Mama kaladka‘ if I may
say so!
Mother – Brother
| |
Woman Son

61. Ans. Most important thing is to go one sentence after another and
solving the clues without getting confused of the male names!
(i) What is Gony‘s profession? – Lawyer .
(ii) Who are the four males? – Raina, Balaji, Morkel, and Gony.
(iii) Who is the teacher? – Raina, male.
(iv) Who is married to Balaji and what is the person‘s profession? – Patel,
female, wife, Manager.

Females Males
1.Haiden + 1.Raina
2.Patel + 2.Balaji
3.Murli – 3.Morkel
4. Gony
2 managers – Patel(f), Morkel (third male)
2 Lawyers – Haiden(female), Gony ( the last/fourth male)
One teacher – only male - Raina
One doc – third female - Murli
One eng – only male – Balaji

Coding Decoding

Directions : Study the following information carefully and answer the


questions given below:

www.bankexamstoday.com Page 190


In a certain code language "all players are energetic" is written as
"zee dp zn nt".
"are they really energetic" is written as "mb nt re zn", "related to all players"
is written as "st zee dp ta" , discipline is related to game", is written as
"gt ta no st np" , 'game is really to enjoy is written as 'np re gt ta fr'.

Q.1. Which of the following is the code for 'enjoy'?


1. ta
2. st
3. gt
4. fr
5. no

Q.2. Which of the following does 'dp zn nt' stand for?


1. all are energetic
2. players are energetic
3. they are players
4. all are players
5. Cannot be determined

Q.3. Which of the following is the code for 'discipline'?


1. no
2. st
3. gt
4. re
5. mb

Q.4. If 'the discipline is enjoyed' is written as 'kp no np fr' then what


is the code for ' the game to enjoy'?
1. no fr ta kp
2. kp gt ta fr
3. ta fr no gt
4. mb st gt nt
5. kp gt tec fr

Q.5. Which of the following stand for 'zee'?


1. all
2. player

www.bankexamstoday.com Page 191


3. energetic
4. players
5. Either 1 or 4

Directions: In the following questions, a group of number/Symbol followed


by five combinations of inters codes is given. You have to find out which of
the combinations correctly represents the group of number/symbol based on
the given coding system and the conditions
and mark that combination as your answer.

Number/symbol 4 @ 5 ^ + 2 7 8 & # % ᵟ 3 $ 9

Letter Code H D A T U E M L P Z B K X G Y
Conditions
I. If the first and the last elements are symbols then their codes are to be
interchanged.
II. If a symbol is immediately followed as well as immediately preceded by a
number then that symbol is to be coded as ‗1‘
III. If the last element is an odd number then the second elements is to be
coded as the code of the odd number.
IV. If the third element is an even number then the code of that even
number is to be interchanged with the code of first element.
(Note All the elements have to be counted from left to right to fulfill the
conditions.)

Q.6. #@3+42
a. ZDKTHE
b. ZDXUHE
c. ZKE1DH
d.EDK1HZ
e. EDUXHZ

Q.7. 8^@537
a. LMDAXM
b. LTMAKM

www.bankexamstoday.com Page 192


c. DTLAKM
d. LMKDAM
e. MTDAXL

Q.8. @32&#8
a. EXDPZL
b. EZDPXL
c. XEDZ1L
d. XLEZPL
e. ZLEXPL

Q.9. @98&^ᵟ a. LYDPTK


b. DY1PTK
c. KYLPTD
d. DYL1TK
e. LPDYTK

Q.10. $452#%
a. GH1EZB
b. GHA1EZ
c. BHZEAG
d. BHAEZG
e. AHGEZB

Directions: Consider the following steps for given input and read the
instructions to reach the last step:

www.bankexamstoday.com Page 193


Directions: Input to Step I: Only alphabets are exchanged on the basis of
arrows given.

Step I to Step II:


(i) If both letters are consonant then replace the both letters with
succeeding alphabet and subtract 3 from the number.
(ii) If there are one vowel and one consonant in the value then replace the
both letters with preceding alphabet and subtract 2 from the number.
(iii) If there is only one letter in the value then add 4 in number.
Step II to Step III: It is also follows some pattern but is not same as of Step
I and II.

www.bankexamstoday.com Page 194


Q.11. What is the sum of all numbers in Step II of given input?
A. 55
B. 45
C. 50
D. 48
E. 65

Q.12. Which of the following represents the third element in 3rd row in Step
III?
A. M6
B. Z9
C. A5
D. Q5
E. R8

Q.13. In Step III how many alphabets are repeated?


A. 4
B. 2
C. 1
D. None
E. 3

Direction: Study the following information carefully and answer the


questions given below:- With a certain code language,

―The meeting separated dramatic‖ is written as ―!e48‖ ―@h08‖ ―#r63‖


―$e80‖

―Since India annual Hundreds‖ is written as ―#u63‖ ―-n35‖ ―%i24‖ ―%n24‖

―Scientists that discovered bacteria‖ is written as ―*h15‖ ―#a63‖ ―&c99‖


―&i99‖

www.bankexamstoday.com Page 195


―Immense pressure compared fine‖ is written as ―#o63‖ ―*i15‖ ―#r63‖
―!m48‖

Q.14. What is the code for ‗species?


A. #p49
B. #n63
C. !p49
D. %n24
E. None of these

Q.15. What is the code for ‗Since meeting pressure?


A.―*a24 !e48 *i15‖
B. ―%i24 @h48 *i15‖
C.―%i24 !e48 *#c63‖
D. ―%i24 !e48 #r63‖
E. None of these

Q.16. What does ―!e48 @i99 #n63 @h08‖ stand for?


A. sine India have fine
B. The meeting discovered internet
C. meeting gained the internet
D. The meeting have
E. Cannot be determined

Q.17. What could be the code for ‗ancient pollution?


A. !n48 $o80
B. %i24 *a15
C. !e48 #n63
D. #c63 –a35
E. None of these

Q.18. What is the code for ‗bacteria?


A. #c63
B. *a63
C. @h08
D. $a63

www.bankexamstoday.com Page 196


E. None of these

Direction: Study the following information carefully and answer the


questions given below:-With a certain code language-

" While many celebrated day " is written as "%D14, &K09, #D25 , !H12.

" This institution has essential " is written as "$D19 , *Q19, %V19, @K09 .

" From now massive consumers " is written as "#R14, !D19, &U15, %R13 .

" The good qualities compared " is written as " *R13 , $X01, #R15, !K05.

Q.19. What is the code for ‗massive‘ in the given code language?
A. %E19
B. &U15
C. #D14
D. *E19
E. !D19

Q.20. What is the code for ‗Has compared‘ in the given code language?
A. $D19, *R13
B. &K05, #R15
C. %E19, *R13
D. !D19, !Q14
E.#S16, $X01

Q.21. What may be the possible word for ‗#D25, *R13, #R14‘ in the given
code language?
A. report compared good
B. consumers from report
C. compared consumers day
D. day good consumers
E.None of these

Q.22. What may be the possible code for ‗massive ancient in the given code
language?
A. %R03, #O12
B. *D19, %Q03

www.bankexamstoday.com Page 197


C. @K19, *N09
D. $D04, @K03
E.%R21, $J21

Q.23. What is the code for ‗from many‘ in the given code language?
A. #R15, %R18
B. #S16, %E14
C. !Q14, &U15
D. &U15, %D14
E.&K15, !H14

Description: In a certain code:-

―Paper milk Refuse Involve aboard‖ is coded as - ―@7E #4K @6D #6E #5R‖

―Question Laptop Keyboard Eye‖ is coded as - ―@3E #8D @8N #6P‖

―Reverse Apple Rose Biology‖ is coded as - ―@5E #4E #7Y #7E‖

Q.24. What will be the code for “Compulsory”?


A. @10Y
B. #9C
C. #10Y
D. #10P
E. None of these

Q.25. What must be the code for “Heroin”?


A. #5O
B. @6H
C. #5N
D. #6N
E. None of these

Q.26. Which of the following can be coded as “@6A”?


A. Idea
B. Arrow
C. Seminar
D. Common
E. Agenda

www.bankexamstoday.com Page 198


Q.27. If all the vowels of the word “University” are removed, then
what would be the code for the resulting word?
A. #10Y
B. @9Y
C. @10Y
D. #9Y
E. None of these

Q.28. What is the code of “aboard” in above-coded information?


A. #7D
B. #6D
C. None of these
D. @6D
E. @5

Description: In a certain code:-


―Permanent Refute Invoke Refuse‖ is coded as - ―24RUN 15FIS 15FIT
15VYK‖
―Free Vague member Distribution‖ is coded as - ―12GJU 15MPE 9EHE
36SVO‖
―Immediate between Coalesce Bigotry‖ is coded as - ―18TWE 18GJR 24MPT
21ADC‖

Q.29. What will be the code for “Bear not”?


A. 9ADA 6TWO
B. 7TWO 8SAD
C. 5WOT 6DAS
D. 8TOD 16ADA
E. None of these

Q.30. What must be the code for “notice”?


A. 21CIT
B. 15TOE
C. 18TCE

www.bankexamstoday.com Page 199


D. 15TWC
E. None of these

Q.31. Which of the following can be coded as “18OSS”?


A. Swallow
B. Poacher
C. Stealth
D. Amongst
E. None of these

Q.32. If all the vowels of the word “Immediate” are removed, then
what would be the code for the resulting word?
A. 24EHT
B. 15ADN
C. 21EHT
D. 15ILT
E. None of these

Q.33. What is the code of “Coalesce”?


A. 21ADC
B. 20GJR
C. 12GJS
D. 21AEC
E. None of these

Directions : In a certain code language


"Love, Cry, Hate, Happy" is written as "V%19, X&2, S&2, V%15"

"Wire, Charger, Mobile, Eye" is written as "V%14, X&9, V%4, V&22"

"Jug, Soap, State, Power" is written as "K%8, K&9, Q&20, H&22"

"Country, India, Best, Healthy" is written as "G%25, R&26, S&2, X&2"

Q.34. The code for the word Having is?

A. 19&T
B. T&19
C. 19%T

www.bankexamstoday.com Page 200


D. T%19
E. None of these

Q.35. What will be the code for 'Gone'?


A. T&19
B. V%20
C. U%18
D. V&19
E. None of these

Q.36. The code 'X&7' denotes which of the following word?


A. Court
B. Judge
C. File
D. Shirt
E. None of these

Q.37. What is the code for 'Just For Challenge'?


A. G&17, U%9, X%2
B. G%17, U&9, X&22
C. G&16, V&9, X%22
D. F%17, U&8, X%21
E. None of these

Q.38. What is the code for 'Spirit'?


A. G&8
B. F&8
C. G%8
D. F&7
E. None of these

Study the following information carefully and answer the given


questions:

In a certain code language ―bought a big house‖ is written as ―dn co he rh‖,


―Charity starts at house‖ is written as ―rh na ek sa‖, ―Bought big charity
shop‖ is written as ―na dn he ku‖, and ―a shop at station‖ is written as ―co ek
ku ze‖ (All codes are two letters only)

www.bankexamstoday.com Page 201


Q.39. What does the code „ze‟ stand for in the given code language?
1) big
2) station
3) bought
4) shop
5) starts

Q.40. What does the code for 'shop' in the given code language?
1) rh
2) ek
3) ku
4) ze
5) dn

Q.41. What does the code 'ek' stand for in the given language code?
1) shop
2) at
3) charity
4) either 'bought' or 'big'
5) a

Q.42. In the given code language, which of the following possibly


means 'bought big food'?
1) he dn ku
2) rh dn he
3) dn bk he
4) ku ek na
5) na rh ek

Q.43. What is the code for 'charity' in the given code language?
1) na
2) ku
3) he
4) rh
5) dn

www.bankexamstoday.com Page 202


Study the following information carefully and answer the given
questions:
In a certain code language: ―how do it name‖, is written as ―ku ru mu ju‖,
―name do the fake‖ is written as ―pu ku su ru‖, ―the Milton has how‖ is
written as ―su mu ho ro‖, and ―do has and not‖ is written as ―kho rob u ru‖.

Q.44. Which of the following is the code of "not"?


1) kho
2) Ro
3) Either kho or bu
4) bu
5) ru

Q.45. The code 'pu ho ju ku' may represent?


1) your name not how
2) how has fake name
3) fake milton it name
4) it not fake the
5) how the has milton

Q.46. Which of the following may represent "milton has had how
name "?
1) Ro mu ho wo ku
2) ku mu ho ro ru
3) ho ku mu kho bu
4) bu ju ru Ro ku
5) mu ku su pu ru

Q.47. Which of the following is the code of 'milton'?


1) su
2) ro
3) mu
4) ho
5) None of these

Q.48. Which of the following is code of 'it fake and not'?


1) ru bu ju ro
2) ju bu pu kho

www.bankexamstoday.com Page 203


3) ku kho pu ju
4) su pu kho bu
5) can't be determined

Coding decoding

all players are energetic →zee dp zn nt .............. (i)


are they really energetic → mb nt re zn .............. (ii)
related to all players → st zee dp ta......................(iii)
discipline is related to game → gt ta no st np ......(iv)
game is really to enjoy → np re gt ta fr ................(v)
From (iii) and (v), to → ta ..............(vi)
From (iii), (iv) and (vi), related → st..................(vii)
From (ii) and (v), really → re ................... (viii)
From (i), (ii) and (viii), they → mb ................ (ix)
From (i), (ii), (viii) and (ix),
are energetic → nt or zn/zn or nt ................(x)
From (i) and (iii), all players → zee or dp/dp or zee ..(xi)
From (iv), (v) and (vi), is game → gt or np/np or gt... (xii)
From (v), (vi), (viii) and (xii), enjoy → fr .................(xiii)
From (iv), (vi), (vii) and (xii), discipline → no ..........(xiv)

Ans.1. fr

Ans.2. Cannot be determined

Ans.3. no

Ans.4. kp gt ta fr

Ans.5. Either 1 or 4

Ans.6. Option B

Ans.7. Option A

Ans.8. Option A

www.bankexamstoday.com Page 204


Ans.9. Option A

Ans.10. Option D

Ans.11. D
Solution:
=6+6+5+6+8+5+3+9
=48

Ans.12. E
Solution:
The third element in 3rd row in Step III = R8

Ans.13. C

Solution:
Only alphabet ‗W‘ is repeated in Step III.

Ans.14. C

Ans.15. D

Ans.16. B

www.bankexamstoday.com Page 205


Ans.17. A

Ans.18. E

Solution(19-23):

The pattern for this coding is-


 Each symbol is denoted by number of words like-

3=@ 5=% 7=! 9=$

4=* 6=- 8=# 10 = &


 Each alphabet is a 2nd letter of the each word like
1. ―For meeting – e
2. For Scientists – c‖
 Each number is stands for total number of words square
1. ―bacteria – number of words = 8 = 8² – 1 = 64 - 1 = 63
2. fine - number of words = 4 = 4² – 1 = 16 – 1 =15‖

Ans.19.E

Ans.20.A

Ans.21.C

Ans.22.B

Ans.23.D

Solution(24-28) :

The pattern for all the above code is:


The letter denotes second letter of each word + 3
Like ―qualities‖ – Second letter ―U‖ + 3 = X
The number denotes the alphabet number of 3rd letter (1-26)
Like ―institution‖ – ―S‖- 19
And rest of the one symbol and can be anything.

www.bankexamstoday.com Page 206


Ans.24. C. #10Y

Ans.25. D. #6N

Ans.26. E. Agenda

Ans.27. D. #9Y

Ans.28. D. @6D

Detailed Explanation:-
(i) First element of code is based on vowel starting word. If any word started
with Vowel
Alphabet then symbol is used @, and for consonant starting word # is used.
(ii) Second code is total number of letters in a particular word.
(iii) Third code is last alphabet of a particular word.
Ans.29. A. 9ADA 6TWO

Ans.30. D. 15TWC

Ans.31. None of these

Ans.32. C. 21EHT

Ans.33. A. 21ADC

Detailed Explanation:
(i) First element of code is thrice of total number of letters in a word - 3
(ii) 2nd element is a 3rd letter of a that particular word from starting
(iii) 3rd element of a code is 3rd letter of that particular word from starting
+3 in alphabetical series.
(iv) 4th element is 2nd last alphabet of that word.

Ans.34. D
If total numbers of word is even then we use:
Letter= Opposite of last alphabet
Symbol= %
Number= Opposite of first alphabet

www.bankexamstoday.com Page 207


If total numbers of word is odd then we use:
Letter= Opposite of the first alphabet
Symbol= &
Number= Opposite of the last alphabet
Having (even)- T%19

Ans.35. B
If total numbers of word is even then we use:
Letter= Opposite of last alphabet
Symbol= %
Number= Opposite of first alphabet
If total numbers of word is odd then we use:
Letter= Opposite of the first alphabet
Symbol= &
Number= Opposite of the last alphabet
Gone (even)- V%20

Ans.36.A
If total numbers of word is even then we use:
Letter= Opposite of last alphabet
Symbol= %
Number= Opposite of first alphabet
If total numbers of word is odd then we use:
Letter= Opposite of the first alphabet
Symbol= &
Number= Opposite of the last alphabet
X&7- Court (odd)

Ans.37. B
If total numbers of word is even then we use:
Letter= Opposite of last alphabet
Symbol= %
Number= Opposite of first alphabet
If total numbers of word is odd then we use:
Letter= Opposite of the first alphabet
Symbol= &
Number= Opposite of the last alphabet
Just (even) For(odd) Challenge(odd)- G%17, U&9, X&22

www.bankexamstoday.com Page 208


Ans.38.C
If total numbers of word is even then we use:
Letter= Opposite of last alphabet
Symbol= %
Number= Opposite of first alphabet
If total numbers of word is odd then we use:
Letter= Opposite of the first alphabet
Symbol= &
Number= Opposite of the last alphabet
Spirit (even)- G%8

Ans.39. 2

Ans.40. 3

Ans.41. 2

Ans.42. 3

Ans.43. 1

Ans.44. 3

Ans.45. 3

Ans.46. 1

Ans.47. 4

Ans.48. 2

Inequalities

www.bankexamstoday.com Page 209


In each of these questions, two equations (I) and (II) are given. You have to
solve both the equations and given answer.

Q.1.

Q.2.

Q.3.

Q.4.

Q.5.

A. If x < y

B.If x > y

www.bankexamstoday.com Page 210


C. If x = y

D. If x ≥ y

E. If x ≤ y or no relationship can be established between x and y

Q.6.

Q.7.

Q.8.

Q.9.

Q.10.

www.bankexamstoday.com Page 211


A. If x ≥ y

B. If x > y

C. If x < y

D. If x ≤ y

E. If x = y

(Directions 11-13) symbols represents certain time in a clock as:-


@ - Either the hour or minute hand of clock on 2
# - Either the hour or minute hand of clock on 8
$ - Either the hour or minute hand of clock on 10
% - Either the hour or minute hand of clock on 4
& - Either the hour or minute hand of clock on 6
© - Either the hour or minute hand of clock on 5

Time @# represents 2 hour 40minutes. All the time are in P.M.


First symbol represents the hours and second symbol represents the
minutes.

Q. 11.If a train departed from a station at #% and it takes 2 hr 10


minute to reach the Delhi then when it will reach to the Delhi?

www.bankexamstoday.com Page 212


A. %#
B. #$
C. $&
D. $%
E. %#

Q. 12.If Anmol takes 50 min to reach the bus station and his bus is
scheduled at ©% and he wants to reach the station 5 mint earlier
then at what time should he leaves to reach the station?
A. #$
B. %&
C. #%
D. %©
E. $©

Q. 13.If the speed of the train is 60 km/h. Find the time taken by the
train to cover a distance of 250 km.
A. %@

B. #$
C. %$
D. @#
E. @$

Q. 14.

W≥D<M<P<A=F

Conclusions:

I. F > D
II. P < W

A. Only Conclusion I is true.

B. Only Conclusion II is true.

C. Either Conclusion I or II is true

D. Neither conclusion I nor II is true.

www.bankexamstoday.com Page 213


E. Both conclusions I and II are true.

Q. 15.

H ≥ M > F <A = B> S

Conclusion:

I. H > B
II. F < S

A. Only Conclusion I is true.

B. Only Conclusion II is true.

C. Either Conclusion I or II is true

D. Neither conclusion I nor II is true.

E. Both conclusions I and II are true.

Q. 16.

B>T>Q>R=F

Conclusion:

I. Q ≥ F
II. T> F

A. Only Conclusion I is true.

B. Only Conclusion II is true.

C. Either Conclusion I or II is true

D. Neither conclusion I nor II is true.

E. Both conclusions I and II are true.

Q. 17. H < J, F < H, I ≤ J = K

Conclusion:

www.bankexamstoday.com Page 214


I. H > I
II. I ≥ F

A. Only Conclusion I is true.

B. Only Conclusion II is true.

C. Either Conclusion I or II is true

D. Neither conclusion I nor II is true.

E. Both conclusions I and II are true.

Direction: Read the information given below and solve the questions that
follow.

% means not greater than (% ➔ ≤)


& means not smaller than (& ➔ ≥)
# means neither equal to nor smaller than (# ➔ >)
$ means neither equal to nor greater than ($ ➔ <)
@ means neither smaller than nor greater than (@ ➔ =)
From the above statements, we can conclude,

Q. 18.

Statements

M $ K,K & T,
T$J

Conclusions:

I. J # K
II. T # M

III. M # J

A. None of the conclusion is true

B. Only Conclusion I is true

www.bankexamstoday.com Page 215


C. Either Conclusion I and ll is true

D. Only conclusion lll is true

E. All of them is true

Q. 19.

Statements:

F@T
T%M
M#R

Conclusion:

R$T
F@M
F$M

A. None is true

B. Only Conclusion II is true.

C. Only conclusion lll is true

D. Neither conclusion I nor II is true.

E. Both conclusions Il and lll is true.

Q. 20.

Statements:

J & H,
H @ B,
B%N

Conclusion:

I. N & H
II. N @ J
III. J & B

www.bankexamstoday.com Page 216


A. Only Conclusion I is true

B. Only Conclusion IIl is true

C. Both l and lll is true

D. Neither conclusion I nor II is true.

E. All of them is true

Directions: Following questions are based on given information below and


you‘ve to tell which conclusion(s) follow:

Q. 21.

Statements:

P< A ≤ C ≥ K; C ≤ O < L; R ≤ N ≤ C

Conclusions:

I. A = K

II. R < L

a) Only Conclusion I is true.

b) Only Conclusion II is true.

c) Either Conclusion I or II is true

d) Neither conclusion I nor II is true.

e) Both conclusions I and II are true.

Q. 22.

Statements:

S ≤ T ≤ K ≤ E; N ≥ E ≤ P

Conclusions:

I.S ≤ N

www.bankexamstoday.com Page 217


II. S ≤ P

a) Only Conclusion I is true.

b) Only Conclusion II is true.

c) Either Conclusion I or II is true.

d) Neither conclusion I nor II is true.

e) Both conclusions I and II are true.

Q. 23.

Statements:

S > O ≥ U ≥ P; O ≤ T < R; T ≥ E > A

Conclusions:

I. R < A

II. U = A

a) Only Conclusion I is true.

b) Only Conclusion II is true.

c) Either Conclusion I or II is true.

d) Neither conclusion I nor II is true.

e) Both conclusions I and II are true.

Q. 24.

Statements:

P< A ≤ C ≥ K; C ≤ O < L; R ≤ N ≤ C

Conclusions:

I. O ≥ R

www.bankexamstoday.com Page 218


II. P ≤ O

a) Only Conclusion I is true.

b) Only Conclusion II is true.

c) Either Conclusion I or II is true.

d) Neither conclusion I nor II is true.

e) Both conclusions I and II are true.

Q. 25.

Statements:

S > O ≥ U ≥ P; O ≤ T < R; T ≥ E > A

Conclusions:

I.T > P

II. T = P

a) Only Conclusion I is true.

b) Only Conclusion II is true.

c) Either Conclusion I or II is true.

d) Neither conclusion I nor II is true.

e) Both conclusions I and II are true.

Directions: In the following questions, the symbols £, $, @, ≈, and > are


used with the following meanings:

P £ Q means P is greater than Q.

P $ Q means P is either greater than or equal to Q.

P @ Q means P is equal to Q.

P ≈ Q means P is smaller than Q.

www.bankexamstoday.com Page 219


P > Q means P is either smaller than or equal to Q.

Now in each of the following questions, assuming the given statementtobe


true, find which of the two conclusions I and II given below them is/are
definitely true. Give answer.

A. if only conclusion I is true;

B. if only conclusion II is true;

C. if either I or II is true;

D. if neither I NOR II is true; and

E. if both I and II are true.

Q. 26. Statements:

J>R, P@R>L, H$R

Conclusions:

I. R£L

II. J>H

Q. 27.

Statements:

G$F, D>S, S@F

Conclusions:

I. S≈G

II.S@G

Q. 28.

Statements:

www.bankexamstoday.com Page 220


Y>H, P≈N, H@P

Conclusions:

I. N£Y

II.H>N

Q. 29.Statements:

M£N, N>Q, Q>D

Conclusions:

I. M£D

II.Q@D

Q. 30.

Statements:

V£X, U>Z, X@U

Conclusions:

I. V£Z

II. X@Z

Answers

Ans. 1. Option A

Ans. 2. Option D

www.bankexamstoday.com Page 221


Ans. 3. Option E

Ans. 4. Option B

Ans. 5. Option D

Ans. 6. Option c

Ans. 7. Option E

Ans. 8. Option B

Ans. 9. Option C

Ans. 10. Option D

Ans. 11.. Option C

Solution: If the hour indicator of the clock is at 4 then it shows 4 hour and
if the minute indicator of the clock at 4 it shows 20 minute.
#%- 8:20 P.M.
8:20 P.M.+ 2 hr 10 minute = 10 hr 30 minute
= $&

Ans. 12. Option D

Solution: ©% = 5:20 P.M.


5:20 P.M. – (50 minute + 5 minute) = 4:25 P.M.
= %©

Ans. 13. Option A

www.bankexamstoday.com Page 222


Solution: Time= Distance / Speed
Time = 250/60
Time = 4 hr 10 minute
= %@

Ans. 14. Option A

Ans. 15. Option D

Ans. 16. Option B

Ans. 17. Option D

Ans. 18. Option A


Solution:
Convert the statement and conclusion from symbols to mathematical
operation.
Statement: M < K ≥ T < J
Conclusion:
I. J > K
II. T > M
III. M > J
To solve Conclusion I
J>K, consider statement from J to K
K≥T<J
Since it is the mixer of both < and > symbol
Conclusion 1 is false

To solve Conclusion II
T > M, consider statement from T to M
M<K≥T
Since it is the mixer of both < and > symbol
Conclusion 2 is false

To solve Conclusion III


M > J, consider statement from M to J
M<K≥T<J
Since it is the mixer of both < and > symbol
Conclusion 3 is false
Therefore none of the three conclusions is true

www.bankexamstoday.com Page 223


Ans. 19. Option E

Solution:
Convert the statement and conclusion from symbols to mathematical
operation.
Statement: F = T ≤ M > R
Conclusion:
I. R < T
II. F = M
III. F < M

To solve Conclusion I
R < T, consider statement from R to T
T≤M>R
Since it is the mixer of both < and > symbol
Conclusion 1 is false

To solve Conclusion 2
F = M, consider statement from F to M
F=T≤M
From the above statement, It is possible that F can be equal to M
Conclusion II may be True

To solve Conclusion 3
F < M, consider statement from F to M
F=T≤M
From the above statement, it is possible that F can be lesser than M
Conclusion III may be True
Therefore Either Conclusion 2 or 3 can be true

Ans. 20. Option C

Solution:
To solve Conclusion I
N ≥ H, consider statement from H to N
H=B≤N
It clearly implies N ≥ H
Conclusion I is True
To solve Conclusion II
N=J, consider statement from N to J
J≥H=B≤N
Since it is combination of <and > symbol we can‘t predict N=J

www.bankexamstoday.com Page 224


Conclusion II is False
To solve Conclusion III
J ≥ B, consider statement from J to B
J≥H=B
It clearly implies J ≥ B

Conclusion III is True


Therefore Conclusion I and III are True

Ans. 21. b) Only Conclusion II is true.

Ans. 22. e) Both conclusions I and II are true.

Ans. 23. d) Neither conclusion I nor II is true.

Ans. 24. a) Only Conclusion I is true.

Ans. 25. c) Either Conclusion I or II is true.

Ans. 26. Option B

Ans. 27. Option C

Ans. 28. Option A

Ans. 29. Option D

Ans. 30. Option D

Direction Sense

www.bankexamstoday.com Page 225


Directions: There are 8 people standing randomly. B is 4m to the east of A.
C is 5m to the north of B. Distance between C and D is 10m. D is to the east
of C. E is 10 m to the South of D. H is 5m to the west of G. E is 6m to the
east of F.F is 1m to the south of G.

Q.1
What is the shortest distance between A and H?
A. 6m
B. 4m
C. 5m
D. 3m
E. None of these

Q.2 In which direction is H with respect to E?


A. West-North
B. East-North
C. East-South
D. West-South
E. None of these

Q.3 What is the longest distance between C and F?


A. 20
B. 24
C. 26
D. 30
E. None of these

Q.4 A person starts walking in east direction and takes turn of 45


degree clockwise and then a 90 degree shift in anti-clockwise
direction and finally a 135 degree shift clockwise direction. Now in
which direction he is facing?
A. East
B. South-East
C. South
D. North
E. North-West

www.bankexamstoday.com Page 226


Q.5
V walked 20 meters towards south, took a left turn and walked 30
meters, again he took a left turn and walked 20 meters. How far is
he from the starting point?
A. 20
B. 30
C. 50
D. 40
E. None of these

Direction: These questions are designed to test candidate‘s ability to sense


direction. Questions on direction are comparatively simpler than other
questions of reasoning if the aspirant possesses the right knowledge of the
main four directions and four cardinal directions. Confusion is created in the
question by the paper setter by giving frequent right and left turns to a
specific direction.
There are four main directions - East, West, North and South as shown
below

There are four cardinal directions - North-East (N-E), North-West (N-W),


South-East (S-E), and South-West (S-W) as shown below:

www.bankexamstoday.com Page 227


Important Points:
At the time of sunrise if a man stands facing the east, his shadow will be
towards west.
At the time of sunset the shadow of an object is always in the east.
If a man stands facing the North, at the time of sunrise his shadow will be
towards his left and at the time of sunset it will be towards his right.
At 12:00 noon, the rays of the sun are vertically downward hence there will
be no shadow.
A person facing towards North, on taking left turn will face towards West and
on taking the right turn towards East.
Left turn means anticlockwise rotation and right turn means clockwise
rotation. Thus a person facing towards South, on taking left turn will face'
towards East and on taking right turn towards West.
A person facing towards East, on taking left turn will face towards North and
on taking right turn towards South.
A person facing towards West, on taking left turn will face towards South
and on taking right turn towards North.

www.bankexamstoday.com Page 228


A person facing towards North-West, on taking left turn will face towards
South-West and on taking right turn will face towards North-East.
A person facing towards South-West on taking left turn will face towards
South-East and on taking right turn towards North-West.
A person facing towards South-East, on taking left turn will face towards
North-East and on taking right turn towards South-West.
A person facing towards North-East, on taking left turn will face towards
North-West and on taking right turn towards South-East.
In order to determine the distance travelled or the shortest straight distance
between the two given points, the Pythagoras formula

Q.6: One evening, two friends riya and priya were talking to each other,
with their backs towards each other, sitting in a park. If riya‘s shadow was
exactly to the left of her, then which direction was priya facing?

Q.7: One morning after sunrise, Vikram and Shailesh were standing in a
lawn with their backs towards each other. Vikram‘s shadow fell exactly
towards left hand side. Which direction was Shailesh facing?

Q.8: K is 40 m South-West of L. If M is 40 m South-East of L, then M is in


which direction of K?

Q.9: A man starting from his home moves 4 km towards East, then he turns
right and moves 3 km. Now what will be the minimum distance covered by
him to come back to his home?

Directions: questions asked in the exam are based on two principles


Distance
Direction
You might be asked to calculate the distance or the direction. The foremost
thing that you are supposed to do is draw basic direction map.

www.bankexamstoday.com Page 229


For example if you are asked to calculate the distance
If I move 4 km towards east and then I travel 3 km towards north. What is
my distance from the starting point?
Remember your starting point is always the centre of the direction
map.

Q.10: If I move 4 KM towards east and then I travel 3 Km towards north. In


which direction I am from the starting point?

Q.11: A person starts from position A. He goes towards the east and then
takes a right turn .Again the person takes a left turn and moves and then
again he move 45 degree right. In which direction he is from the starting
position?

Q.12: A person P is standing at a point , another person Q is standing in the


north of P. Q‘s friend R is in right of Q and P‘s friend S is standing left of P.In
which direction R is standing with respect to S.

Q.13: In which direction is S with respect to R ?

Q.14: I start from my house and go straight and come at a cross road. The
road which is coming from opposite direction ends at a hospital. When I walk
from my house and turn right , there is a park on that road and opposite to

www.bankexamstoday.com Page 230


park there is a school.What would be the direction of my house from the
school.

Q.15: What would be direction of school from my house ?


SUNRISE and SUNSET
Sun rises in the EAST then it shadow will be in the WEST.
Sun sets in the WEST then it shadow will be in the EAST.

Q.16: A man is facing towards the NORTH. At sunrise where will his shadow
be? And if he is facing SOUTH. Then where will his shadow be?

Answers

1. Ans: C

2. Ans: A

www.bankexamstoday.com Page 231


3. Ans: C

4. Ans: C

5. Ans: B

www.bankexamstoday.com Page 232


6. Ans:
Riya‘s shadow fell to her left i.e. towards East (as it was evening). So, Riya was
facing ―South‖. As Priya had her back towards Riya, hence, Priya was facing
"North". North is the correct answer.

7. Ans:
Since Vikram‘s shadow fell towards left, therefore, Vikram is facing ―North‖. So,
Shailesh standing with his back towards Vikram will be facing ―South‖. South is
the correct answer.

8. Ans:
M is in the East of K as shown in the following diagram. East is the correct
answer.

9. Ans:
Here the Pythagoras formula is to be adopted.

10. Ans:
First of all, draw the map according to the directions

www.bankexamstoday.com Page 233


It is a right angled triangle so by Pythagorean theorem
H2 = P2 +B2
= 3 2 + 42
= 9 + 16
= 25
H=5
So I am at a distance of 5 KM from the starting point.
Similarly you can be asked to find the direction

11. Ans:
Draw the map and try to superpose the direction map like this

Now it is quite clear from the map that I am in NE direction from the starting point.
This question can also be asked in a different way like :

www.bankexamstoday.com Page 234


In which direction I am from my final position ?,
Now scenario will change and the final position would act as the centre of the map.

It is clear from the map I am in SW direction from my starting position.

There are a few thing that you need to keep in mind while solving these questions
I. Direction : N S E W
II. Left –Right
III. Straight / opposite
IV. Sunrise and sunset

12. Ans:
With respect to S , R is in North East direction

13. Ans:

It is clear from the picture with respect to R , S is SW direction.

www.bankexamstoday.com Page 235


It is clear that with respect to R, S is in the NE direction.

14. Ans:
If it is straight , we can assume any direction.

Here the assumed direction is EAST.

My house is in the SW from the school.

15. Ans:

Clearly school is NE direction from the school.

www.bankexamstoday.com Page 236


16. Ans:

If he is facing NORTH, his shadow will be in the WEST or in the left of the person.

And If he is facing South , Then also his shadow will be in THE WEST or in the RIGHT
of the person

Problem Based on Ages

Ques 1.
The sum of present ages of Ashok, Anil and Ajay is 75 years.If 5 years ago
the ratio of their ages was 3 : 4 : 5, What is the present age of Anil?

Ques 2.
The ratio of ages of Ajay and vijay at present is 3 :2 and 8 years earlier, the
ratio was 5 : 3. What are their present ages?

Ques 3.
One year ago the ratio between samir and Saurabh's age was 4 : 3 One year
hence, the ratio of their ages will be 5 : 4 .What is the sum of their present
ages in years ?

www.bankexamstoday.com Page 237


Ques 4.
The average age of a man and his son is 48 years. The ratio of their age is 5
: 3 respectively .What is the son's age ?

Ques 5.
The ratio of ages of Namrata and Divya is 4 : 3 .The sum of their ages is 28
years.The ratio of their ages after 4 years will be :
(a) 3: 4
(b) 5 : 4
(c) 5 : 6
(d) 6 :5

Ques 6.
Ten years ago, the age of Divya was half of the age os namrata .If the ratio
of present ages of both is 3 :4 the sum of their present ages is :
(a) 35 years
(b) 30 years
(c) 25 years
(d) 18 years

Ques 7.
The ratio of ages of a man and his wife is 4:3 .After 4 years, this ratio will be
9:7 . If at time of marriage, the ratio was 5 :3, then how many years ago
were they married ?
(a) 15 years
(b) 12 years
(c) 10 years
(d) 8 years

Ques 8.
Sudha's grandfather was 8 times older to her 16 years ago.He would be 3
times of her age 8 years from now. Eight years ago, What was the ratio of
Sudha's age to that to her grandfather?
(a)11:53
(b) 3: 8
(c) 1 : 5
(d) 1: 2

www.bankexamstoday.com Page 238


Ques 9.
The sum of ages of A and B is 12 years more than the sum of ages B and
C.Then C is how many years younger than A ?
(a) 10
(b) 12
(c) 24
(d) Data inadequate

Ques 10.
The sum of the ages of 5 children born at the interval of 3 years each is 50
years.the age of youngest child is :
(a) 10 years
(b) 8 years
(c) 4 years
(d) None of these

Ques 11.
A father said to his son,"I was as old as you are at present at the time of
your birth." If the father's age is 38 years now. Five years ago the age of
son was:
(a) 38 years
(b) 33 years
(c) 19 years
(d) 14 years

Ques 12.
Nisha's father was 38 years of age when she was born while her mother was
36 years old when her brother four years younger to her was born. The
difference of ages of her parents is :
(a) 8 years
(b) 6 years
(c) 4 years
(d) 2 years

Answers

www.bankexamstoday.com Page 239


Ans 1. Let 5 years age, the age of ashok, Anil and Ajay were 3x, 4x and 5x
years. Then, their present ages are ( 3x + 5), (4x + 5) abd ( 5x + 5) years.
Now, 3x +5 + 4x +5 + 5x + 5 = 75 => 12x = 60 x=5
Hence, present age of Anil = 4x + 5 = 4 x 5 + 5 = 25 years.

Ans 2. Let the present ages of Ajay and Vijay are 3x years and 2x years
respectively ; then

Ans 3. Let one year ago ages of Samir and Saurabh was 4x and 3x years
respectively; then

Ans 4. Sum of ages of man and his son = 2 x 48 = 96


Let the ages of man and his son be 5x and 3x years; then
5x + 3x = 96 => x = 12
Hence, son's age = 3x = 3 x12 = 36 years.

Ans 5. Let the present ages of Namrata and Divya are 4x and 3x
years respectively;then 4x + 3x = 28

www.bankexamstoday.com Page 240


Hence, their present ages are 16 and 12 years.
so, required ratio = ( 16 + 4) : ( 12 + 4) = 20 : 16 = 5 : 4

Ans 6. Let the present ages of Divya and Namrata are 3x and 4x years
respectively ; then

Ans 7. Let the present ages of the man and his wife are 4x and 3x years
respectively ; then

Ans 8. Let 16 years ago, the age of Sudha and her grandfather were x and
8x years respectively; then

Ans 9. Here, ( A+B) - ( B + C) = 12 A -C = 12


Hence,C is 12 years younger than A.

www.bankexamstoday.com Page 241


Ans 10. Let the age youngest child be x years ; then
x + ( x + 3) + x + 6) + x+ 9) + ( x + 12) = 50 => 5x =
20 . x = 4 years

Ans 11. Let the age of father was x years at the time of his son's birth then
present age of father and his son will be 2x and x years.
Now, 2x = 38 . x = 19 years
Hence, 5 years ago the age of son was 19 -5 = 14 years.

Ans 12. The difference of ages of her parents = ( 38 + 4) - 36 = 43 -36 =


6 years.

Order and Ranking

Directions: To find out the position of a person in the row from L.H.S/R.H.S
(To find out the position of a person in a row i.e., either from top/bottom)
The position of a person from L.H.S/top = Total number of people in a row + 1
– Position of the same person from R.H.S/bottom.
The position of a person from R.H.S/bottom = Total number of people in a row
+ 1 – Position of the same person from L.H.S/top.

Que.1. There are 12 persons in a row. The position of Vipul is 7th from left.
What is his position from the right end?

Que.2.The position of Sita is 11th from the top in a column of 50 students.


What is her position from the bottom?

Direction: To find out the total number of persons in the row

CASE 1: Here the position of a person from L.H.S as well as R.H.S is already
given, and we are asked to find out the number of persons in the row
A total number of persons in a row = Position of a person from L.H.S/top +
Position of the same person from R.H.S/bottom – 1.

www.bankexamstoday.com Page 242


Que.3.Vishal is standing in a row. His position from the top is 6th and his
position from the bottom is 12th.How many people are there in a row?

CASE-2:
We need to find out the total number of a person in the row when - Position
of one person is given from the left end and of the other is given from the
right end. The number of persons in between them is also given.

Que.4.In a row, the position of Rahul is 10th from left in a row. Kajol is 17th
from the right. Prem is ahead of Rahul by four positions and between Prem and
Kajol, there are 6 persons between them. What is the total number of persons
in the row?

CASE-3: Position of one is given from the left end and of the other is given
from the right end. The position of persons in between them is also given.

Que.5.The position of Farhaan is 15th from left in a row. Sushant is 20th from
the right end. There are 6 persons in between them. What is the total number
of persons in the row?

CASE-4: Position of one is given from the left end and of the other is given
from the right end. The total number of persons in the row is also given. We
have to determine the number of persons in between them.

Que.6.The position of Sushma is 7th from the left in a row. Bhisma is


23rd from the right end. There is total of 57 persons in the row. What is
the total number of persons in between them?

CASE-5: We need to obtain the total number of persons sitting in a row


when two people interchange their position.

Que.7.James is 14th from left and Nancy is 25th from the right. They
interchange their positions. Now James is 39th from left and Nancy is 50th from
the right. What is the total number of persons in the row?

Que.8.Rohit is 7 ranks ahead of Karan in a class of 39. If Karan‘s rank is


17th from the last, what will be Rohit‘s rank from the start?
a) 14th

www.bankexamstoday.com Page 243


b) 15th
c) 16th
d) 17th
e)None of these

Que.9. In a class of 39 students, the ratio of boys and girls is 2:1. Akruthi
ranks 15th among all the students from top and 8th among girls from the
bottom. How many boys are there below Akruthi?
a) 16
b) 17
c) 15
d) Data Inadequate
e) None of these

Que10. In a queue, Roshan is 14th from the front and Jeelani is 17th from
the end, while Aysha is in between them. If Roshan is ahead of Jeelani and
there are 48 persons in the queue. How many persons are there between
Roshan and Aysha?
a) 8
b) 7
c) 6
d) 7
e) None of these

Que.11. In a row of 40 boys when Kushal was shifted to his left by 4 places,
his place from the left end of the row became 10. What is the position of
Suraj from the right end of the row, if Suraj was three places to the right of
Kushal‘s original position?
a) 22
b) 23
c) 25
d) 24
e) None of these

Que.12. In a row of 25 children facing South, R is 16th from the right end
and B is 18th from the left end. How many children are there between R and
B?
a) 2
b) 4

www.bankexamstoday.com Page 244


c) 3
d) Data inadequate
e) None

Que.13. In a row of 35 children, M is 15th from the right end and there are
10 children between M and R. What is R‘s position from the left end of the
row?
a) 15th
b) 5th
c) 30th
d) Data inadequate
e) None

Que.14. In a row of 40 children, Q is 14th from the left end and there are
16 children between Q and M. What is M‘s position from the right end of the
row?
a) 11th
b) 10th
c) 30th
d) Data Inadequate
e) None

Que.15. Three persons A, B and C are standing in a queue. There are 5


people between A and B and 8 people between B and C. If there are three
people ahead of C and 21 people behind A, then what could be the minimum
number of persons in the queue?
a) 27
b) 28
c) 40
d) 41

Que.16. In a row of children facing North, Sheila is shifted to her right by


four places becomes 18th from the right end of the row. Sahil, who is 15th
from the left end of the row, is 5th to the left of Sheila. How many children
are there in the row?
a) 42
b) 38
c) 41
d) 39

www.bankexamstoday.com Page 245


e) None of these

Que.17. In a row of boys facing North, Rinku is 10th to the left to Pink who
is 21st from the right end. If Miku who is 17th from the left end, is 4th to
the right of Rinku, then how many boys are there in the row?
a) Data inadequate
b) 44
c) 37
d) 43
e) None
Que.18.Unita is 11th from either end of a row of girls. How many girls are
there in that row?
a) 19
b) 20
c) 21
d) 22
e) 24

Que.19. Raja walks slower than Raghu and Raghu walk as fast as Guru and
Krishna walk faster than Guru. Who walks the fastest?
a) Raghu
b) Raja
c) Krishna
d) Both Raghu and Guru

Directions: Among A, B, C and D, B is heavier than A and C but C is taller


than him. D is not as tall as C while A is shortest. C is not as heavy as A.
D is heavier than B but shorter than him.

Que.20. Who is the heaviest?


a) B
b) A
c) D
d) C
e) Cannot be determined

Que.21. Who is the tallest?


a) D
b) C
c) Either A or D

www.bankexamstoday.com Page 246


d) B
e) Cannot be determined

Que.22. P, Q, R and S are four males. P is the eldest in the group but he is
not the poorest. R is the richest but not the eldest, Q is elder than S but he
is not the eldest, Q is poorer than S. How can the four persons be arranged
in descending order of their age and money?
a) PQRS, RPSQ
b) PRQS, RSPQ
c) PRQS, RSQP
d) PRSQ, RSPQ
e) None of these

Five people I, J, K, L and M type at different speeds. I type faster than L


but slower than J. M types slower than L but is not the slowest. The one
who types the second fastest, types at a speed of 75 words/minute.
The one who types the second slowest types at the speed of 50
words/minute.

Que.23. How many people type slower than I?


a) 4
b) 1
c) 2
d) 3
e) None of these

Que.24. On the given information, which of the following statements is


true?
a) J types the fastest
b) K types faster than L
c) No statement is true
d) I type at a speed of 50 words/minutes
e) M types faster than only two persons

Que.25. Which of the following can be L‘s speed?


a) 77 words/minute
b) 60 words/minute
c) 30 words/minute
d) 45 words/minute
e) 85 words/minute

www.bankexamstoday.com Page 247


Directions: Among five persons – P, Q, R, S and T, each has different
height. Only two persons are shorter than S. T is shorter than S but
taller than R. The one who is the second tallest among them is of 158
cm.

Que.26. Which of the following statement is definitely true with respect to


the given information?
a) R is definitely 150cm
b) P is possibly 153cm
c) T is shorter than S
d) Q is shorter than S
e) None of the above

Que.27. Which of the following is the height of Q?


a) Cannot be determined
b) 156cm
c) 152cm
d) 150cm
e) 158cm

Que.28.How many persons are shorter than Q?


a) 2
b) 3
c) 4
d) Cannot be determined
e) 1

Answers

Ans.1.
→Total number of people = 12
→Position of Vipul from the left end = 7th
→Position of Vipul from the right end = Total number of people in a row + 1 –
position of Vipul from →the left end
= 12 +1 – 7
=6
→Therefore, Vipul is 6th from the right end

www.bankexamstoday.com Page 248


Ans.2.→Total number of people = 50
→The position of Sita from top = 11th
→The position of side from bottom = (Total number of people + 1 – position of
Sita from top)
= 50 + 1 – 11
= 40
ஃTherefore, the position of Sita from the bottom = 40th

Ans.3.→Position of Vishal from both sides is provided to us.


So,
→A total number of persons in a row = Position of Vishal from Top + Position of
Vishal from →Bottom – 1.
→Total number of people in a row = 6th + 12th – 1
= 17

www.bankexamstoday.com Page 249


Ans.4.
Case – 1
→Position of Rahul = 10th from left
→Position of Kajol = 17th from right
→Position of Prem from Rahul = 4
→Persons between Prem and Kajol =6
→Total number of people in a row = 10+17+4+6 = 37

Case – 2
→Total number of people in a row = 7 + 17 = 24

Ans.5.
Case – 1
→Position of Farhaan = 15th from the left end
→Position of Sushant = 20th from right end

www.bankexamstoday.com Page 250


→Persons in between them = 6
→Total number of people in the row = 15 + 20 + 6 = 41

Case – 2
→Total number of people in the row = 7 + 20 = 27

Ans.6.→Position of Sushma = 7th from left


→Position of Bhisma = 23rd from right
→Total number of people in the row = 57
→Number of people between Sushma and Bhisma = 57 – ( 23 + 7 ) = 27

Ans.7.

→As we have discussed earlier, to determine the total number of people, we


need the L.H.S and R.H.S value of the same person.
→If we consider position-1, we observe that James is 14th from left and Nancy

www.bankexamstoday.com Page 251


is 50th from the right. →Nancy is 50th from right after interchanging their
positions. This implies that the initial position of →James from the right end was
50th.
→A total number of persons in a row = Position of James from left + Position of
James from right – 1.
14 + 50 – 1 = 63
→The Same result is obtained if we look at the position-2.
→Nancy was 25th from right and now at the same position, James is 39th from
left. So, we simply add up both the rankings i.e., from left and right and reduce
by 1.
25 + 39 – 1 = 63
→Therefore, the total number of people in the row = 63.

Ans.8.→Given Karan is 17th from last


→Rohit is 7 ranks ahead of Karan. Since, the position of Karan is given from the
last, Rohit rank from last is
17 + 7 = 24
→Position of Rohit from the start = Total strength of the class + 1 - Position of
Rohit from the bottom
= 39 + 1 – 24
=16th

Ans.9.

There are 26 boys and 13 girls

www.bankexamstoday.com Page 252


→ Among 13 girls, 7 girls are above Akruthi. So, the remaining girls should be
below her i.e., (13 – 8 ) = 5 girls are below her.
→ Since there are only 7 girls above Akruthi, the remaining 7 places are
occupied by boys. So, the remaining boys will be below Akruthi.
→ Below Akruthi, there are 39-15 = 24 students. Among those 24 students, the
number of boys below Akruthi is
24 – 7 = 17

Ans.10.

→The number of people between Roshan and Jeelani = 48 – (14 + 17 ) = 17


person
→Now, Aysha is exactly between Roshan and Jeelani.
→Therefore, there are 8 persons between Roshan and Aysha.

Ans.11.→On shifting 4 positions to the left, Kushal is 10th from the left end.
→So, initially, he was at 14th position from the left end, which implies that
Suraj is at 17th position from the left end.
→The position of Suraj from the right end = Total number of boys + 1 –

www.bankexamstoday.com Page 253


Position of Suraj from left end
= 40 + 1 – 17 = 24

Ans.12.

→To determine the number of people between B and R, = Total number of


people – ( B‘s position + R‘s position ) =25 – ( 18 + 16 ) = - 9
→Since the number of people can never be negative. Let us find out the position
of B and R from the other end.
→Position of R from L.H.S = Total number of students + 1 – Position of R from
R.H.S = 25 + 1 – 16 = 10
→Position of B from the R.H.S = Total number of students + 1 – Position of B
from L.H.S = 25 + 1 – 18 = 8

→Number of students between B and R = Total number of students – ( B‘s


position + R‘s position )
= 25 – ( 10 + 8 )
=7

Ans.13.→If we observe the question clearly, we notice that there are two
possible cases
→In one case, R might be right of M and in another case, R might be left of M.

www.bankexamstoday.com Page 254


→If R is towards the right side of M, then it will be in 5th position from the right
end.
→Position of R from L.H.S = 35 + 1 – 5 =31st
→If R is towards the left side of M, then it will be on 25th position from the right
end.
→Position of R from L.H.S = 35 +1 – 25 =11th
→Since R can be either 31st or 11th position from the left end. Option (d) is the
correct option.

Ans.14.→Since Q is 14th from left; there is no chance for M to be left of Q.

→M‘s position from the left end = 14 + 16 + 1


= 31st
→M‘s position from the right end = Total no of children + 1 – M‘s position from
the left end
= 40 + 1 – 31
= 10th

Ans.15.→As per the given conditions, there are two possible arrangements as
shown below:

www.bankexamstoday.com Page 255


→The number of people in the queue = 21 + 1 + 5 +1 +8 + 1 + 3 =40

→The number of people in the queue = 3 + 1 +2 + 1 + 21 = 28


Clearly, for minimum number of people, we should consider Case-2.

Ans.16.→After shifting the position of Sheila, her position is 18th from the
right end. So, initially, Sheila was 15th from the right end. Saahil is 5th to the
left of Sheila.

→Total number of children in a row = 15 + 15 + 4


= 34

Ans.17.Given,
→Pink is 21st from the right end and Rinku is 10th to the left of Pink
→So, Rinku is 31st (i.e., 21 + 10 ) from the right end.
→Miku is 17th from the left end and 4th to the right of Rinku.
→So, Rinku‘s position from the left end with respect to Miku = 14th
→Total number of boys = Position of Rinku from R.H.S + Position of Rinku from
L.H.S – 1
= 31 + 14 – 1
= 43

Ans.18.→Total number of girls in row = Position of Unita from L.H.S + Position


of Unita from R.H.S - 1
= 11 + 11 -1
= 21

Ans.19.

www.bankexamstoday.com Page 256


Ans.(20-21).

Ans. 22.

Ans. (23-25).

Ans. (26-28).

www.bankexamstoday.com Page 257


→The height of Q will be greater than or equal to 158cm

www.bankexamstoday.com Page 258

You might also like